FAR Missed MC Questions

Ace your homework & exams now with Quizwiz!

Kale Co. purchased bonds at a discount on the open market as an investment and intends to hold these bonds to maturity. Kale should account for these bonds at: a. Cost b. Amortized cost c. Fair value d. Lower of cost or market

B Bond investments which are intended to be held until the maturity date are classified as held-to-maturity securities and are reported at their amortized cost

Jones Wholesalers stocks a changing variety of products. Which inventory costing method will be most likely to give Jones the lowest ending inventory when its product lines are subject to specific price increases? a. Specific identification b. Dollar-value LIFO c. Weighted average d. FIFO periodic

B Dollar value LIFO most likely gives the lowest ending inventory when product lines are subject to specific price increases

When a company declares a cash dividend, retained earnings is decreased by the amount of the dividend on the date of: a. Declaration or record, whichever is earlier b. Declaration c. Record d. Payment

B When a company declares a cash dividend, retained earnings is decreased by the amount of the dividend on the date of declaration by the board of directors

Goodwill should be tested for value impairment at which of the following levels? a. Entire business as a whole b. Each acquisition unit c. Each reporting unit d. Each identifiable long-term asset

Choice "3" is correct. U.S. GAAP requires that goodwill be tested for impairment at the reporting unit level. The evaluation of goodwill impairment involves comparing the carrying amount and the fair value of the reporting unit. If the carrying amount of the reporting unit (including goodwill) exceeds the fair value of the reporting unit (including goodwill), an impairment loss in the amount of the difference must be booked. The loss is capped at the amount of goodwill currently on the books.

Mio Corp was the sole stockholder of Plasti Corp. On September 30, Mio declared a property dividend of Plasti's 2,000 outstanding shares of $1 par value common stock, distributable to Mio's stockholders. On that date, the book value of Plasti's stock was $1.50 per share. Immediately after the distribution, the market value of Plasti's stock was $4.50 per share. What amount should Mio report in its financial statements as gain on disposal of the Plasti's stock? a. $1,000 b. $3,000 c. $2,000 d. $6,000

Choice "4" is correct. $6,000 gain on disposal of Plasti stock. Rule: The difference between book value and fair market value of the property dividend should be recorded as gain/loss on disposal of asset. Per-sharevalue2,000 sharestotal value FMV of stock at declaration 4.50 9,000 Book value of Plasti stock investment (1.50) (3,000) Gain on disposal of stock 3.00 6,000

On January 2, Judd Co. bought a trademark from Krug Co. for $500,000. Judd retained an independent consultant, who estimated the trademark's remaining life to be 50 years. Its unamortized cost of Krug's accounting records was $380,000. In Judd's December 31 balance sheet, what amount should be reported as accumulated amortization? a. $7,600 b. $12,500 c. $9,500 d. $10,000

D 500k/50

When a snowplow purchased by a governmental unit is received, it should be recorded in the general fund as a(n) a. Appropriation b. Capital Asset c. Encumbrance d. Expenditure

D Expenditures are decreases in net financial resources other than through interfund transfers. General fixed assets, such as land, buildings, and equipment, purchased with general fund resources should be recorded as expenditures in the general fund.

Which of the following is not a common modification used to prepare modified cash basis financial statements? a. Capitalizing inventory b. Recording long-term liabilities c. Accrual of income taxes d. Recognizing revenues when earned

D If revenues are recognized when earned, rather than when received, then the financial statements are prepared using the accrual basis. Recording long-term liabilities, accrual of income taxes, and capitalization of inventory are all common modifications made to cash basis financial statements

Which U.S. GAAP inventory costing method would a company that wishes to maximize profits in a period of rising prices use? a. Moving Average b. Dollar-value LIFO c. Weighted Average d. FIFO

D Under FIFO, the first cost inventories are the first costs transferred to cost of goods sold. In a period of rising prices, FIFO results in the lowest COGS and the highest net income

The equity method would be used if a company owned what percentage of its investee company's stock? a. 15% b. 75% c. 5% d. 25%

D The equity method is appropriate for use when an investor exercises significant influence over an investee. Although "significant influence" may be evident based on qualitative factors, the general rule is that a company that owns between 20 to 50 percent of the voting stock of another investee company is able to exercise significant influence. In this question, 25 percent ownership would indicate significant influence

Cash receipts from grants and subsidies to decrease operating deficits should be classified in which of the following sections of the statement of cash flows for governmental, not-for-profit entities? a. Noncapital financing b. Operating c. Capital and related financing d. Investing

A Cash flows from nonfinancing activities include: -Proceeds from payments to borrowing not attributable to the acquisition, construction or improvement of capital assets -Cash receipts or payments related to grants or subsidies not attributable to capital purposes -Property taxes not designated for capital resources -Cash paid to other funds (other than for interfund services)

Scott Co. exchanged similar nonmonetary assets with Dale Co. and no cash was exchanged. The carrying amount of the asset surrendered by Scott exceeded both the fair value of the asset received and Dale's carrying amount of that asset. Scott should: a. Recognize the difference between the carrying amount of the asset it surrendered and the fair value of the asset it surrendered as a loss. b. Recognize the difference between the carrying amount of the asset it surrendered and the fair value of the asset it received as a gain. c. Recognize the difference between the carrying amount of the asset it surrendered and the carrying amount of the asset it received as a loss. d. Recognize no gain or loss.

A In all nonmonetary transactions, the fair value given is equal to the fair value received. Therefore, the carrying amount of the asset surrendered must exceed the fair value of the asset surrendered, which means Scott has a loss on the transaction. Losses on nonmonetary transactions must be recognized in full

Which of the following statements regarding the presentation guidelines for other comprehensive basis of accounting (OCBOA) financial statements is correct? a. Financial statement titles should differentiate OCBOA financial statements from accrual basis financial statements b. OCBOA financial statement disclosures are significantly less detailed than GAAP financial statement disclosures c. An explanation of changes in equity is not required because OCBOA financial statements do not report equity interests d. OCBOA financial statements should include financial statements equivalent to the accrual basis balance sheet, income statement and statement of cash flows

A OCBOA financial statement titles should differentiate the financial statements from accrual basis financial statements

Land was purchased to be used as the site for the construction of a plant. A building on the property was sold and removed by the buyer so that construction on the plant could begin. The proceeds from the sale of the building should be: a. Deducted form the cost of the land b. Netted against the costs to clear the land and expensed as incurred c. Netted against the costs to clear the land and amortized over the life of the plant d. Classified as other income

A The proceeds from the sale of the building should be deducted from the cost of the land Rule: Cost of land includes all necessary to put the land in place and condition for construction of the plant. Any proceeds from the sale of any existing buildings (or standing timber, or soil) or scrap are deducted from the cost of the land

Long Co. invested in marketable securities. At year-end, fair-value changes in this investment were included in Long's other comprehensive income. How would Long classify this investment? a. Available-for-sale debt securities b. Trading debt securities c. Held-to-maturity securities d. Equity securities

A Unrealized gains and losses from marking available for sale securities to fair value (assuming they are non-impaired) at the balance sheet date are treated as other comprehensive income items and bypass the income statement

A corporation entered into a purchase commitment to buy inventory. At the end of the accounting period, the current market value of the inventory was less than the fixed purchase price, by a material amount. Which of the following accounting treatments is most appropriate? a. Describe the nature of the contract in a note to the financial statements, recognize a loss in the income statement, and recognize a liability for the accrued loss c. Neither describe the purchase obligation, nor recognize a loss on the income statement or balance sheet d. Describe the nature of the contract and the estimated amount of the loss in a note to the financial statements, but do not recognize a loss in the income statement

A When the current market value of the inventory is less than the fixed purchase price in a purchase commitment, the loss must be recognized at the time of the decline in price, a liability must be recognized on the balance sheet and a description of the losses must be described in the footnotes

On December 1, Year 1, Tom V. Company entered into an operating lease for office space for its executives for 10 years at a monthly rental of $200,000, increasing to $400,000 halfway through the lease. On that date, Tom V. paid the landlord the following amounts: First months rent $200,000 Last Months Rent $400,000 Installation of new carpet $600,000 The entire amount was charged to rent expense in Year 1. What amount should Tom V. have charged as rent expense for the year? a. $305,000 b. $300,000 c. $200,000 d. $1,200,000

A Amortization of carpet plus average rent

At December 31, Year 1, Eagle Corp. reported $1,750,000 of appropriated retained earnings for the construction of a new office building, which was completed in Year 2 at a total cost of $1,500,000. In Year 2, Eagle appropriated $1,200,000 of retained earnings for the construction of a new plant. Also, $2,000,000 of cash was restricted for the retirement of bonds due in Year 3. In its Year 2 balance sheet, Eagle should report what amount of appropriated retained earnings? a. $1,450,000 b. $1,200,000 c. $2,950,000 d. $3,200,000

B 1.2 mil appropriated retained earnings at Dec 31, Year 2 (for the construction of a new plant only). When the purpose of the appropriation has been achieved it should be restored to unappropriated retained earnings.

On July 1, Year 1, Cobb Company issued 9% bonds in the face amount of $1,000,000 which mature in 10 years. The bonds were issued for $939,000 to yield 10%, resulting in a bond discount of $61,000. Cobb uses the effective interest method of amortizing bond discount. Interest is payable annually on June 30. At June 30, Year 3, Cobb's unamortized bond discount would be: a. $43,000 b. $52,810 c. $57,100 d. $48,800

B 61k 939k * .1 = (93,900-90000) = 3900 rollforwards = 939k + 3900 = 942,90 942,900 * .1 = 94,290 - 90k = 4,290 61 - 3900 - 4290 = 52810

Which of the following financial instruments issued by a public company should be reported on the issuer's books as a liability on the date of issuance? a. Cumulative preferred stock b. Common stock that contains an unconditional redemption feature c. Preferred stock that is convertible to common stock five years from the issue date d. Common stock that is issued at a 5% discount as part of an employee share purchase plan

B Common stock that contains an unconditional redemption feature should be reported on the issuer's books as a liability on the date of issuance because there is an obligation of a cash outflow in the future that the company has no ability to prevent

On January 1, Year 1, a company granted some of its key employee stock options for 100,000 shares of $3 par common stock when the fair value of each option was $6 per share. The options vest after three years of service. What is the compensation expense, if any, for the year ended December 31, Year 1? a. $0 b. $200,000 c. $100,000 d. $600,000

B Compensation expense for stock options is derived based on fair value and is allocated over the service period, which represents the period from the grant date to the vesting date

Farleigh Co. has not declared or paid dividends on its cumulative preferred stock in the last three years. These dividends should be reported: a. As a current liability b. In a note to the financial statements c. As a reduction in stockholder's equity d. As a non-current liability

B Dividend's in "arrears" (undeclared dividends on cumulative preferred stock) should be reported in the footnotes. Since they are not declared, no journal entry is made. Neither liabilities nor equity are affected

Which of the following types of assets would typically be reported on a company's balance sheet as an intangible asset? a. Leasehold improvements b. Cost of patent registrations c. Derivative securities d. Cost of research and development

B Patents are intangible assets. Legal and registration fees incurred to obtain a patent are capitalized and reported as an asset

If a corporation sells some of its treasury stock at a price that exceeds its cost, this excess should be: a. Reported as gain in the income statement b. Credited to additional paid-in capital c. Credited to retained earnings d. Treated as a reduction in the carrying amount of remaining treasury stock

B Rule: There is no gain or loss on the purchase and/or sale of treasury stock. Any "difference" goes to "paid-in capital", or if there is not enough paid-in capital to absorb a loss, the loss would be debited (subtracted) from RE

Gown Inc. sold a warehouse and used the proceeds to acquire a new warehouse. The excess of the proceeds over the carrying amount of the warehouse sold should be reported as a: a. Reduction of the cumulative depreciation expense on the old warehouse b. Part of continuing operations c. Reduction of the cost of the new warehouse d. Gain from discontinued operations, net of income taxes

B Rule: When a fixed asset is sold, gain or loss is recognized as part of income from continuing operations. The amount of the gain or loss is equal to the difference between the proceeds from the sale and the carrying amount of the fixed asset sold

Which of the following statements would most likely be included among a set of financial statements prepared in conformity with a special purpose framework? a. The statement of comprehensive income b. The statement of cash receipts and disbursements c. The statement of financial position d. The statement of operations

B Special purpose frameworks are non-GAAP presentations that include other bases of accounting, such as the cash basis and modified cash basis. The statement of cash receipts and disbursements is an example of a cash basis income statement

In a compensatory stock option plan for which the grant and exercise dates are different, the stock options outstanding account should be reduced at the: a. Date of grant b. Exercise date c. Beginning of the vesting period d. Beginning of the service period

B Stock options outstanding are reduced at the exercise date

On June 1, Year 2, Archer, Inc. issued a purchase order to Cotton Co. for a new copier machine. The machine requires one month to produce and is shipped f.o.b. destination on July 1, Year 2, and is received by Archer on July 15, Year 2. Cotton issues a sales invoice dated July 2, Year 2, for the machine. As of what date should Archer record a liability for the machine? a. July 1, Year 2 b. July 15, Year 2 c. June 1, Year 2 d. July 2, Year 2

B The machine should be recorded as a liability when Archer Inc. has legal title to the machine. According to the shipping terms of FOB Destination, title does not pass until the machine is received at its destination by Archer Inc. Therefore, the asset and related liability will be recorded on July 15, Year 2.

As of December 15, Year 3, Aviator had dividends in arrears of $200,000 on its cumulative preferred stock. Dividends for Year 3 of $100,000 have not yet been declared. The board of directors plan to declare cash dividends on its preferred and common stock on January 16, Year 4. Aviator paid an annual bonus to its CEO based on the company's annual profits. The bonus for Year 3 was $50,000, and it will be paid on February 10, Year 4. What amount should Aviator report as current liabilities on its balance sheet at December 31, Year 3? a. $200,000 b. $50,000 c. $150,000 d. $350,000

B Under the matching principle, expenses are recognized when an entity's economic benefits are used up. In the case of the CEO bonus of $50,000, that was clearly earned in Year 3 and should be accrued for accordingly. The dividends were not declared in Year 3 and as such should not be accrued for. Under declaration, dividends become a debt of the corporation (credit to dividends payable) and are debited to retained earnings

On October 1, 20X4, Host Co. approved a plan to dispose of one of the company's operating segments. The decision represents a strategic shift for Host and will have a significant effects on its operations and financial results. Host expected that the sale would occur on April 1, 20X5 at an estimated gain of $350,000. The segment had actual and estimated operating losses as follows: 1/1/X4 to 9/30/X4 $(300,000) 10/1/X4 to 12/31/X4 $(200,000) 1/1/X5 to 3/31/X5 $(400,000) In its 20X4 income statement, what should Host report as a loss from discontinued operations before income taxes? a. $200,000 b. $500,000 c. $550,000 d. $900,000

B In its 20X4 Income statement, Host would include in its loss from discontinued operations the 20X4 losses but not the 20X5 operating losses and not the projected gain or disposal. The 20X4 losses are $500,000 in total. The projected 20X5 operating loss of $400,000 and the projected gain on disposal would be included in its 20X5 income statement

Bonds issued with scheduled maturities at various dates are called: Serial bonds Term bonds a. N N b. Y N c. N Y d. Y Y

B Serial bonds are those issued with scheduled maturities at various dates Term bonds are issued with a single fixed maturity date

Under U.S. GAAP, a gain that is both unusual and infrequent should be reported as which of the following? a. Comprehensive income b. Income from continuing operations c. Income from continuing operations, net of tax d. Net of tax, following discontinued operations

B Under U.S. GAAP, items that are both unusual and infrequent are reported as a separate component of income from continuing operations

When debt is issued at a discount, interest expense over the term of debt equals the cash interest paid: a. Plus discount plus par value b. Plus discount c. Minus discount minus par value d. Minus discount

B When debt is issued at a discount, interest expense over the term of the debt equals the cash interest paid plus amortization of the discount

The estimated revenues control account balance of a governmental fund type is eliminated when: a. The budgetary accounts are closed b. Appropriations are closed c. Property taxes are recorded d. The budget is recorded

Choice "1" is correct. The estimated revenues control account is eliminated when the budgetary accounts are closed. Choice "2" is incorrect. Appropriations (budgeted expenditures) are a separate control account from the estimated revenues control account. Choice "3" is incorrect. When property taxes are recorded, the actual revenue account is increased and there is no effect on the estimated revenues control account. Budgetary accounts are altered only when there is an approved change in the budget. Choice "4" is incorrect. The estimated revenues control account is created when the budget is recorded.

What is the measurement focus and the basis of accounting for the government-wide financial statements? Measurement Focus Basis of Accounting a. Current Financial Resources Accrual b. Economic Resources Accrual c. Economic Resources Modified accrual d. Current financial resources Modified accrual

Choice "2" is correct. The government-wide financial statements used the economic resources measurement focus and the accrual basis of accounting.

Newt Co. sold a warehouse and used the proceeds to acquire a new warehouse. The excess of the proceeds over the carrying amount of the warehouse sold should be reported as a(an): a. Gain from discontinued operations, net of income taxes b. Part of continuing operations c. Reduction of the cost of the new warehouse d. Part of other comprehensive income

Choice "2" is correct. The net gain from the sale of a warehouse and purchase of a new warehouse will fall under continuing operations on the income statement, under "other" revenues and gains. Choice "1" is incorrect. The sale of a warehouse and consequent replacement with a new warehouse do not qualify as discontinued operation. Choice "3" is incorrect. The fact that the proceeds exceeded the carrying amount of the warehouse sold would not serve to reduce the cost basis of the new warehouse. Choice "4" is incorrect. The gain is a normal operating item and is not reported in other comprehensive income.

Line Co's worksheet for the preparation of its current year statement of cash flows included the following: December 31 January 1 AR 29k 23k ADA 1k 800 Prepaid Rent 8,200 12,400 AP 22,400 19,400 Lino's net income is $150,000. What amount should Lino include as net cash provided by operating activities in the statement of cash flows? a. $151,400 b. $151,000 c. $148,600 d. $145,400

Choice "1" is correct. Cash flows from operating activities: Net income 150,000 Adjustments to reconcile net income to net cash provided by operating activities: Change in current assets and liabilities: Increase in accounts receivable (net) (5,800) Decrease in prepaid rent expense 4,200 Increase in accounts payable 3,000 Total adjustments 1,400 Net cash provided by operating activities 151,400 Choice "2" is incorrect. $151,000 is calculated by adding the allowance for doubtful accounts to the accounts receivable instead of subtracting it. The increase in accounts receivable would have been $6,200 instead of $5,800. Choice "3" is incorrect. $148,600 is calculated by reversing what was done with each of the increases/decreases in the question. The total adjustments would have been a decrease of $1,400 instead of an increase of $1,400. Choice "4" is incorrect. $145,400 is calculated by reversing what was done with the accounts payable change. The increase in accounts payable would have been subtracted instead of added.

Troop Co. frequently borrows from the bank to maintain sufficient operating cash. The following loans were at a 12% interest rate, with interest payable at maturity. Troop repaid each loan on its scheduled maturity date. Date of Maturity Term Loan Amount Date of Loan 11/1/Yr 3 $10,000 10/31/Yr 4 1 year 2/1/Yr 4 30,000 7/31/Yr 4 6 months 5/1/Yr 4 16,000 1/31/Yr 5 9 months Troop records interest expense when the loans are repaid. Accordingly, interest expense of $3,000 was recorded in Year 4. If no correction is made, by what amount would Year 4 interest expense be understated? a. $1,080 b. $1,440 c. $1,280 d. $1,240

Choice "1" is correct. $1,080 understated interest expense. Actual interest expense: DateAmount Rate Time 11/1/ Year 1 Loan $10,000 x 12% x 10/12 mos = $1,000 2/1/ Year 2 Loan 30,000 x 12% x 6/12 mos = 1,800 5/1/ Year 2 Loan 16,000 x 12% x 8/12 mos = 1,280 4,080 Stated interest expense on the cash basis 3,000 Understated interest expense $1,080

Winn Co. manufactures equipment that is sold or leased. On December 31, Year 1, Winn leased equipment to Bart for a 5-year period ending December 31, Year 6, at which date ownership of the leased asset will be transferred to Bart. Equal payments under the lease are $22,000 (including $2,000 executory costs) and are due on December 31 of each year. The first payment was made on December 31, Year 1. Collectibility of the remaining lease payments is reasonably assured, and Winn has no material cost uncertainties. The normal sales price of the equipment is $77,000, and cost is $60,000. On December 31, Year 1, what amount of income should Winn realize from the lease transaction? a. $17,000 b. $22,000 c. $23,000 d. $33,000

Choice "1" is correct. $17,000 in income recognized for the year ending December 31, Year 1. Rule: In a finance lease, the difference between the fair value of the leased asset and its cost at inception is recognized as a gain or loss. Cash selling price of equipment 77,000 Less cost of equipment at inception 60,000 Profit recognized on sale 17,000

On January 2 of the current year, Kine Co. granted Morgan, its president, compensatory stock options to buy 1,000 shares of Kine's $10 par common stock. The options call for a price of $20 per share and are exercisable for three years following the grant date. Morgan exercised the options on December 31 of the current year. The market price of the stock was $45 on January 2 and $70 on December 31. Using an acceptable options pricing model, Morgan determined that the fair value of the options granted was $30,000. By what net amount should stockholder's equity increase as a result of the grant and exercise of the options? a. $20,000 b. $70,000 c. $30,000 d. $50,000

Choice "1" is correct. $20,000 increase in stockholders' equity. Compensation cost should be charged to expense over the service period. In this problem, because the options are exercised in the same period as the grant date, the total compensation cost must be charged to expense in Year 1. Effect on Stockholders' Equity DRCREquity Jan. 2, Year 1 (when options granted) Compensation expense* 30,000 (30,000) Paid-in capital-stock-options-outstanding 30,000 30,000 Dec. 31, Year 1 (when options exercised) Cash ($20 × 1,000) 20,000 Paid-in capital-stock-options-outstanding 30,000 (30,000) Common stock at par ($10 × 1,000) 10,000 10,000 Paid in capital in excess of par (squeeze) 40,000 40,000 Net effect on stockholders' equity 20,000 * Note: A charge to expense lowers retained earnings.

House Publishers offered a contest in which the winner would receive $1,000,000, payable over 20 years. On December 31, Year 1, House announced the winner of the contest and signed a note payable to the winner for $1,000,000, payable in $50,000 installments every January 2. Also on December 31, Year 1, House purchased an annuity for $418,250 to provide the $950,000 prize monies remaining after the first $50,000 installment, which was paid on January 2, Year 2. In its December 31, Year 1, balance sheet, what amount should House report as note payable-contest winner, net of current portion? a. $468,250 b. $0 c. $418,250 d. $1,000,000

Choice "1" is correct. $468,250 contest prize expense. First payment on 1/2/Year 2 50,000 Present value of 19 subsequent payments 418,250 Contest prize expense in Year 1 income statement 468,250

Doren Co.'s officers' compensation expense account had a balance of $490,000 at December 31, 20X4, before any appropriate year-end adjustment relating to the following: No Salary accrual was made for the week of December 25-31, 20X4. Officers' salaries for this period totaled $18,000 and were paid on January 5, 20X5. Bonuses to officers for 20X4 were paid on January 31, 20X5 in the total amount of $175,000. The adjusted balance for officers' compensation expense for the year ended December 31, 20X4, should be: A. $683,000. B. $665,000. C. $508,000. D. $490,000.

Choice "1" is correct. $683,000 compensation expense for year ended Dec. 31, Year 1. Compensation exp balance before year-end adjustments 490,000 Add: Salary accrual for week of Dec. 25-31, Year 1 not paid until Jan. 5, Year 2 18,000 Add: Year 1 officers' bonuses not paid until Jan. 31, Year 2 175,000 Compensation exp after year-end adjustments 683,000

Instead of the usual cash dividend, Evie Corp. declared and distributed a property dividend from its overstocked merchandise. The excess of the merchandise's carrying amount over its market value should be: a. Reported as a reduction in income before income from continuing operations b. Reported as a separately disclosed reduction of retained earnings c. Reported as a separate item on the income statement, net of income taxes d. Ignored

Choice "1" is correct. A loss is recognized for the merchandise's carrying amount over its market value. This results in a reduction in income from continuing operations. Rule: Dividends declared and paid in the form of assets other than cash are recorded by the distributing corporation at fair market value at date of declaration. Choice "2" is incorrect. The loss cannot be charged directly to retained earnings as it is not a prior period adjustment or an accounting change. Choice "3" is incorrect. The loss is not reported net of tax and would not be reported separately unless it was unusual or infrequent and material in amount. Choice "4" is incorrect. The loss must be recognized; it cannot be ignored.

Within the context of the qualitative characteristics of accounting information, which of the following is a fundamental qualitative characteristic? a. Relevance b. Timeliness c. Comparability d. Feedback value

Choice "1" is correct. As defined in the Statements of Financial Accounting Concepts, relevance and faithful representation are the two fundamental qualitative characteristics of accounting information. Choice "2" is incorrect. According to the Statements of Financial Accounting Concepts, timeliness is an enhancing qualitative characteristic. These enhance the usefulness of relevant, faithfully represented information. Choice "3" is incorrect. According to the Statements of Financial Accounting Concepts, comparability is an enhancing qualitative characteristic. These enhance the usefulness of relevant, faithfully represented information. Choice "4" is incorrect. Feedback value is no longer used in the conceptual framework.

Harbor City's Appropriations Control Account at December 31, 2005 had a balance of $7,000,000. When the budgetary accounts were closed at year-end, this $7,000,000 Appropriations Control balance should have: A. Been debited. B. Been credited. C. Remained open. D. Appeared as a contra account

Choice "1" is correct. Debited. When closing the budgetary accounts at year-end, the "appropriations control account" should have been debited $7,000,000 to offset the $7,000,000 credit balance that existed before closing. Debit (Dr)Credit (Cr)Appropriations control 7,000,000 Estimated revenues 7,000,000

A business combination is accounted for properly as an acquisition. Direct costs of combination, other than registration and issuance costs of equity securities, should be: a. Deducted in determining the net income of the combined corporatio for the period in which the costs were incurred b. Included in the acquisition cost to be allocated to identifiable assets according to their fair values c. Deducted directly from the retained earnings of the combined corporation d. Capitalized as a deferred charge and amortized

Choice "1" is correct. Direct costs are expensed in the period incurred.

Which of the following must be present in order to require the disclosure of the estimated effect of a change in an estimate used in the preparation of financial statements? I. It is reasonably possible that the estimate will change in the near term II. The effect of a change in the estimate would be material III. The actual result is different from the estimate a. I and II b. II only c. II and III d. I only

Choice "1" is correct. Disclosure of the estimated effect of a change in an estimate used in the preparation of financial statements must be disclosed if it is reasonably possible that the estimate will change in the near term and if the change in the estimate would be material. It is not necessary that the actual result is different from the estimate (whether or not the actual result is different from the estimate would probably not be known by the time the financial statements are issued). Choices "4", "2", and "3" are incorrect based on the above explanation.

Balfour Charities, a not-for-profit organization, received a $200,000 cash donation from Emily Balfour under a charitable remainder annuity trust agreement designating the Balfour Charities as both trustee and remainder beneficiary. The trust agreement specifies that Balfour Charities must both invest the $200,000 and pay $10,000 per year to Roscoe Balfour, Emily's cousin, until his death. Any funds remaining after Roscoe's death will be retained by Balfour Charities and used in a manner consistent with their vision. The present value of the annuity payable to Roscoe is $65,000. As a result of this transaction, Balfour would recognize contributions of: a. $135,000 b. $0 c. $65,000 d. $200,000

Choice "1" is correct. Emily Balfour's contribution represents a split interest agreement that is valued at $135,000, the fair value of the contribution ($200,000) net of the present value of the estimated annuity payments ($65,000). Choice "2" is incorrect. Balfour charities should recognize contributions to the extent that the fair value of the contribution received exceeds the present value of estimated annuity payments. Choice "3" is incorrect. The amount of the estimated annuity payments is a liability, Annuity Obligations, and is not revenue. Choice "4" is incorrect. Emily Balfour's contribution represents a split interest agreement that must be reduced by the present value of the estimated annuity payments.

Which of the following will not have a separate earnings per share calculation and disclosure under U.S. GAAP? I. Extraordinary items of the period II. Discontinue operations III. Unrealized gains/losses on AFS securities a. I and III b. I and II c. All of the items will be shown with an earnings per share calculation and disclosure d. II and III

Choice "1" is correct. Extraordinary items are no longer recognized under U.S. GAAP. Only discontinued operations have separate earnings per share calculations and disclosures. Unrealized gains and losses on available-for-sale securities are part of other comprehensive income. Other comprehensive income items are direct charges to stockholders' equity and do not affect net income. They have no earnings per share calculations and disclosures. Choices "3", "2", and "4" are incorrect, per the above discussion.

Which of the following should be disclosed in the footnotes to the financial statements? a. Information about changes in stockholder's equity b. An analysis of the company's major competitors c. Management's estimate of sales for the upcoming year d. A projection of future market conditions

Choice "1" is correct. Footnote disclosures should include information on changes in stockholders' equity as well as any other information about significant asset and/or liability accounts. Choices "3", "2", and "4" are incorrect. Footnotes would not include management's estimate of sales for the upcoming year, an analysis of the company's major competitors, or a projection of future market conditions. Although such qualitative assessments may be included in the "management discussion and analysis" that precedes the financial statements and footnotes in the Form 10-K required to be filed by publicly held corporations with the Securities and Exchange Commission, they would not be footnote disclosures.

The Lester Corporation prepares its second-quarter interim report under U.S. GAAP. In preparing the reports, Lester will most likely: a. Prepare the interim report with less due diligence than it would use for the annual report in order to issue it faster b. Book expenses based on when the company pays its vendors c. Book revenue based on when cash is received, with a reconciliation to accrual numbers at year-end d. Have its auditors review and "sign off" on the report

Choice "1" is correct. For interim reports, timeliness is emphasized over reliability. Therefore, the company will make more of an effort to get the reports out faster even if it sacrifices some of the reliability of the data presented. Choice "2" is incorrect. Expenses should be booked based on the periods receiving the benefits from the expense. If an expense benefits more than one period, it should be allocated across those periods. This may or may not coincide with when a vendor is paid. Choice "3" is incorrect. Revenue should be booked based on when it is earned and realized/realizable. Choice "4" is incorrect. Auditors will not typically review and "sign off" on interim reports.

During the current year, Vann County's motor pool internal service fund sold two vehicles for $5,000. The vehicles had a cost of $6,000 and a carrying value of $4,000. How should Vann County's motor pool internal service fund report this transaction in its fund financial statements? a. Gain of $1,000 b. Special item of $1,000 c. Revenue of $5,000 d. Other financing source of $5,000

Choice "1" is correct. Gain of $1,000. The motor pool internal service fund is a proprietary fund that uses the economic resources measurement focus. Noncurrent assets are capitalized, depreciated and, upon disposal, are removed from the records with possible recognition of a gain or loss. The gain is appropriate based on the fact pattern and would be recorded/computed as follows: Debit (Dr)Credit (Cr) Cash 5,000 Accumulated depreciation 2,000 Equipment (Cars) 6,000 Gain 1,000 The facts stated that the cost was $6,000 and the carrying value was $4,000. This yields an accumulated depreciation of $2,000. (Cost Less Accumulated Depreciation = Carrying Value) Choice "2" is incorrect. The motor pool internal service fund is a proprietary fund that uses the economic resources measurement focus. Noncurrent assets are capitalized, depreciated and, upon disposal, are removed from the records with potential recognition of a gain or loss, While gains and losses may be recognized as an unusual or infrequent event qualifying for display as a special item, the amount must be material. A $1,000 gain is likely not material and thus not qualified for treatment as a special item. Choice "3" is incorrect. The motor pool internal service fund is a proprietary fund that uses the economic resources measurement focus. Noncurrent assets are capitalized, depreciated and, upon disposal, are removed from the records with potential recognition of a gain or loss, not revenue. Choice "4" is incorrect. The motor pool internal service fund is a proprietary fund that uses the economic resources measurement focus. Noncurrent assets are capitalized, depreciated and, upon disposal, are removed from the records with potential recognition of a gain or loss, not other financing sources (used for proceeds from sale of equipment in governmental funds).

An asset group is being evaluated for an impairment loss. The following financial information is available for the asset group: Carrying value $100,000,000 Sum of the undiscounted cash flows 95,000,000 Fair value 80,000,000 What amount of impairment loss, if any, should be recognized? a. $20,000,000 b. $0 c. $5,000,000 d. $15,000,000

Choice "1" is correct. If the carrying value of an asset group is greater than its fair value, an impairment loss may need to be recognized. The impairment loss is recognized, however, only if the undiscounted cash flows for the asset group are less than the carrying value. In that case, the amount of the impairment loss is the difference between the carrying value and fair value. The fair value of $80,000,000 as given is the appropriate value to use rather than the undiscounted cash flows because in order to be an accurate representation of fair value, the future cash flows would need to be discounted. If the undiscounted cash flows for the asset group are NOT less than the carrying value, no impairment loss is recognized (even if the carrying value is greater than the fair value). Choice "2" is incorrect. An impairment loss should be recognized as: 1) the undiscounted cash flows of the asset group are less than the carrying value, and 2) the fair value of the asset group is less than the carrying value. Choice "3" is incorrect. Though discounted future cash flows may be used in determining the fair value for purposes of recording an impairment, these cash flows are undiscounted. They would need to be discounted at an appropriate interest rate in order to determine the fair value. Choice "4" is incorrect. The difference between the undiscounted cash flows and fair value plays no role in determining whether an impairment will be recorded nor does it play a role in the amount of impairment recorded, if an impairment is recorded.

Which of the following is not an example of a variable interest in an entity? a. Accounts payable b. An explicit guarantee of the entity's debt c. A forward contract to sell assets owned by the entity d. An option to acquire a leased asset at fair value at the end of the lease term

Choice "1" is correct. Most liabilities, excluding short-term trade payables (accounts payable), represent variable interests. Choice "2" is incorrect. An explicit guarantee of the entity's debt is a variable interest. Choice "3" is incorrect. A forward contract to sell assets owned by the entity is a variable interest. Choice "4" is incorrect. An option to acquire a leased asset at fair value at the end of the lease term represents a variable interest.

Which of the following funds of a local government would report transfers to other funds as an other financing use? a. General b. Pension Trust c. Internal Service d. Enterprise

Choice "1" is correct. Only the governmental funds (general, special revenue, debt service, capital projects, and permanent) report transfers to other funds as an "other financing use." Choice "2" is incorrect. Pension trust funds are a type of fiduciary fund which will typically not report transfers at all. Transactions between the government and its fiduciary funds are handled as if the fiduciary activity was accounted by a trustee or other entity outside of the government. Choice "3" is incorrect. Internal service funds are a type of proprietary fund which will report transfers to other funds as "nonoperating expenses." Choice "4" is incorrect. Enterprise funds are a type of proprietary fund which will report transfers to other funds as "nonoperating expenses."

West, Inc., acquired 60% of East Co.'s outstanding common stock. West paid $800,000 to acquire the stock. West plans to relocate East's company headquarters, which is expected to cost between $100,000 and $300,000. The present value of the probability-adjusted relocation cost is $240,000. What is West's acquisition cost? a. $800,000 b. $900,000 c. $1,040,000 d. $1,100,000

Choice "1" is correct. The acquisition cost of the stock does not include any measure of the relocation costs associated with East's company headquarters. (Such costs are accounted for separately from the acquisition according to the requirements for exit or disposal costs in FASB ASC 420.) The investment would be valued at the fair value of the consideration given which is $800,000. Choice "1" is incorrect. The acquisition cost of the investment is the fair value of the consideration given or the fair value of the consideration received, whichever is the more clearly evident. In this situation, the fair value of the consideration given is $800,000. The present value of the probability-adjusted relocation cost is $240,000 and should not be included as part of the acquisition cost. Choice "3" is incorrect. The $300,000 higher range estimate of the cost to relocate the East company headquarters is not to be considered as part of the acquisition cost. Choice "4" is incorrect. The $100,000 lower range estimate of the cost to relocate the East company headquarters is not to be considered as part of the acquisition cost.

On January 2 of the current year, LTTI Co. entered into a three-year, noncancelable cotract to buy up to 1 million units of a product each year at $0.10 per unit with a minimum annual guarantee purchase of 200,000 units. At year-end, LTTI had purchased only 80,000 units and decided to cancel sales of the product. What amount should LTTI report as a loss related to the purchase commitment as of December 31 of the current year? a. $52,000 b. $12,000 c. $8,000 d. $0

Choice "1" is correct. The contract overall has a minimum total guarantee of 600,000 units over the three-year contract, and only 80,000 units will be purchased. At $0.10 per unit, the company is responsible for 520,000 units at $0.10 per unit. The loss as a result will be $52,000 (520,000 × $0.10). Choice "2" is incorrect. This only accounts for the current year, and fails to account for the remaining two years when no purchases will be made. Choice "3" is incorrect. This represents the amount actually purchased. Choice "4" is incorrect. Because the contract is noncancelable and there is a minimum guarantee, there will be a loss resulting from not honoring that guarantee.

Which of the following is not a cost associated with exit and disposal activities? a. Costs associated with the retirement of a fixed asset b. Costs to relocate employees c. Costs to terminate contract that is not a direct finance lease d. Benefits related to involuntary employee termination

Choice "1" is correct. The cost of retiring a fixed asset is not considered an exit or disposal cost. Choice "2" is incorrect. Costs to relocate employees are costs associated with exit and disposal activities. Choice "3" is incorrect. Exit and disposal activities include costs to terminate a contract that is not a direct finance lease. Direct finance lease termination costs are accounted for separately from exit and disposal activities. Choice "4" is incorrect. Exit and disposal activities include benefits related to involuntary employee termination.

Metro General is a municipally owned and operated hospital and a component unit of Metro City. In Year 1, the hospital received $7,000 in unrestricted gifts and $4,000 in unrestricted bequests. The hospital has $800,000 in long-term debt and $1,200,000 in fixed assets. Before the hospital's financial statements were combined with those of the city, the city's statements included data on one special revenue fund and one enterprise fund. The city's statements showed $100,000 in enterprise fund long-term debt, $500,000 in enterprise fund fixed assets. $1,000,000 in general long-term debt, and $6,000,000 in general fixed assets. The hospital's long-term debt should be reported in the city's financial statements as: a. Long term debt in the component unit column of the government-wide long-term debt in the financial statements b. An $800,000 contra amount against fixed assets c. Part of $1,800,000 in long-term debt displayed in the governmental activities column of the government-wide statement of net position d. A separate "discrete presentation" of $800,000 in the hospital column

Choice "1" is correct. The hospital's long-term debt should be reported in the city's financial statements as part of $800,000 component unit long-term debt in the government-wide financial statements. Choice "2" is incorrect. Debt is not offset against assets. Choice "3" is incorrect. The hospital is an enterprise fund component, not part of governmental activities. Choice "4" is incorrect. There is no hospital column.

A defined benefit pension plan had the following activity during the fiscal year: Dividends and interest received $92,000 Contributions received from employers and employees $340,000 Administrative expenses $45,400 Investments Purchased $155,000 Increase in fair value of investments at year-end $36,750 What amount should be reported as the total additions in the pension plan's statement of changes in ent assets available for benefits? a. $468,750 b. $423,350 c. $432,000 d. $313,750

Choice "1" is correct. The total additions of $468,750 include the $92,000 in dividends and interest received, the $340,000 in contributions received from employers and employees, and the $36,750 increase in investment fair value. The administrative expenses of $45,400 and the investments purchased of $155,000 represent subtractions from net assets available. Choice "2" is incorrect. This answer choice incorrectly subtracts the administrative expenses of $45,400 from the total additions. Choice "3" is incorrect. This answer choice fails to account for the increase in investment fair value. Choice "4" is incorrect. This answer choice incorrectly subtracts investments purchased from the total additions.

On January 17, Year 2, an explosion occurred at a Sims Co. plant causing extensive property damage to area buildings. Although no claims had yet been asserted against Sims by March 10, Year 2, Sim's management and counsel concluded that it is likely that claims that will be asserted and that it is reasonably possible Sims will be responsible for damages. Sim's management believed that $1,250,000 would be a reasonable estimate of its liability. Sim's $5,000,000 comprehensive public liability policy has a $250,000 deductible clause. In Sims' December 31, Year 1 Financial statements, which were issued on March 25, Year 2, how should this item be reported? a. As a footnote disclosure indicating the possible loss of $250,000 b. As an accrued liability of $250,000 c. As a footnote disclosure indicating the possible loss of $1,250,000 d. No footnote disclosure or accrual is necessary

Choice "1" is correct. This is a nonrecognized subsequent event as it occurred after the December 31, Year 1 balance sheet date. Because there is a reasonably possible contingent loss of $250,000, footnote disclosure is required. Rule: Only footnote disclosure is required for a "reasonably possible" loss. The nature of the contingency should be disclosed as well as the nature of the possible loss or range of loss. Since insurance appears to be available for the entire loss except the deductible portion, only the possible loss of $250,000 needs to be included in the footnote. Note: Even though the explosion occurred (January 17, Year 2) after the date of the FS (December 31, Year 1) disclosure is still required since the statements had not yet been issued.

Milton Co. pledged some of its accounts receivable to Good Neighbor Financing Corporation in return for a loan. Which of the following statements is correct? a. Milton will retain control of the receivables b. Good Neighbor Financing assuming the responsibility of collecting the receivables c. Good Neighbor Financing cannot take title of the receivables if Milton does not repay the loan. Title can only be taken if the receivables are factored d. Good Neighbor Financing will take title of the receivables, and will return title to Milton after the loan is paid

Choice "1" is correct. When a company pledges (assigns) receivables in return for a loan, the assigning company (Milton in this example) will retain title to the receivables and will use the proceeds collected from the receivables to repay the loan. Choice "2" is incorrect. Milton still assumes the responsibility of collecting the receivables. Choice "3" is incorrect. Factoring is not the only means by which title can be passed. Choice "4" is incorrect. Milton will retain title to the receivables.

The December 31, Year 1, financial statements of Frank Corp. were first available to be issued and were, in fact, actually issued on February 23, Year 2. The financial statements were then reissued on March 18, Year 2, to reflect the recognition of a subsequent event that occurred on January 9, Year 2. Which of the following events would require recognition by Frank Corp. through a second reissuance of the December 31, Year 1, financial statements? a. None of the above b. The destruction of its only manufacturing facility by fire on March 17, Year 2 c. The settlement of a lawsuit on March 7, Year 2, for substantially less than the amount of liability recorded for it on the December 31, Year 1, balance sheet as originally ossued d. The bankruptcy on February 25, Year 2, of a major customer that owed a significant amount to Frank Corp

Choice "1" is correct. When an entity reissues financial statements, the entity should not recognize events (i.e., adjust for events) that occurred between the date the original financial statements were issued or available to be issued (February 23, Year 2) and the date the financial statements were reissued (March 18, Year 2). Thus, none of the events described in answer choices "4", "3", or "2" would be recognized in a second reissuance of financial statements.

A lease is classified as a finance lease because it contains a purchase option that the lessee is reasonably certain to exercise. Over what period of time should the lessee amortize the leased property? a. The economic life of the asset b. The economic life of the asset, not to exceed 40 years c. The lease term or the economic life of the asset, whichever is shorter d. The term of the lesae

Choice "1" is correct. With a finance lease, the lessee should amortize the leased property over the economic life of the asset when there is a written purchase option or when the lessee takes ownership of the asset at the end of the lease term. Choice "2" is incorrect. There is not a provision for a maximum of 40 years. Choice "3" is incorrect. When the determination is made that a lease is a finance lease, the leased asset is to be amortized over its economic life or the asset life based on the capitalization criterion met. Choice "4" is incorrect. The term of the lease is not the appropriate period over which to amortize the leased asset in this case. It is appropriate when the 75 percent or 90 percent criteria are met.

Burgess Co. purchase 35% of Egg Co's outstanding common stock on December 31 for $300,000. On that date, Egg's stockholders' equity was $600,000, and the fair value of its identifiable assets was $700,000. On December 31, what amount of goodwill should Burgess attribute to this acquisition? a. $90,000 b. $55,000 c. $0 d. $35,000

Choice "2" is correct. The allocation of the investment purchase price is calculated as follows: Investment 300,000 Less: NBV ($600,000 × 35%) 210,000 Total excess 90,000 Allocation to identifiable net assets FMV 700,000 NBV (600,000) 100,000 × 35% 35,000 Excess to goodwill ($90,000 − $35,000) 55,000

Rabb Co. records its purchases at gross amounts but wishes to change to recording purchases net of purchase discounts. Discounts available on purchases recorded from October 1, Year 3, to September 30, Year 4, totaled $2,000. Of this amount, $200 is still available in the accounts payable balance. The balances in Rabb's accounts as of and for the year ended September 30, Year 4, before conversion are Purchases $100,000 Purchase discounts taken 800 Accounts payable 30,000 What is Rabb's accounts payable balance as of September 30, Year 4, after the conversion? a. $28,800 b. $29,800 c. $28,200 d. $29,200

Choice "2" is correct. $29,800. Accounts payable at "gross" amounts before adjustment for purchase discounts 30,000 Purchase discounts in AP balance available, but not yet taken (200) Accounts payable net of purchase discounts 29,800

The following stock dividends were declared and distributed by Sol Corp.: Perc of Common shares outstanding at Dec Date 10 28 FV 15k 40k PV 10k 30,800 What aggregate amount should be debited to retained earnings for these stock dividends? a. $40,800 b. $45,800 c. $50,000 d. $55,000

Choice "2" is correct. $45,800. For a small stock dividend (less than 20-25% of the shares outstanding previously) the fair value of the shares is capitalized from retained earnings. For a large stock dividend (more than 20-25% of the shares outstanding previously), only the amount legally required to be capitalized is transferred from retained earnings (typically the par value of the stock). Choice "1" is incorrect. Fair value not par value should be used on a small stock dividend (less than 20% of the shares outstanding prior to the dividend). Choice "3" is incorrect. Fair value should be used for a small dividend, and par value should be used for a large stock dividend. Choice "4" is incorrect. Par value should be used for a large stock dividend.

Able Co. provides an incentive compensation plan under which its president receives a bonus equal to 10% of the corporation's income before income tax but after deduction of the bonus. If the tax rate is 40% and net income after bonus and income tax was $360,000, what was the amount of the bonus? A $36,000 B $60,000 C $66,000 D $90,000

Choice "2" is correct. $60,000 bonus. Step 1 - Determine pre-tax income After tax income - $360,000 (after bonus & income tax) Pre-tax income - $360,000 ÷ 60% Pre-tax income - $600,000 after deduction for bonus Step 2 - Determine bonus Bonus = Pre-tax income after deduction for bonus x 10% Bonus = $600,000 x 10% Bonus = $60,000

At December 31, Year 2, Off-Line Co. changed its method of accounting for demo costs from writing off the costs over two years to expensing the costs immediately. Off-Line made the change in recognition of an increasing number of demos placed with customers that did not result in sales. Off-Line had deferred costs of $500,000 at December 31, Year 1, $300,000 of which were to be written off in Year 2 and the remainder in Year 3. Off-Line's income tax rate is 30%. In its Year 3 financial statements, what amount should Off-Line report as cumulative effect of change in accounting principle? a. $200,000 b. $0 c. $350,000 d. $500,000

Choice "2" is correct. A change in method of accounting for demo costs is a change in accounting principle inseparable from a change in estimate. When a change in accounting principle is considered inseparable from a change in estimate, the change is handled as a change in estimate - prospectively. No cumulative effect adjustment is made. Choices "1", "3", and "4" are incorrect since no cumulative effect adjustment is made.

Applebee Farms, a not-for-profit organization, has received the following support during its fiscal year ended June 30, Year 1: -A contribution of $300,000 stipulating that the gift is to be used for research -A pledge from a long-time benefactor for $250,000 intended for the support of the mission of Applebee Farms. The pledge was collected August 13, Year 1. -A contribution of $50,000 along with a pledge for an additional $150,000 to be used for programming consistent with the organization's mission only upon completion of a new research facility. The research facility was 80 percent at fiscal year end In its June 30, Year 1 financial statements, Applebee Farms would report: Rev without Donor Restrictions Rev w/donor restrictions a. $40,000 $700,000 b. $0 $550,000 c. $250,000 $350,000 d. $300,000 $450,000

Choice "2" is correct. Applebee Farms would recognize $550,000 in revenue with donor restrictions comprised of the $300,000 restricted by the donor for research and the $250,000 pledge receivable with an implied time restriction. Choice "1" is incorrect. Although the revenue with donor restrictions is $550,000, the proposed solution also includes a conditional pledge for $150,000 that would not be recorded until all conditions are fully satisfied. The $50,000 conditional contribution would be treated as a refundable advance, not revenue. The answer implies recognition of the revenue without donor restriction in proportion to completion of the condition ($50,000 × 80% = $40,000), which is incorrect. Choice "3" is incorrect. The $250,000 pledge has an implied time restriction and would not be classified as without donor restriction. The addition of the $50,000 refundable advance, which is a liability, to earnings with donor restrictions is incorrect. Choice "4" is incorrect. The combination of the refundable advance and the pledge revenue is incorrectly classified as revenue without donor restrictions and the combination of the $300,000 revenue with donor restrictions and the conditional pledge as revenue with donor restrictions is also incorrect.

The following are held by Smite Co: Cash in checking account $20,000 Cash in bond sinking fund account $30,000 Post-dated check from customer dated one month from balance sheet date $250 Petty Cash $200 Commercial paper (matures in two months) $7,000 Certified of deposit (matures in six months) $5,000 What amount should be reported as cash and cash equivalents on Smite's balance sheet? a. $57,200 b. $27,200 c. $27,450 d. $32,200

Choice "2" is correct. Cash is defined as actual unrestricted cash and cash equivalents are defined as short-term, liquid investments that are so near maturity (original maturity date was within three months of the purchase date) that the risk of changes in the value because of interest rate changes is insignificant. Cash in checking account 20,000 Petty cash 200 Commercial paper (2-month maturity) 7,000 Cash and cash equivalents 27,200 Choice "1" is incorrect. Cash and cash equivalents would not include the $30,000 in the bond sinking fund. Cash in a bond sinking fund is restricted cash. Choice "3" is incorrect. The post-dated check should not be included in cash and cash equivalents because it is dated after the balance sheet date. Choice "4" is incorrect. The certificate of deposit with six months to maturity is not included in cash and cash equivalents because it does not meet the definition of a cash equivalent (maturity date within 3 months of the purchase date).

For a finance lease, the amount recorded initially by the lessee as a liability should normally: a. Exceed the present value of the minimum lease payments at the beginning of the lease b. Equal the present value of the minimum lease payments at the beginning of the lease c. Equal the total of the minimum lease payments d. Exceed the total of the minimum lease payments

Choice "2" is correct. Equal the present value of the minimum lease payments at the beginning of the lease. Rule: A lessee should record a finance lease as if he owned the asset at the present value of the minimum lease payments at the beginning of the lease (excluding any "executory costs"). Choice "1" is incorrect. Liability cannot exceed the PV of the minimum lease payments. Choice "3" is incorrect. Rule is: "PV" (not "total") of the minimum lease payments. Choice "4" is incorrect. Liability cannot exceed the PV (not "total") of the minimum lease payments.

Which of the following should be considered part of one of the three primary user groups of the external financial reports of a state government? a. Internal managers in the executive branch of the state government b. Advocate groups within the state c. Citizens of a neighboring state d. Preparers of state government financial reports

Choice "2" is correct. External financial reports are used by three primary user groups including citizens, legislative/oversight groups, and investors/creditors. Advocate groups within the state are logically associated with citizens, citizen groups, etc. Choice "1" is incorrect. Internal managers in the executive branch of the state government would not be included in the three primary user groups commonly identified with use of external financial reports. Primary user groups include citizens of the jurisdiction, individuals and groups charged with oversight (e.g., executives, legislative bodies, etc.), and investors/creditors. Internal managers within the executive branch would likely not have oversight authority. Choice "3" is incorrect. Citizens of a neighboring state would not be included in the three primary user groups commonly identified with use of external financial reports. Primary user groups include citizens of the jurisdiction, individuals and groups charged with oversight (e.g., executives, legislative bodies, etc.), and investors/creditors. Choice "4" is incorrect. Preparers of state government financial reports would not be included in the three primary user groups commonly identified with use of external financial reports. Primary user groups include citizens of the jurisdiction, individuals and groups charged with oversight (e.g., executives, legislative bodies, etc.), and investors/creditori

Famous, a nongovernmental not-for-profit art museum, has elected not to capitalize its donated permanent collections. In Year 1, a bronze statue was stolen. The statue was not recovered and insurance proceeds of $35,000 were paid to Famous in Year 2. This transaction would be reported in: I. The statement of activities as revenues without donor restrictions II. The statement of cash flows as cash flows from investing activities a. Both I and II b. II only c. I only d. Neither I nor II

Choice "2" is correct. Investing activities in the statement of cash flows should include proceeds from the sale of long lived assets or insurance proceeds associated with the loss of long lived assets. Entities that do not capitalize their permanent collections display insurance proceeds from lost, stolen or damaged items on the statement of activities in an appropriate change in net asset classification separate from revenues, expenses, gains, and losses. Contributed collection items should not be recognized as revenues or gains if collections are not capitalized. Cash flows from purchases, sales, and insurance recoveries of unrecognized, noncapitalized collection items should be reported as investing activities in a statement of cash flows. A not-for-profit organization that does not recognize and capitalize its collections should report the following on the face of its statement of activities separately from revenues, expenses, gains, and losses: Costs of collection items purchased as a decrease in the appropriate class of net assets Proceeds from sale of collection items as an increase in the appropriate class of net assets Proceeds from insurance recoveries of lost or destroyed collection items as an increase in the appropriate class of net assets

Pharm, a nongovernmental not-for-profit entity, is preparing its year-end financial statements. Which of the following statements is required? a. Statement of changes in financial position b. Statement of cash flows c. Statement of revenue, expenses and changes in fund balance d. Statement of changes in fund balance

Choice "2" is correct. Not-for-profit corporations are required to produce the following financial statements: Statement of financial position Statement of activities Statement of cash flows Not-for-profit corporations are also required to disclose, display, or separately report the relationship between functional classifications and natural classifications of expenses. Choice "1" is incorrect. The statement of changes in financial position is no longer used by not-for-profit organizations. Choice "3" is incorrect. Not-for-profits do not account for fund balances in financial statements issued for external use and therefore do not produce a statement of revenues, expenses, and changes in fund balance. Choice "4" is incorrect. Not-for-profits do not account for fund balances in financial statements issued for external use and therefore do not produce a statement of changes in fund balance.

The primary purpose of a not-for-profit organization's statement of activities is to provide relevant information for its: a. State regulatory body b. Resource providers c. Beneficiaries d. Managers

Choice "2" is correct. Regardless of whether an entity is a for-profit or a not-for-profit, the objective of financial reporting (and the associated financial statements) is to provide information about the entity that is useful to primary users in making decisions about resources to provide to the entity. Choice "1" is incorrect. Although the state regulatory body does review the financial statements for not-for-profits, that is not the primary reason why the statements are produced. Choice "3" is incorrect. Although beneficiaries may be interested in a not-for-profit organization's financial statements, the primary purpose of the statement of activities is to provide information to resource providers. Choice "4" is incorrect. Although managers will see value in reviewing an entity's financial statements, they are not considered primary users of these statements.

Shared revenues received by an enterprise fund of a local government for operating purposes should be recorded in fund financial statements as: a. Operating revenues b. Non-operating revenues c. Other financing sources d. Interfund transfers

Choice "2" is correct. Shared revenues are revenues levied by one government but shared on a predetermined basis with another government. Shared revenues received by a proprietary fund should be recorded as non-operating revenue if received for operations or if received for either operations or capital expenditures at the discretion of the recipient governmental unit. Choice "1" is incorrect. Shared revenues are revenues levied by one government but shared on a predetermined basis with another government. Shared revenues received by proprietary fund types are not classified as operating revenues. Choice "3" is incorrect. "Other financing sources" is not a proper classification used in the financial statements of enterprise funds. Choice "4" is incorrect. Shared revenues are not classified as interfund transfers.

Justification for the method of determining periodic deferred tax expense is based on the concept of: a. Objectivity in the calculation of periodic expense b. Recognition of assets and liabilities c. Consistency of tax expense measurements with actual tax planning strategies d. Matching of periodic expense to periodic revenue

Choice "2" is correct. The justification for the method of determining periodic deferred tax expense is based on recognition of assets and liabilities.

On a statement of cash flows, cash flows from investing activities would be decreased by which of the following? a. Issuance of common stock b. Purchase of long-term investments c. Issuance of bonds payable d. Payment of dividends

Choice "2" is correct. The purchase of long-term investments would be reported in the investing activities section as an outflow of cash. The other items listed would be reported in the financing activities section. Choices "1", "3", and "4" are incorrect, per the above explanation.

How would the purchase of treasury stock affect each of the following? Total Stockholders equity EPS a. NE NE b. DE INC c. INC DE d. DE NE

Choice "2" is correct. The purchase of treasury stock would decrease stockholders' equity and the number of outstanding shares. As such, the amount of earnings per share increases. Choice "1" is incorrect. The purchase of treasury stock would have decreased total stockholders' equity and increased the number of outstanding shares. Choice "3" is incorrect. The purchase of treasury stock would not have increased total stockholders' equity and would not have decreased earnings per share. This answer is the opposite of the correct answer. Choice "4" is incorrect. The purchase of treasury stock would have decreased total stockholders' equity. However, earnings per share would have increased since the number of outstanding shares would have decreased.

Which of the following is not a characteristic that is used to determine the primary beneficiary of a variable interest entity (VIE) under U.S. GAAP? a. The power to direct the activities of the VIE b. Greater than 50% ownership of the VIE c. The obligation to absorb expected VIE losses d. The right to receive the expected VIE residual returns

Choice "2" is correct. Under the VIE model, the primary beneficiary is not required to have greater than 50% ownership of the VIE. The primary beneficiary is the entity that has the power to direct the activities of a variable interest entity that most significantly impact the entity's economic performance and absorbs the expected VIE losses and/or receives the expected VIE residual returns.

On December 1, Nilo Corp. declared a property dividend of marketable securities to be distributed on December 31 to stockholders on record on December 15. On December 1, the marketable securities had a carrying amount of $60,000 and a fair value of $78,000. What is the effect of this property dividend on Nilo's retained earnings, after all nominal accounts are closed? a. $18,000 increase b. $0 c. $60,000 decrease d. $78,000 decrease

Choice "3" is correct. $60,000 decrease. Two factors will affect retained earnings as a result of this transaction. Nilo will recognize a gain on disposition of marketable securities as well as a dividend, as follows: Gain on marketable securities 18,000 Property dividend (78,000) Impact on RE (60,000) Note that the question asks for the effect on retained earnings after all nominal accounts (income and expense accounts) are closed. Choice "1" is incorrect. The $18,000 increase in retained earnings from the gain on the marketable securities will be offset by the $78,000 decrease in retained earnings from the dividend. Choice "2" is incorrect. All dividends decrease retained earnings. Choice "4" is incorrect. The $78,000 decrease in retained earnings from the dividend will be offset by the $18,000 increase in retained earnings from the gain on marketable securities.

On September 1, Year 1, Phillips Inc. issued common stock in exchange for 20 percent of Sago's Inc.'s outstanding common stock. On July 1, Year 3, Phillips issued common stock for an additional 75 percent of Sago's common stock. Sago continues in existence as Phillip's subsidiary. How much of Sago's Year 3 net income should be reported as accruing to Phillips? a. 20 percent of Sago's net income to June 30 and all of Sago's net income from July 1 to December 31 b. All of Sago's net income c. 20 percent of Sago's net income to June 30 and 95 percent of Sago's net income from July 1 to December 31 d. 95 percent of Sago's net income

Choice "3" is correct. 20 percent of Sago's net income to June 30 and 95 percent of Sago's net income from July 1 to December 31. Rule: In an acquisition, the net income of a newly acquired subsidiary will only be included in consolidated net income from the date of acquisition. Therefore, only 20 percent of Sago net income is included in consolidated earnings until June 30 and 95 percent thereafter.

A corporation declared a 10 percent stock dividend on 15,000 shares outstanding of $5 par common stock when the fair value was $10 per share. Which change in the corporation's stockholders' equity accounts is correct? a. Common stock is increased by $15,000 b. Common stock is decreased by $7,000 c. Retained earnings is decreased by $15,000 d. Additional paid-in-capital is increased by $15,000

Choice "3" is correct. A 10 percent stock dividend qualifies as a small stock dividend because the issuance is not expected to affect the market price of the stock. The stock dividend totals 1,500 shares (15,000 shares × 10% dividend). The fair value of the stock on the date of declaration is transferred from retained earnings to common stock and additional paid-in-capital as follows: DRRetained earnings15,000 (1,500 shares × $10/share fair value)CRCommon stock 7,500 CRAdditional paid-in-capital 7,500 Choice "1" is incorrect. The increase to total capital stock is $15,000, but the increase to common stock is only $7,500 because it is based upon the $5 par value of the stock (1,500 shares × $5 par value). Choice "2" is incorrect. Common stock increases because of a stock dividend. Retained earnings decreases. Choice "4" is incorrect. The increase to total capital stock is $15,000, but the increase to additional paid-in-capital totals $7,500. The credit to common stock is $7,500 (1,500 shares × $5 par value) and the remaining credit is to additional paid-in-capital for the excess of the fair value over the par value of the stock issued.

Bale Co. incurred $100,000 of acquisition costs related to the purchase of the net assets of Dixon Co. The $100,000 should be: a. Capitalized as an other asset and amortized over five years b. Capitalized as part of goodwill and tested annually for impairment c. Expensed as incurred in the current period d. Allocated on a pro rata basis to the nonmonetary assets required

Choice "3" is correct. Acquisition costs associated with a business transaction must be expensed as incurred in the current period. Choice "1" is incorrect. Acquisition costs associated with a business transaction are not capitalized as an other asset and amortized over five years but expensed as incurred in the current period. Choice "2" is incorrect. Acquisition costs associated with a business transaction are not capitalized as part of goodwill and tested annually for impairment but expensed as incurred in the current period. Choice "4" is incorrect. Acquisition costs associated with a business transaction are not allocated on a pro rata basis to the nonmonetary assets acquired but expensed as incurred in the current period.

A company performing its long-lived asset impairment testing is reviewing the fair value of equipment. Each of the following valuation techniques may be appropriate for measuring the fair value of the equipment, except the: a. Market approach b. Income approach c. Net realizable value approach d. Cost approach

Choice "3" is correct. Although net realizable value is used to measure many items on the balance sheet, the fair value of equipment cannot be measured using this approach. Choice "1" is incorrect. The market approach is appropriate for measuring fair value. Choice "2" is incorrect. The income approach may be used to measure fair value. Choice "4" is incorrect. The cost approach is a valid technique for measuring fair value.

A company has the following accrual-basis balances at the end of its first year of operation: Unearned consulting fees $2,000 Consulting fees receivable $3,500 Consulting Fee revenue $25,000 The company's cash-basis consulting revenue is what amount? a. $26,500 b. $30,500 c. $23,500 d. $19,500

Choice "3" is correct. Cash-basis consulting revenue is $23,500. This amount includes the $2,000 in cash received recorded as unearned consulting fees plus $21,500 in cash received from consulting services provided during the period. The cash received from consulting services provided during the period can be calculated by analyzing the changes in the consulting fees receivable account during the period: Beginning consulting fees receivable $0 Plus: Accrual basis consulting fee revenue 25,000 Less: Cash collections ??? ← $21,500 Ending consulting fees receivable $3,500 Choices "4", "1", and "2" are incorrect, per the above.

A public entity sells steel for use in construction. One of its customers accounts for 43% of sales, and another customer accounts for 40% of sales. What should the entity disclose in its annual financial statements about these two customers? a. The payment terms of accounts receivable due from each of the two customers b. The names of the two customers c. The amount of the entity's revenue from each of the two customers d. The financial condition of the two customers

Choice "3" is correct. Concentrations in the volume of business transacted with a particular customer should be disclosed in the notes to the financial statements because these two customers individually contribute to significant sales. These concentrations increase the risk of loss, and information stating that fact should be disclosed to the financial statement user. Choice "1" is incorrect. Information about the credit terms does not require disclosure as it does not affect risk. Choice "2" is incorrect. The names of the customers involved in the concentration of business transactions do not require disclosure. Choice "4" is incorrect. The financial condition of the customers involved in the concentration of business transactions does not require disclosure.

When the fair value of an investment in debt securities exceeds its amortized cost, how should each of the following debt securities be reported at the end of the year? Debt securities classified as HTM AFS a. FV AC b. FV FV c. AC FV d. AC AC

Choice "3" is correct. Debt securities (bonds) classified as held-to-maturity are reported at amortized cost (that is, cost adjusted for amortization of premium or discount; approaches face value). Debt securities classified as available-for-sale are reported at fair value. Choice "1" is incorrect. Fair value is not the appropriate treatment for debt securities classified as held-to-maturity. Nor is amortized cost the appropriate treatment for debt securities classified as available-for-sale. Choice "2" is incorrect. Fair value is not the appropriate treatment for debt securities classified as held-to-maturity. Choice "4" is incorrect. While amortized cost is the appropriate treatment for debt securities classified as held-to-maturity, this is not the correct treatment for securities classified as available-for-sale.

Which of the following is correct concerning financial statement disclosure of accounting policies? a. The format and location of accounting policy disclosures are fixed by GAAP b. Disclosures should be limited to principles and methods peculiar to the industry in which the company operates c. Disclosures of accounting policies is an integral part of the financial statements d. Disclosures should duplicate details disclosed elsewhere in the financial statements

Choice "3" is correct. Disclosure of accounting policies (and all other disclosure also) is an integral part of the financial statements. Choice "1" is incorrect. For disclosure of accounting policies, the format and location of accounting policies are not fixed by GAAP. Accounting policy disclosures are normally Note 1, but that is a (reasonable and very general) practice and not a "rule." It does make sense to disclose the "why" before the "what." Choice "2" is incorrect. For disclosure of accounting policies, disclosure should not be limited to principles and methods peculiar to the industry in which the company operates. All material accounting policies should be disclosed. Choice "4" is incorrect. Disclosure of accounting policies should not duplicate details disclosed elsewhere in the financial statements.

Dugger County appropriated $45,000 in its General Fund for miscellaneous supplies for its fiscal year ended September 30, Year 2. The County found that it had paid $15,000 for miscellaneous supplies in November Year 1 and issued a $30,000 purchase order to a sole source vendor for miscellaneous supplies in December Year 1. By August Year 2, the County had received $20,000 related to the order but did not pay the vendor until October pending tax receipts. Appropriations do not lapse. What entry would Dugger County make at year-end to close its books on September 30, Year 2? a. Debit appropriations Control for $10,000 b. Debit appropriations Control for $35,000 c. Debit Appropriations control for $45,000 d. Credit Appropriations Control for $35,000

Choice "3" is correct. Dugger County would use BAE-BAE accounting in its governmental funds by recording budget, activity, and encumbrances separately and then reversing budget, activity, and encumbrances separately. The county would reverse the full amount of the budgeted amount of the appropriations, originally recorded as a credit, as a debit of $45,000. Choices "2", "1", and "4" are incorrect. Different permutations of netted transactions would not be appropriate. Budget, activity, and encumbrance transactions are handled separately.

Sayon Co. issues 200,000 shares of $5 par value common stock to acquire Trask Co. in an acquisitionbusiness combination. The market value of Sayon's common stock is $12. Legal and consulting fees incurred in relationship to the purchase are $110,000. Registration and issuance costs for the common stock are $35,000. What should be recorded in Sayon's additional paid-in capital account for this business combination? a. $1,545,000 b. $1,400,000 c. $1,365,000 d. $1,255,000

Choice "3" is correct. In an acquisition method business combination, registration and issuance costs are recorded as a direct reduction to the value of the stock issued by reducing APIC and direct out-of-pocket costs such as legal and consulting fees are expensed. This business combination will be recorded as follows: Debit (Dr)Credit (Cr)Legal & consulting expense 110,000 Investment in Trask 2,400,000* Cash 145,000** Common stock 1,000,000*** APIC 1,365,000**** * $2,400,000 = 200,000 shares × $12/share ** $145,000 = $110,000 + $35,000 *** $1,000,000 = 200,000 shares × $5 par **** $1,365,000 = [200,000 shares × ($12 - $5)] - $35,000

A company incurred the following costs to complete a business combination in the current year: Issuing debt securities $30,000 Registering debt securities $25,000 Legal Fees $10,000 Due diligence costs $1,000 What amount should be reported as current-year expenses, not subject to amortization? a. $66,000 b. $36,000 c. $11,000 d. $1,000

Choice "3" is correct. Legal fees and due diligence costs are expensed in the period incurred. Debt securities create liabilities, and debt security registration costs are capitalized and amortized. Choice "1" is incorrect. The costs associated with the issuance and registration of debt securities are not expensed. Choice "2" is incorrect. The debt security registration costs are not expensed; rather, they are capitalized and amortized. Choice "4" is incorrect. This choice incorrectly excludes the legal fees from the expense total.

Which of the following would be reported as an investing activity in a company's statement of cash flows? a. Collection of an overdue account receivable from a customer b. Collection of a tax refund from the government c. Collection of a note receivable from a related party d. Collection of proceeds from a note payable

Choice "3" is correct. Loans to other entities and the consequent collection of the loans are reflected in the investing activity section of the cash flow statement. Choice "1" is incorrect. Accounts receivable collections fall under operating activities in the statement of cash flows. Choice "2" is incorrect. Collecting a tax refund will fall under operating activities in the statement of cash flows. Choice "4" is incorrect. Notes payable fall under financing activities in the statement of cash flows.

The Bygone Historical Society, a not-for-profit organization, received a donation of fifteen Daguerre-Type (metal) photos of Bygone's riverfront from a family estate. The photos were not suitable for display but were accepted by the historical society for their potential value to researchers and historians. The photos have no alternative use. The Bygone Historical Society adopted a policy of capitalizing its contributed works of art and historical treasures. Under these circumstances, Bygone Historical Society would: a. Recognize the photos as assets (historical treasures) and contribution revenue in the amount of their fair value b. Disclose the receipt of historical treasures not eligible for display c. Not recognize the contributed photos as assets an contribution revenue d. Recognize the photos as assets (historical treasures) and contribution revenue in the amount of the value of the metal

Choice "3" is correct. Regardless of Bygone Historical Society's policies, no asset or contribution revenue will be recognized since the contributed photos are subject to major uncertainties with regard to their value and have no alternative use. Choices "1" and "4" are incorrect. The photos have no determinable value and will not be displayed. Regardless of the organization's policies with regard to capitalization of qualified historical treasures and works of art, no recognition of assets or contribution will be made. Choice "2" is incorrect. Bygone Historical Society is not obligated to disclose the contribution of the photos.

On December 31, Year 1, the Board of Directors of Maxy Manufacturing, Inc. committed to a plan to discontinue the operations of its Alpha division. The decision represents a major strategic shift and will have a significant effects on its operations and financial results. Maxy estimated that Alpha's Year 2 operating loss would be $500,000 and that the fair value of Alpha's facilities was $300,000 less than their carrying amounts. Alpha's Year 1 operating loss was $1,400,000, and the division was actually sold for $400,000 less than its carrying amount in Year 2. Maxy's effective tax rate is 30%. In its Year 1 income statement, what amount should Maxy report as loss from discontinued operations? a. $980,000 b. $1,700,000 c. $1,190,000 d. $1,400,000

Choice "3" is correct. Since the fair value of Alpha's facilities was $300,000 less than its carrying value, there has been an impairment loss, and that loss should be recognized in Year 1. That $300,000 impairment loss plus the $1,400,000 Year 1 operating loss would be recognized in Year 1 net of tax. The total loss would be $1,700,000 x 70% (100% - 30%) or $1,190,000. Choice "1" is incorrect. It includes the Year 1 operating loss of $1,400,000 but not the $300,000 impairment loss but does report the Year 1 operating loss net of tax. Choice "2" is incorrect. It reports the Year 1 loss from discontinued operations gross of tax and not net of tax. Choice "4" is incorrect. It includes the Year 1 operating loss of $1,400,000, but not the $300,000 impairment loss, and reports the Year 1 operating loss gross of tax and not net of tax.

A company has outstanding accounts payable of $30,000 and a short-term construction loan in the amount of $100,000 at year end. The loan was refinanced through issuance of long-term bonds after year end but before issuance of financial statements. How should these liabilities be recorded in the balance sheet? a. Current liabilities of $130,000 b. Long-term liabilities of $130,000 c. Current liabilities of $30,000, long-term liabilities of $100,000 d. Current liabilities of $130,000, with required footnote disclosure of the refinancing of the loan

Choice "3" is correct. The $30,000 account payable will be reported as a current liability. The $100,000 short-term construction loan will be reported as a long-term liability because the company refinanced the liability on a long-term basis prior to the issuance of the financial statements. Choice "1" is incorrect. The $100,000 short-term construction loan will be reported as a long-term liability because the company refinanced the liability on a long-term basis prior to the issuance of the financial statements. Choice "2" is incorrect. Accounts payable should not be classified as a long-term liability. Choice "4" is incorrect. Footnote disclosure of the refinancing is required. However, the $100,000 short-term construction loan will be reported as a long-term liability because the company refinanced the liability on a long-term basis prior to the issuance of the financial statements.

Selected amounts from Rufus Inc.'s December 31, Year 3 and December 31, Year 4 trial balances are: December 31, Year 3 December 31, Year 4 Sales $1,250,000 $1,320,000 COGS $860,000 $940,000 Inventory $280,000 $271,000 AP $86,000 $113,000 AR $124,000 $109,000 Rufus Inc.'s Year 4 statement of cash flows will report "cash payments for purchases" in the amount of: a. $958,000 b. $922,000 c. $904,000 d. $976,000

Choice "3" is correct. The calculation is: Cost of goods sold 940,000 - Decrease in inventory during period (9,000) - Increase in accounts payable during period (27,000) Cash payments for purchases 904,000 Choices "1", "2", and "4" are incorrect, per the above calculations.

For Year 1, Pac Co. estimated its two-year equipment warranty costs based on $100 per unit sold in Year 1. Experience during Year 2 indicated that the estimate should have been based on $110 per unit. The effect of this $10 difference from the estimate is reported: a. As an accounting change requiring Year 1 financial statements to be restated b. As a correction of an error requiring Year 1 financial statements to be restated c. In Year 2, as income from continuing operations d. As an accounting change, net of tax, below Year 2 income from continuing operations

Choice "3" is correct. The effect of the new estimate of warranty costs (from $100 to $110) is a change in estimate and will be reported in Year 2 income from continuing operations. Rule: Changes in estimates affect only the current and subsequent periods (not prior periods and not retained earnings). Choice "1" is incorrect. Restating prior years financial statements is only required when comparative financial statements are shown for prior period adjustments of subsequently discovered corrections of errors, changes in entity or changes in accounting principle. Choice "2" is incorrect. The facts stating a new estimate of warranty costs indicate a change of estimate, not a correction of an error. Choice "4" is incorrect. An accounting change of principle is shown net of tax on the retained earnings statement.

The City of Point Sparrow is preparing its government-wide financial statements for the year ended December 31, and reports total fund balances in its governmental funds of $1,200,000. The City had fixed assets of $4,000,0000 and accumulated depreciation of $2,275,000, along with long term debt of $2,000,000 related to its governmental funds. In addition, the government had a sales tax levy of 4% that is routinely collected by the State by the 30th of the month following the month of reported sales revenue and remitted by the State to the benefiting jurisdictions within 45 days of collection. Both the merchants and the State fully complied with the law in the months of October, November, and December when sales were 3,000,000, $4,000,000, and $5,000,000 respectively. What would the City of Point Sparrow display as its governmental activities Net Position at December 31? a. $450,000 b. $250,000 c. $650,000 d. $290,000

Choice "3" is correct. The fund balances of governmental funds reconcile to the net position of the governmental activities column of the government-wide financial statement using the CANS mnemonic. Consideration of assets net of accumulated depreciation and non-current debt as shown below represent the measurement focus issues, while the accrual of December revenues represent basis of accounting issues. December revenues are received by the city 75 days after year-end and therefore would not be accrued for modified accrual accounting since earnings are not available. However, these revenues would be recognized under accrual accounting since the earnings process is complete. Government-wide net position is computed as follows: GRaSPP—Fund Balance 1,200,000 Capital assets (non-current) 4,000,000 Accumulated depreciation (2,750,000) Non-current liabilities (2,000,000) Revenue — December sales 5,000,000 Sales tax 4.0% Accrual of sales tax revenue 200,000 Internal Service fund net position — Government-wide net position 650,000 Choices "4", "2", and "1" are incorrect, per the above computation.

An holder of a variable interest that is not the primary beneficiary acquired additional variable interests in the variable interest entity (VIE). What action, if any, should follow? a. No action is necessary because the primary beneficiary of a VIE does not change subsequent to the initial assessment b. The holder of the variable interest should reconsider whether it is now the primary beneficiary c. The holder of the variable interest should reconsider whether it is now the primary beneficiary d. The primary beneficiary should discontinue consolidation of the VIE because the election to consolidate is no longer allowed

Choice "3" is correct. The holder is the primary beneficiary if it has the power to direct the activities of a variable interest entity that most significantly impacts the entity's economic performance, and the company absorbs the expected VIE losses or receives the expected VIE residual returns. If the additional interest acquired in the VIE changes the terms, such that the holder now has the power to direct activities of the VIE, this should be considered in determining what party consolidates the VIE. Choice "1" is incorrect. Acquiring additional ownership in a VIE could change who the primary beneficiary of the VIE is. The VIE model is applied to determine if the primary beneficiary has changed as a result of the additional ownership. Choice "2" is incorrect. If an entity is identified as a variable interest entity, the ownership percentage does not necessarily indicate the primary beneficiary in the agreement. First, entities are subject to the variable interest entity model. Because this entity is already a VIE, the voting-interest model is insufficient to determine the party that should consolidate the VIE. Choice "4" is incorrect. Acquiring additional ownership in a VIE does not necessarily change who the primary beneficiary of the VIE is. The VIE model is applied to determine if the primary beneficiary has changed as a result of the additional ownership.

A firm granted stock options to its eligible employees on January 1, with each employee able to purchase 15,000 shares of $10 par value common stock after two years at a price of $20. Using the Black-Scholes options pricing model, the firm estimated that the individual employee stock options had a total fair value of $75,000. If a production exercises 50% of her stock options in the first quarter of the eligible period, what is the balance in the additional paid-in capital- stock options account at the end of the period (assume all remaining stock options are eligible)? a. $112,500 credit b. $0 c. $37,500 credit d. $75,000 credit

Choice "3" is correct. The journal entry for Year 1 and for Year 2 is as follows: Debit (Dr)Credit (Cr)Compensation expense 37,500 APIC—stock options 37,500 The journal entry to record the exercise of the stock options in the first quarter of the third year is: Debit (Dr)Credit (Cr)Cash (7,500 × $20) 150,000 APIC—stock options 37,500 Common stock (7,500 × $10) 75,000 APIC in excess of par-C/S 112,500 Over a two-year period, the additional paid-in capital—stock options accumulated $75,000 for each employee. When 50% of the stock options were exercised with the remaining still eligible, $37,500 of the additional paid-in capital—stock options amount was reversed, leaving a credit balance of $37,500. Choice "1" is incorrect. This amount represents the additional paid-in capital in excess of par (common stock) account. Choice "2" is incorrect. This account would have a $0 balance if 100% of the stock options were exercised or if 50% were exercised and the remaining options expired. Choice "4" is incorrect. This assumes no reversal when 50% of the stock options are exercised, as the original amounts recorded in Years 1 and 2 remain unchanged.

The primary purpose of a statement of cash flows is to provide relevant information about: a. Differences between net income and associated cash receipts and disbursements b. An enterprise's ability to generate future positive net cash flows c. The cash receipts and cash disbursements of an enterprise during a period d. An enterprise's ability to meet cash operating needs

Choice "3" is correct. The primary purpose of a statement of cash flows is to provide relevant information about the cash receipts and cash disbursements of an enterprise during an accounting period. Choice "1" is incorrect. The operating activities section of the statement of cash flows provides information about the differences between net income and associated cash receipts and disbursements. However, this is not the primary purpose of a statement of cash flows. Choice "2" is incorrect. The statement of cash flows is an analysis of historical cash flows rather than a projection of an enterprise's ability to generate future cash flows. Choice "4" is incorrect. The statement of cash flows is an analysis of historical cash flows rather than a projection of an enterprise's ability to meet cash operating needs.

Ridge Township's governing body adopted its general fund budget for the year ended July 31, 1994, comprised of estimated revenues of $100,000 and appropriations of $80,000. Ridge formally integrates its budget into the accounting records. To record the appropriations of $80,000, Ridge should: a. Credit estimated expenditures control b. Debt estimated expenditures control c. Credit appropriations control d. Debit appropriations control

Choice "3" is correct. To record the budget, the journal entry would be: Debit (Dr)Credit (Cr)Estimated revenues 100,000 Appropriations 80,000 Budgetary control 20,000 Choice "1" is incorrect. Estimated expenditures control is not an appropriate account title. Choice "2" is incorrect. Estimated expenditures control is not an appropriate account title. In addition, the $80,000 would be credited, not debited. Choice "4" is incorrect. Appropriations is credited, not debited.

An entity, upon initial recognition of an asset retirement obligation, should not take which of the following actions? A. Allocate asset retirement cost to expense over the useful life of the related asset B. Measure the asset retirement cost at fair value C. Capitalize the asset retirement cost by increasing the carrying amount of the related asset D. Capitalize the asset retirement cost at its undiscounted cash flow value

Choice "3" is correct. When an asset retirement obligation exists, the entity should record an asset retirement cost (ARC) which increases the carrying value of the long-lived asset as well as an asset retirement obligation (ARO), which is the liability recorded on the balance sheet related to the retirement. The amount recorded to both the asset and liability will be equal to the fair value of the asset retirement obligation (which is determined by discounting the future cash flows required). The ARC will be depreciated over the useful life of the related asset while the ARO will be "accreted" based on the relevant accretion rate. Choices "1", "4", and "2" are incorrect based on the above answer.

Due to a decline in market price in the second quarter, Petal Co. incurred an inventory loss. The market price is expected to return to previous levels by the end of the year. At the end of the year the decline had not reversed. When should the loss be reported in Petal's interim income statements? a. Ratably over the second, third, and fourth quarters b. Ratably over the third and fourth quarters c. In the fourth quarter only d. In the second quarter only

Choice "3" is correct. When the loss is probable and estimable, the expected loss must be recorded in full. This loss becomes such at the end of the fourth quarter. Therefore, the inventory must be valued on the year-end at the lower of cost or market, recognizing the loss at that time. Choice "1" is incorrect. Expected losses must be recorded in full when the loss is probable and estimable and not ratably over several quarters. Choice "2" is incorrect. Expected losses must be recorded in full when the loss is probable and estimable and not ratably over several quarters. Choice "4" is incorrect. Since the loss is not probable at the end of the second quarter, no amount should be recognized at that time.

PDX Corp. acquired 100% of the outstanding common stock of Sea Corp. in an acquisition transaction. The cost of the acquisition exceeded the fair value of the identifiable assets and assumed liabilities. The general guidelines for assigning amounts to the inventories acquired provide for: a. Work in process to be valued at the estimated selling prices of finished goods, less both costs to complete and costs of disposal. b. Raw materials to be valued at original cost. c. Finished goods to be valued at estimated selling prices, less both costs of disposal and a reasonable profit allowance. d. Finished goods to be valued at replacement cost.

Choice "3" is correct. With acquisition accounting the net assets acquired are based on fair market value. The fair value of finished goods and merchandise inventory are based upon selling price less disposal costs and a reasonable profit allowance. Choice "2" is incorrect. The fair value of raw materials should be based upon replacement costs. Choice "3" is incorrect. Replacement costs are an appropriate measure of fair market value for raw materials inventory, but not finished goods. Choice "4" is incorrect. The fair value of work in process should be based upon the estimated selling price of finished goods less the costs to complete and dispose and a reasonable profit allowance.

A statement of financial position for a nongovernmental not-for-profit organization reports amounts for which of the following classes of net assets? a. With donor restrictions b. Conditionally restricted c. Long-Term d. Current

Choice "4" is correct. $408,000 merchandise purchased plus freight-in, insurance, and warehousing costs (net of purchase returns and cash discounts) are inventoriable costs. Freight out is a selling expense which is treated as a period expense, not a product (inventoriable) cost.

On July 1, Year 1, Rey Corp. purchased computer equipment at a cost of $360,000. This equipment was estimated to have a 6-year life with no residual value and was depreciated by the straight-line method. On January 3, Year 4, Rey determined that this equipment could no longer process data efficiently, that its value had been permanently impaired, and that $70,000 could be recovered over the remaining useful life of the equipment. What carrying amount should Rey report on its December 31, Year 4, balance sheet for this equipment? a. $150,000 b. $0 c. $70,000 d. $50,000

Choice "4" is correct. $50,000 carrying amount in Dec. 31, Year 4 balance sheet. This is a change in "accounting estimate" that should be reported in the period of change and future periods. The net carrying value as of Jan. 3, Year 4 is estimated to be $70,000 and would be depreciated over the remaining useful life of 3.5 years ($70,000/3.5 = $20,000/year). The carrying amount would therefore be $50,000 as of Dec. 31, Year 4. Choice "1" is incorrect. $150,000 would be the carrying amount based on the original cost of $360,000 without considering the impairment in value. Choice "2" is incorrect. $0, since the computer still has value. Choice "3" is incorrect. $70,000 would be the revised carrying value at the beginning of Year 4.

Dunbarn Co. had the following activities during the year: Purchase of inventory $120,000 Purchase of equipment 80,000 Purchase of available-for-sale securities 60,000 Purchase of treasury stock 70,000 Issuance of common stock 150,000 What amount should Dunbarn report as cash provided (used) by investing activities in its statement of cash flows for the year? a. ($120,000) b. ($210,000) c. $150,000 d. ($140,000)

Choice "4" is correct. Cash flows from investing activities come from transactions involving the purchase or sale of non-current assets. Both equipment and available-for-sale (AFS) security purchases are cash flows used by investing activities. Purchase of equipment 80,000 Purchase of AFS security 60,000 Total 140,000 outflow Choice "1" is incorrect. This choice counts only the purchase of inventory as a cash flow used by investing activities. Inventory is a current asset, which means this cash outflow would factor into the cash flow from operations calculation. Choice "2" is incorrect. This choice includes the purchase of treasury stock, which is usually a financing cash outflow. Choice "3" is incorrect. This choice only counts the issuance of common stock, which is a financing cash inflow.

Which of the following is a cost associated with exit and disposal activities? a. Benefits related to voluntary employee termination b. Costs to terminate a capital lease c. Costs associated with the retirement of a fixed asset d. Costs to relocate employees

Choice "4" is correct. Costs to relocate employees are costs associated with exit and disposal activities. Choice "1" is incorrect. Exit and disposal activities include benefits related to involuntary (not voluntary) employee termination. Choice "2" is incorrect. Exit and disposal activities include costs to terminate a contract that is not a capital lease. Capital lease termination costs are accounted for separately from exit and disposal activities. Choice "3" is incorrect. The cost of retiring a fixed asset is not considered an exit or disposal cost.

Quentin University is organized as a not-for-profit organization. The university reaches out to its alumni each year in a mass telephonic fund raising appeal that includes scripted dinner hour calls appealing for ongoing support. In Year 1, the University used untrained student volunteers to make the calls. In Year 2, an alumnus who owns a successful telemarketing company offers to perform the task. The university accepts the offer and provides the alumnus with the script along with appropriate phone numbers and contributions accounting forms. Based on the usual and customary charges used in his business, the alumnus anticipates that the value of these services is $10,000. For each year, contributed revenue associated with this transaction would be: Year 1 Year 2 a. $10k $10k b. $0 $10k c. $10k $0 d. $0 $0

Choice "4" is correct. Donated services are recorded as contribution revenue "SOME" of the time: when a specialized skill is required, when the donation is otherwise needed and purchased and when measurable easily. Although the purchase price of the telemarketing appeal can be measured, it does not require a specialized skill (it can be performed by college students) and would not be purchased if it were not donated (the college would use its student volunteers). Choices "2", "1", and "3" are incorrect. Donated services are recorded as contribution revenue SOME of the time: when a specialized skill is required, when the donation is otherwise needed and purchased and when measurable easily. Contribution revenue would not be recognized in either year.

On January 1, Year 1, Billy Co. signed a three-year lease for office space. The lease required monthly rent of $8,000 for the first year, $8,100 for the second year and $8,200 for the third year. Billy has the option to renew the lease for a fourth year at $8,300 per month. On January 1, Year 2, Billy permanently installed new overhead lighting fixtures at a cost of $2,000. How much amortization expense on this leasehold improvement should Billy record for Year 2? a. $667 b. $2,000 c. $0 d. $1,000

Choice "4" is correct. In accordance with the matching principle, the $2,000 leasehold improvement should be amortized over the period of expected benefit. The amortization period would be two years. $2,000 / 2 = $1,000 Choices "2", "1", and "3" are incorrect, per the above discussion.

When Rolan County adopted its budget for the year ending June 30, Year 1, $20 million was recorded for estimated revenues control. Actual revenues for the year ended June 30, Year 1, amounted to $17 million. In closing the budgetary accounts at June 30, Year 1. a. Revenues control should be credited for $20,000,000 b. Revenues control should be debited for $3,000,000 c. Estimated revenues control should be debited for $3,000,000 d. Estimated revenues control should be credited for $20,000,000

Choice "4" is correct. In closing the budgetary accounts at June 30, Year 1, the estimated revenues control account should be credited for $20,000,000 to offset the $20,000,000 debit balance that existed before closing. Debit (Dr)Credit (Cr)Appropriations control 20,000,000 Estimated revenues control 20,000,000

How should plan investments be reported in a defined benefit plan's financial statements? a. At actuarial present value b. At net realizable value c. At cost d. At fair value

Choice "4" is correct. Pension plan investment assets must be reported at fair value in a defined benefit plan's financial statements. Choices "1", "3", and "2" are incorrect. Actuarial present value, cost, and net realizable value are not appropriate bases for measuring pension plan asset investments.

Which of the following is true regarding the comparison of managerial to financial accounting? a. Managerial accounting has a past focus and financial accounting has a future focus b. Managerial accounting is generally more precise c. The emphasis on managerial accounting is relevance and the emphasis on financial accounting is timeliness d. Managerial accounting need not follow generally accepted accounting principles (GAAP) while financial reporting must follow them

Choice "4" is correct. Public companies must follow GAAP for (external) financial reporting purposes. GAAP need not be followed for (internal) managerial accounting purposes. Choice "1" is incorrect. Managerial accounting has a future focus, while financial accounting focuses on reporting past results. Choice "2" is incorrect. Financial accounting is generally more precise. Choice "3" is incorrect. The emphasis of financial accounting is providing useful information to financial statement users (including the characteristic of relevance), while the emphasis of managerial accounting is providing timely information to management decision makers.

Which of the following characteristics of service efforts and accomplishments is the most difficult to report for a governmental entity? a. Comparibility b. Consistency c. Timeliness d. Relevance

Choice "4" is correct. Relevance means the information must bear a logical relationship with the needs for its purpose. It must also be reliable. The achievement of this reporting objective is more difficult to establish than it is to demonstrate consistency (adherence to the consistent application of GAAP from year to year) or comparability (the use of GAAP in a manner comparable to other governments). The timeliness objective requires that reports be issued early enough to facilitate timely decisions. The timeliness of reporting is relatively easy to demonstrate. Choice "1" is incorrect. Demonstrating that governments use the same principles of accounting as other governments is relatively easy to report. Choice "2" is incorrect. Demonstrating that a government consistently uses the same accounting principles year over year is relatively easy to report. Choice "3" is incorrect. Demonstrating timeliness is relatively easy to report in relation to reporting dates and year end.

Thyme Inc. owns 16,000 of Sage Co.'s 20,000 outstanding common shares. The carrying value of Sage's equity is $500,000. Sage subsequently issues an additional 5,000 previously unissued shares for $200,000 to an outside party that is unrelated to either Thyme or Sage. What is the total noncontrolling interest after the additional shares are issued? a. $140,000 b. $172,000 c. $300,000 d. $252,000

Choice "4" is correct. Sage's equity (assets - liabilities) is $500,000 prior to the stock issuance. When Sage issues the additional shares, equity increases by $200,000 to $700,000. Thyme Inc. still owns 16,000 shares, but outstanding shares total 25,000 after issuance. Thyme owns 64 percent of the stock (16,000 / 25,000); therefore the noncontrolling interest (the portion Thyme does not own) is 36 percent. $700,000 × 36% = $252,000. Choice "1" is incorrect. The $140,000 answer uses the correct equity amount of $700,000 but multiplies it by a 20 percent noncontrolling interest, which was the noncontrolling interest prior to the stock issuance ((20,000 - 16,000) / 20,000). Choice "2" is incorrect. The $172,000 answer does not use the correct equity value of $700,000 or the correct noncontrolling interest percentage of 36 percent. Choice "3" is incorrect. The $300,000 answer accounts for the stock issuance as a reduction of equity and does not calculate the noncontrolling interest portion of equity.

Seafood Trading Co. commenced operations during the year as a large importer and exporter of seafood. The imports were all from one country overseas. The export sales were conducted as drop shipments and were merely transshipped at Seattle. Seafood Trading reported the following data: Purchases during the year $12.0 million Shipping costs from overseas 1.5 million Shipping costs to export customers 1.0 million Inventory at year end 3.0 million What amount of shipping costs should be included in Seafood Trading's year-end inventory valuation? a. $625,000 b. $0 c. $250,000 d. $375,000

Choice "4" is correct. The $1.5 million in overseas shipping costs must be allocated between ending inventory and cost of goods sold at year end, as follows: Beginning inventory 0.0 Purchases 12.0 Goods available for sale 12.0 Ending inventory 3.0 Cost of goods sold 9.0 So, of the inventory purchased during the year, 75% ($9/$12) was sold and 25% ($3/$12) remained in ending inventory at year end. Therefore, of the shipping costs from overseas, $1.125 million ($1.5 million x 75%) should be included in cost of goods sold and $375,000 ($1.5 million x 25%) should be included in ending inventory. The $1.0 million in shipping costs to export customers are a selling cost that will be included in SG&A expenses.

The City of Sharpton budgeted $230,000 for capital equipment purchases in its police department for the current budget year. The City issued a purchase order on January 1 for the acquisition of five police cars. EAch vehicle is priced at $40,000. The City received four of the vehicles on June 30 for $180,000. On its June 30 interim General Fund financial statements the City would report: a. Budgetary control of $20,000 b. Available appropriations of $30,000 c. PPE and additions of $180,000 d. Capital outlay expenditures of $180,000

Choice "4" is correct. The City of Sharpton would record capital acquisitions in its General Fund as capital outlay expenditures. The amount of the capital outlay expenditure is equal to the amount spent or obligated, $180,000. Choice "1" is incorrect. A budgetary control account for encumbered appropriations would be equal to the amount of the unused encumbrance, $40,000 Budgetary control - Encumbered appropriations ($40,000 × 5) 200,000 Less: Completed order ($40,000 × 4) 160,000 Budgetary control - Encumbered appropriations 40,000 Unused encumbrances do not specifically appear on the face of the financial statements. Encumbrance amounts would likely appear in a line item titled either "committed" or "assigned" fund balance. Choice "2" is incorrect. The City's available appropriations in its capital outlay line item are computed as follows: Appropriations 230,000 Capital Outlay expenditures 180,000 Encumbered appropriations (unfilled purchase orders) 40,000 Less: Total expended or encumbered 220,000 Available Appropriations 10,000 Choice "3" is incorrect. Property and equipment would not be displayed in the governmental funds.

On January 1, Year 1, an entity recorded an asset retirement obligation of $162,120. The asset retirement obligation is expected to be paid at the end of ten years. The entity uses straight-line depreciation and an accretion rate of 8%. What is the total Year 1 expense related to the asset retirement obligation? a. $3,242 b. $12,970 c. $16,212 d. $29,182

Choice "4" is correct. The asset retirement obligation is recorded with the following journal entry: Debit (Dr)Credit (Cr)Asset retirement cost 162,120 Asset retirement obligation 162,120 The asset retirement cost is depreciated using straight-line depreciation over 10 years: Depreciation expense = $162,120 / 10 years = $16,212 Accretion expense is recorded as the asset retirement obligation increases over time: Year 1 accretion expense = $162,120 × 8% = $12,970 (rounded) Therefore, the total Year 1 expense is: Total ARO expense = $16,212 depreciation + $12,970 accretion = $29,182

Footnote disclosures in the financial statements for pensions do not require inclusion of which of the following? a. The amount of unrecognized prior service cost b. The components of net periodic pension cost c. A detailed description of the plan including employee groups covered d. The differences in executive and non-executive plans

Choice "4" is correct. The difference in executive and non executive plans is not a required disclosure. Choice "1" is incorrect. The amount of unrecognized prior service cost must be disclosed. Choice "2" is incorrect. The components of net periodic pension cost "SIRAGE" is a required disclosure. Choice "3" is incorrect. Disclosure showing a detailed description of the plan including employee groups covered is required.

On January 2, Year 1, Marx Co. as lessee signed a five-year noncancelable equipment lease with annual payments of $200,000 beginning December 31, Year 1. Marx treated this transaction as a finance lease. The five lease payments have a present value of $758,000 at January 2, Year 1, based on interest of 10%. What amount should Marx report as interest expense for the year ended December 31, Year 1? a. $0 b. $48,400 c. $55,800 d. $75,800

Choice "4" is correct. The lease term began January 2, Year 1, on a lease valued at $758,000. The first payment of $200,000 was made on December 31, Year 1. Since the interest rate is 10% and one year has expired, Marx Co.'s interest expense is computed as 10% of $758,000 or $75,800. The remainder of the $200,000 payment reduces the obligation under the lease. Choice "1" is incorrect. If the first payment had been made on January 2, the amount of interest would have been $0 because none of the lease term had elapsed. Interest accrued between January 2 and December 31, which Marx must account for. Choice "2" is incorrect. Marx will pay 5 × $200,000 or 1,000,000 over the life of the lease or $1,000,000 − $758,000 = $242,000 total interest over the lease term. Simple interest for each of the five years would be $242,000 / 5 or $48,400. However, lease interest expense is computed using the effective interest method. Choice "3" is incorrect. If the first payment had been made on January 2, the amount of interest would have been $0 because none of the lease term had elapsed, and the lease obligation would have been reduced by the $200,000 of the payment leaving an obligation of $558,000. Under these circumstances the interest expense accrued between January 2 and December 31, which Marx must account for, would be 10% of the reduced amount or $55,800.

First Commercial Bank received a cash contribution to establish an irrevocable charitable trust for the sole benefit of the Olde Towne Museum, a not-for-profit organization. The terms of the trust name First Commercial as the trustee, require distribution of income earned on the trust each year for three years, and, at the end of the three-year period, require distribution of the principal to the museum. The museum would account for this transaction as follows: a. Museum would recognize an equity interest in the assets of the trust fair value as a contribution with donor restrictions b. Museum would recognize the asset as a conditional net asset with donor restrictions subject to the terms of the trust c. Museum would not recognize the asset in the year the trust was established, but would recognize distributed earnings each year as earnings without donor restrictions d. Museum would recognize a beneficial interest in the assets of the trust at fair value as a contribution with donor restrictions

Choice "4" is correct. The museum and the bank are not financially interrelated so the trust would be recognized as an asset with donor restrictions (a beneficial interest) and as a contribution with donor restrictions. Choice "1" is incorrect. The museum and the bank are not financially interrelated so the trust would not be displayed as an equity interest. Choice "2" is incorrect. This is a distracter. Conditional contributions are dependent upon achievement of a condition prior to recognition. Conditional contributions are displayed as liabilities. Since conditional contributions are not recognized, they will not be classified as net assets. Choice "3" is incorrect. The museum would recognize the asset as a beneficial interest and then display income and changes in the fair value of the asset as with donor restrictions. Distributed earnings, absent any donor restrictions, would be recorded as without donor restrictions.

What are the two required financial statements of a defined contribution retirement plan? a. A statement of fiduciary net assets and a statement of changes in fiduciary net assets b. A statement of financial position and a statement of activities c. A statement of net assets available for benefits of the plan and a statement of change in fiduciary net assets d. A statement of net assets available for benefits of the plan and a statement of changes in ent assets available for benefits

Choice "4" is correct. The only two statements that are required for a defined contribution plan are the statement of net assets available for benefits of the plan and the statement of changes in net assets available for benefits. Choice "1" is incorrect. The statement of fiduciary net assets and statement of changes in fiduciary net assets are required for custodial funds under the governmental accounting framework. Choice "2" is incorrect. The statement of financial position and statement of activities are required financial statements for not-for-profit entities. Choice "3" is incorrect. The statement of changes in fiduciary net assets is applicable to custodial funds.

A nongovernmental not-for-profit organization's statement of activities is similar to which of the following for-profit financial statements? a. Statement of retained earnings b. Statement of cash flows c. Balance Sheet d. Income Statement

Choice "4" is correct. The statement of activities is most similar to an income statement. The not-for-profit organization's statement of activity shows revenues, expenses and changes in net assets (net income). Choice "1" is incorrect. The statement of retained earnings does not have a correlating stand-alone financial statement in not-for-profit settings. Although the statement of activities shows changes in each classification of net assets, there is no statement exclusively devoted to equity transactions. Choice "2" is incorrect. The statement of cash flows for not-for-profits is generally the same as the statement of cash flows in commercial settings. Choice "3" is incorrect. The statement of financial position for not-for-profits is the closest to a balance sheet. The statement shows assets, liabilities and net assets (equity).

Douglas Co. leased machinery with an economic useful life of 6 years. For tax purposes, the depreciable life is 7 years. The lease is for 5 years, and Douglas can purchase the machinery at fair market value at the end of the lease. What is the depreciable life of the leased machinery for financial reporting purposes? a. Six years b. Zero c. Seven years d. Five years

Choice "4" is correct. This lease will be accounted for as a finance lease because the lease term of five years is 75% of the economic life of six years. Assets acquired under finance leases are depreciated using the same theory as purchased assets. The purchase option is for "fair value," which means it cannot be assumed without further information that Douglas will be reasonably certain to exercise. As a result, the machinery should be depreciated over the five-year lease term. Choice "1" is incorrect. Although the machine has an economic life of six years, Douglas should depreciate the machinery over the five-year lease term because Douglas will most likely not keep the machinery after the lease term (there is no indication that Douglas will exercise the fair value purchase option). Choice "2" is incorrect. Assets acquired under a finance lease must be depreciated. Choice "3" is incorrect. Tax life is not used for financial accounting purposes.

On December 1, Line Corp. received a donation of 2,000 shares of its $5 par value common stock from a stockholder. On that date, the stock's market value was $35 per share. The stock was originally issued for $25 per share. By what amount would this donation cause total stockholders' equity to decrease? a. $70,000 b. $50,000 c. $20,000 d. $0

D $0 decrease in total stockholder's equity due to donation of its own stock from a stockholder because there is no cost to the corp

On November 1, Year 1, Dixon Corporation issued $800,000 of its 10-year, 8% term bonds dated October 1, Year 1. The bonds were sold to yield 10%, with total proceeds of $700,000 plus accrued interest. Interest is paid every April 1 and October 1. What amount should Dixon reports for interest payable in its December 31, Year 1 balance sheet? a. $10,667 b. $11,667 c. $17,500 d. $16,000

D (800k * .08 * (3/12))

One business partner signs a loan agreement with another business partner, the terms of which state that $25,000 is to be paid back in five annual installments of $5,000. There is no stated interest rate, but an appropriate rate based on comparable transactions is 6 percent (PV of annuity for 5 years at 6% = 4.21). Using the effective interest method, calculate the total amount of the loan payback that represents interest? a. $0 b. $4,500 c. $1,500 d. $3,950

D 5 * 4.21 = 21050 25k - 21050 = 3950

Band Co. uses the equity method to account for its investment in Guard, Inc. common stock. How should Band record a 2% stock dividend received from Guard? a. As dividend revenue at Guard's carrying value of the stock b. As a reduction in the total cost of Guard Stock owned c. As dividend revenue at the market value of the stock d. As a memorandum entry reducing the unit cost of all Guard Stock owned

D Bank should record the 2% stock dividend received from Guard with a memorandum entry that reduces the unit cost of all Guard stock owned. The total investment in Guard, Inc. will simply be spread over a larger amount of shares, thereby reducing the unit cost of all Guard stock owned

On July 1 of the current year, Dewey Co. signed a 20-year building lease that is reported as a capital lease. Dewey paid the monthly lease that is reported as a capital lease. Dewey paid the monthly lease payments when due. How should Dewey report the effect of the lease payments in the financing activities section of its statement of cash flows? a. An inflow equal to the PV of future lease payments at July 1, less current year principal and interest payments b. An outflow equal to the current year principal and interest payments on the lease c. The lease payments should not be reported in the financing activities section d. An outflow equal to the current year principal payments only

D Cash payments made to reduce debt principal are properly reported as a financing activity. Cash interest payments would be reported as a component of cash from operating activities

The Town of Thomasville recorded more appropriations than estimated revenues for the coming fiscal year. In integrating its adopted budget with its financial accounting records, the town would: a. Credit reserve for encumbrances b. Credit budgetary control c. Debit reserve for encumbrances d. Debit budgetary control

D Deficit

Farleigh Co. has not declared or paid dividends on its cumulative preferred stock in the last three years. These dividends should be reported: a. As a reduction in stockholder's equity b. As a current liability c. As a non-current liability d. In a note to the financial statements

D Dividends in "arrears" (undeclared dividends on cumulative preferred stock) should be reported in the footnotes. Since they are not declared, no journal entry is made. Neither liabilities nor equity are affected

Wind Co. incurred organization costs of $6,000 at the beginning of its first year of operations. How should Wind treat the organization costs in its financial statements in accordance with GAAP? a. Amortized over 60 months b. Never amortized c. Amortized over 40 years d. Expensed immediantely

D Organizational costs expensed for GAAP financial income (no asset) but deducted in later years for tax purposes

At December 31, Year 1, Taos Co. estimates that its employees have earned vacation pay of $100,000. Employees will receive their vacation pay in Year 2. Should Taos accrue a liability at December 31, Year 1, if the rights to this compensation accumulated over time or if the rights are vested? Accumulated Vested a. N Y b. Y N c. N N d. Y Y

D Rule: Employees' compensation for future absences (mostly vacation) should be accrued if: 1) Services have already been rendered, and 2) The obligation relates to vested or accumulated rights, and 3) The amount can be reasonably estimable, and 4) Payment is probable

The following information pertains to Meg Corp.: -Dividends on its 1,000 shares of 6%, $10 par value cumulative preferred stock have not been declared or paid for 3 years -Treasury stock that cost $15,000 was reissued for $8,000 What amount of retained earnings should be appropriated as a result of these items? a. $8,800 b. $7,000 c. $1,800 d. $0

D Rule: There is no requirement to appropriate retained earnings for any purpose. Retained earnings may be set aside for future purposes by classifying a portion as "appropriated"

Slate Co. and Talse Co. exchanged similar plots of land with fair values in excess of carrying amounts in an exchange that lacks commercial substance under U.S. GAAP. In addition, Slate received cash of less than 10% of the total consideration received from Talse to compensate for the difference in land values. As a result of the exchange, Slate should recognize: a. A gain equal to the difference between the fair value and the carrying amount of the land given up b. A loss in an amount determined by the ratio of cash received to total consideration c. Neither a gain nor a loss d. A gain in an amount determined by the ratio of cash received to total consideration

D This transaction is a nonmonetary exchange that lacks commercial substance under U.S. GAAP. As such, the transaction is an exception to the general rule of basing the measurement value of the exchange on fair value. In this question, as in many such questions on the CPA exam, cash(boot) is received. Because the cash is less than 10% of the total consideration, a proportional amount of the gain is recognized

In a business combination accounted for as a purchase, the appraised values of the identifiable assets acquired exceeded the acquisition price. How would the excess appraised value be reported? a. As positive goodwill b. As a reduction of the values assigned to non-current assets and a loss for any unallocated portion c. As negative goodwill d. As a gain, after adjusting the balance sheet, including identifiable intangible assets, to fair value

D When a sub is acquired with an acquisition cost that is less than the fair value of the underlying assets, the following steps are required: 1) The balance sheet is adjusted to fair value, which creates a negative balance in the acquisition account 2) Identifiable intangible assets are recognized at fair value, which increases the negative balance in the acquisition account 3) The total negative balance in the acquisition account is recorded as a gain

A corporation issues quarterly financial statements and uses the lower of cost or market method to value its inventory in its annual financial statements. Which of the following is correct regarding how the corporation should value its inventory in its interim financial statements? a. Temporary market declines should be recognized in the interim statements b. Gains from valuations in previous interim periods should be fully recognized c. Only the cost method of valuation should be used d. Inventory losses generally should be recognized in the interim statements

D Permanent declines in inventory market value should be reflected in interim financial statements in the period incurred

Assuming no outstanding encumbrances at year's end, closing entries for which of the following situations would increase the unassigned fund balance at year's end? A - Actual revenues were less than estimated revenues. B - Estimated revenues exceed actual appropriations. C - Actual expenditures exceed appropriations. D - Appropriations exceed actual expenditures.

RULE: Unassigned fund balance represents the amount of current resources carried forward into the following year that will be available for appropriations. Assuming there are no encumbrances at year end and expenditures were less than appropriations, any commitment or assignment of fund balance for encumbrances would be released and unassigned fund balance would increase. Note that the Budgetary Comparison Schedule illustrated for Progressive Township in your text shows that the "Final" budgeted fund balance at the beginning of the year is equal to the "Actual" fund balance at the beginning of the year. The examiners' question effectively asks when the relationships between budgeted and actual results create an actual fund balance greater than expected. When appropriations are greater than expenditures (expenditures are less than appropriations) unassigned fund balance will likely increase. Choice "4" is correct. Appropriations (estimated expenditures) in excess of actual expenditures would increase the unassigned fund balance. Not all of the original budget (appropriations) was spent, and the excess (assumed to be unencumbered by the fact pattern) was returned to fund balance. Choice "1" is incorrect. The relationship between estimated revenues exceeding actual appropriations is a comparison of budgeted amounts that does not give sufficient information to draw any conclusion with regard to the impact on unassigned fund balance. The "actual" appropriations is a distracter. Choice "2" is incorrect. Actual revenues less than estimated revenues would indicate that the unassigned fund balance would decrease. There was effectively a revenue shortfall, and it had to come out of fund balance. Choice "3" is incorrect. Actual expenditures in excess of appropriations would decrease the unassigned fund balance. More money was spent than was anticipated.

The Jupiter's Children's Center, a not-for-profit organization, receives state grants to fund its ongoing programming. Grants and annual contracts that are structured as cost reimbursement agreements requiring expenditure of grant funding before revenues are deemed to be earned. Assuming the Jupiter Children's Center expended all grant funds in accordance with program requirements, the center would likely record grant revenues as: a. Support (contribution revenue) without donor restrictions, provided that the center discloses and consistently applies this accounting policy b. Revenue without donor restrictions and a reclassification that decreases net assets with donor restrictions and increases net assets without donor restrictions c. A refundable advance and a reclassification to support without donor restrictions d. Conditional revenue with donor restrictions

Rule: Donor-imposed restrictions that are met in the same period they are received may be recorded as support (contribution revenue) without donor restrictions, provided that the organization discloses and consistently applies this accounting policy. Choice "1" is correct. The grant revenues received by the center represent earnings with donor-imposed restrictions that are routinely satisfied in the same period in which they are received. Provided the center consistently applies the policy of recognizing the receipt of earnings for ongoing programming as without donor restrictions, and discloses the policy, otherwise donor-restricted revenue may be displayed purely as support without donor restrictions. Choice "2" is incorrect. Recording receipts with donor restrictions that are met in the same period as net assets with donor restrictions with associated reclassifications is appropriate, but generally needlessly cumbersome. Not-for-profit organizations are most likely to adopt a policy that allows receipts with donor restrictions whose requirements are satisfied in the period of receipt to be recognized in the current period as net assets with donor restrictions. Choice "3" is incorrect. Refundable advances relate to deposits associated with conditional contributions. Classification of otherwise unconditional donor-restricted revenues as a refundable advance would not be appropriate. Choice "4" is incorrect. Conditional contributions are not recognized as revenue until conditions are met. This answer is a distracter.

In applying the criteria used for determination of major funds required for reporting in a government's fund financial statements, a government would consider which of the following statistics? Agg Rev or Expenditures Agg. Assets or Liabilities Agg fund balance a. Y Y N b. N Y Y c. Y N N d. Y Y Y

Rule: The criteria for determining major funds includes qualification as to revenues, expenditures/expenses, assets, or liabilities that are at least 10 percent of the associated total for ALL governmental OR enterprise (as appropriate) AND at least 5 percent of the total of the associated totals for ALL governmental AND enterprise funds. Choice "1" is correct. Governments evaluate the classification of major funds using aggregate revenues and expenditures/expenses and aggregate assets and liabilities. Choices "4", "3", and "2" are incorrect. Governments evaluate the classification of major funds using aggregate revenues and expenditures/expenses and aggregate assets and liabilities per the rule above, not fund balance/equity.

Converge Corp. sold a $100,000 bond at 95 and incurred $3,000 of bond issuance costs. Which of the following statements is correct? a. The discount of $5,000 is amortized using the straight-line method over the life of the bond b. A debit to cash is booked for $100,000 c. The discount of $8,000 us amortized using the effective interest method over the life of the bond d. An asset is booked for $3,000

C Bond issuance costs are deducted from the carrying value of the liability and included in the debit entry to bond discount upon issuance. As part of the discount from par, bond issuance costs are amortized over the life of the bond using the effective interest method. The initial entry upon issuance is: Cash 92k Discount 8k Bonds Payable 100K

Which of the following items would not be included in the operating activities section of an entity's statement of cash flows under U.S. GAAP? a. Income taxes paid b. Proceeds from the sale of trading securities c. Dividends paid d. Interest received

C Dividends paid are a financing cost and are included in the financing section of the statement of cash flows

The market price of a bond issued at a discount is the present value of its principal amount at the market (effective) rate of interest: a. Less the present value of all future interest payments at the rate of interest stated on the bond b. Plus the present value of all future interest payments at the rate of interest stated on the bond c. Plus the present value of all future interest payments at the market (effective) rate of interest d. Less the present value of all future interest payments at the market (effective) rate of interest

C The market value of a bond issued at a discount (or a premium) is the present value of two cash flows 1) The PV of the principal, plus 2) The present value of all future interest payments, 3) Both at the market (effective) rate of interest

All of the following distributions to stockholders are considered asset or capital distributions, except: a. Liquidating dividends b. Property distributions c. Stock splits d. Cash dividends

C A stock cplit increases the number of shares. Only the number of share changes. The capital stock and retained earnings do not change. It is not considered a capital or asset distribution

The senior accountant for Carlton Co., a public company with a complex capital structure, has just finished preparing Carlton's income statement for the current fiscal year. While reviewing the income statement, Carlton's finance director noticed that the earnings-per-share data has been omitted. What changes will have to be made to Carlton's income statement as a result of the omission of the earnings-per-share data? a. Carlton's income statement will only have to be revised to include the earnings per share data if Carlton's market capitalization is greater than $500,000 b. Carlton's income statement will have to be revised to include the earnings per share data if Carlton's net income for the past two years was greater than $5,000,000 c. Carlton's income statement will have to be revised to include in the earnings per share data d. No changes will have to be made to Carlton's income statement. The income statement is complete without the earnings per share data

C All public entities must present EPS on the face of the income statement. In a simple capital structure, basic EPS for income from continuing operations and net income are presented. In a complex capital structure, basic and diluted EPS must be presented for income from continuing operations and net income

Each of the following events is required to be reported to the United States Securities and Exchange Commission on Form 8-K, except: a. The unregistered sale of equity securities b. A change in a registrant's certifying accountant c. The quarterly results of operations and financial condition of a registrant d. The creation of a registration under an off-balance sheet arrangement of a registrant

C Form 8-K is a form required to be filed by all companies with the SEC. The form reports on major corporate events, including corporate asset acquisition/disposals, accountant changes, financial statement changes, management changes, changes in securities, etc. Quarterly results of operations will be reported on the form 10-Q

If a corporation sells some of its treasury stock at a price that exceeds its cost, this excess should be: a. Reported as a gain in the income statement b. Credited to retained earnings c. Credited to additional paid-in capital d. Treated as a reduction in the carrying amount of remaining treasury stock

C Rule: There is no gain or loss on the purchase and/or sale of treasury stock. Any "difference" goes to "paid-in capital", or if there is not enough paid-in capital to absorb a loss, the loss would be debited (subtracted) from retained earnings

Which of the following transactions should be classified as investing activities on an entity's statement of cash flows? a. Issuance of common stock to the shareholders b. Increase in accounts receivable c. Sale of PPE d. Payment of cash dividend to the shareholders

C The purchase of PPE will fall under cash flow from investing

Louis, Inc. acquired 40% of the outstanding non-voting preferred stock of Rich Co. What method for recording the investment should Louis use? a. The equity method because significant influence must be assumed b. The equity method if no other investor has more than a 40% interest c. The fair value method d. The equity method if it can acquire an additional 11% by year-end

C Significant influence cannot be exercised by holding non-voting stock. The fair value method must be used

Which of the following is reported as interest expense? a. Deferred compensation plan interest b. Interest incurred to finance a software development for internal use c. Amortization of discount of a note d. Pension cost interest

C When a discount on a bond or note is amortized, the discount amortization increases interest expense for the period

Which of the following circumstances would indicate that an entity has insufficient level of equity investment at risk? a. The entity's equity investment at risk is less than the equity investment at risk of similar non-VIE entities b. The fair value of the equity investment at risk is greater than expected losses c. The entity can finance its own activities d. The facts and circumstances indicate that there is sufficient equity at risk

Choice "1" is correct. An entity has insufficient equity investment at risk if the entity's equity investment at risk is less than the equity investment at risk of similar non-VIE entities. An entity has sufficient equity investment at risk when the entity's equity investment at risk is at least as much as the equity investment of other non-VIE entities that hold similar assets of similar quality. Choice "2" is incorrect. An entity has sufficient equity investment at risk if the fair value of the equity investment at risk is greater than expected losses. Choice "3" is incorrect. An entity has sufficient equity investment at risk if the entity can finance its own activities. Choice "4" is incorrect. An entity has sufficient equity investment at risk if the facts and circumstances indicate that there is sufficient equity at risk.

On January 1, Read, a nongovernmental not-for-profit organization, received $20,000 and an unconditional pledge of $20,000 for each of the next four calendar years to be paid on the first day of each year. The present value of an ordinary annuity for four years at a constant interest rate of 8 percent is 3.312. What amount of net assets with donor restrictions is reported in the year the pledge was received? a. $66,240 b. $86,240 c. $80,000 d. $100,000

Choice "1" is correct. Read would display net assets with donor restrictions of $66,240. Multiyear pledges are recorded at their net present value at the date the pledge is made. Thus, multiyear pledges should be discounted. Multiyear pledges have an implied time restriction and are classified as with donor restrictions. The question requires that we presume that the discount rate given is appropriate and reasonable so the net present value of a four year annuity beginning on the first of the next year (end to the current year) discounted at 8 percent using the ordinary annuity factor is: $20,000 × 3.312 = $66,240 Choice "2" is incorrect. The proposed solution appears to compound the annuity rather than discount it to its present value in error. Choice "3" is incorrect. The proposed response of $80,000 does not discount the annuity in error. The cash receipts should be discounted to the net present value. Choice "4" is incorrect. The proposed solution appears to consider the annuity for a five year period instead of four and, furthermore, does not discount the income stream in error.

Which of the following accounts of a governmental unit is credited when taxpayers are billed for property taxes? a. Revenues b. Estimated revenues c. Taxes receivable current d. Appropriations

Choice "1" is correct. Revenues are credited and Property Taxes Receivable are debited when taxpayers are billed for property taxes. Note that the revenues are recorded net of estimated uncollectibles. Choice "2" is incorrect. Estimated revenues are credited at fiscal year-end when the estimated revenues and appropriations are closed. Choice "3" is incorrect. Taxes receivable-current are credited when cash is received. Choice "4" is incorrect. Appropriations are credited when the budget is recorded at the beginning of the budget year.

How are discontinued operations that occur at midyear initially reported? a. Included in net income and disclosed in the notes to interim financial statements b. Included in net income and disclosed in the notes to the year-end financial statements c. Disclosed only in the notes to interim financial statements d. Disclosed only in the notes to the year-end financial statements

Choice "1" is correct. To adequately capture the impact of discontinued operations, it should be included in net income and disclosed in the interim financial statement notes.

Interim financial reporting should be viewed primarily in which of the following ways? a. As reporting under a comprehensive basis of accounting other than GAAP b. As reporting for an integral part of an annual period c. As if the interim period were an annual accounting period d. As useful only if activity is spread evenly throughout the year

Choice "2" is correct. Interim financial reporting should be viewed as reporting for an integral part of an annual period. Choices "4", "3", and "1" are incorrect, per the above rule.

For interim financial reporting, the computation of a company's second quarter provision for income taxes uses an effective tax rate expected to be applicable for the full fiscal year. The effective tax rate should reflect anticipated: Foreign Tax rates Available tax planning alternatives a. N Y b. Y Y c. Y N d. N N

Choice "2" is correct. Yes - Yes. The effective income tax rates for operations for the full year should reflect anticipated foreign tax rates and available tax planning alternatives. In addition, the effect of other anticipated tax credits, capital gains rates, and foreign tax credits should be included.

Under East Co.'s accounting system, all paid insurance premiums are debited to prepaid insurance. For interim financial reports, East makes monthly estimated charges to insurance expense with credits to prepaid insurance. Additional information for the year ended December 31, 20X5, is as follows: Prepaid insurance at December 31, 20X4 $105,000 Charges to insurance expense during 20X5 (including a year-end adjustment of 17,500) 437,500 Prepaid insurance at December 31, 20X5 122,500 What was the total amount of insurance premiums paid by East during 20X5? A. $322,500 B. $420,000 C. $437,500 D. $455,000

Choice "3" is correct. $455,000 insurance premiums paid during Year 2. PrepaidInsurance Begin balance at 12/31/Year 1 105,000 Add payments 455,000 Sub Total 560,000 Less expense (437,500) Ending balance at 12/31/Year 2 122,500

After three profitable years, Dodd Co. decided to offer a bonus to its branch manager, Cone, of 25% of income over $100,000 earned by his branch. For Year 4, income for Cone's branch was $160,000 before income taxes and Cone's bonus. Cone's bonus is computed on income in excess of $100,000 after deducting the bonus, but before deducting taxes. What is Cone's bonus for Year 4? a. $32,000 b. $15,000 c. $12,000 d. $25,000

Choice "3" is correct. A $12,000 bonus will be paid to the branch manager. The bonus is calculated as follows: Bonus = ($160,000 − $100,000 − Bonus) × 25% Bonus = ($60,000 − Bonus) × 25% Bonus = $15,000 − 25% Bonus 125% Bonus = $15,000 Bonus = $12,000 Choices "2", "4", and "1" are incorrect, per the above.

On November 2, Year 1, Platt Co. entered into a 90-day futures contract to purchase 50,000 Swiss francs when the contract quote was $.70. The purchase was for speculation in price movement. The following exchange rates existed during the contract period: 30 day futures Spot Rate Nov 2, Year 1 .62 .63 Dec 31, Year 1 .65 .64 Jan 30, Year 2 .65 .68 What amount should Platt report as foreign currency exchange loss in its income statement for the year ended December 31, Year 1? a. $3,500 b. $4,000 c. $2,500 d. $3,000

Choice "3" is correct. Any gain or loss on futures contracts not designated as a hedge is recognized in current income. The loss as of December 31, Year 1 is: Contract rate $.70 30-day futures rate on 12/31/Year 1 .65 .05 x 50,000 SF = $2,500 loss The December 31, Year 1 30-day futures rate is used since, as of December 31, Year 1, 30 days remain in the contract period. If the futures contract were purchased at year-end instead of on November 2, Year 1, 5¢ per Swiss franc would have been saved. It would have only cost us 65¢ per Swiss franc instead of our contract rate of 70¢; therefore, 5¢ per Swiss franc was recorded as a loss. Choice "1" is incorrect. This loss is calculated using the November 2, Year 1 spot rate. It is not appropriate to use November 2nd rates to calculate the loss on December 31st. The loss is equal to the difference between the contract price and the 30-day futures rate on December 31, Year 1. The 30-day futures rate must be used because 30 days remain in the contract period on December 31, Year 1. Choice "2" is incorrect. This loss is calculated using the 30-day futures rate on November 2, Year 1. The loss on December 31, Year 1 must be calculated as the difference between the contract price and the value of a 30-day futures contract on December 31, Year 1. Choice "4" is incorrect. This loss is calculated using the December 31, Year 1 spot rate. It is not appropriate to use the spot rate because 30 days remain in the contract period at December 31, Year 1.

Conlon Co. is the plaintiff in a patent-infringement case. Conlon has a high probability of a favorable outcome and can reasonably estimate the amount of the settlement. What is the proper accounting treatment of the patent infringement case? a. A gain contingency for the estimated probable settlement b. A gain contingency for the minimum estimated amount of the settlement c. Disclosures of the notes only d. No reporting unit is required at this time

Choice "3" is correct. Contingencies that might result in gains are not accrued per the principle of conservatism. As this potential favorable outcome is probable, the amount and nature should be disclosed in the notes to the financials.

Abbott Co. is preparing its statement of cash flows for the year. Abbott's cash disbursements during the year included the following: Payment of interest on bonds payable $500,000 Payment of dividends to stockholders 300,000 Payment to acquire 1,000 shares of Marks Co. common stock 100,000 What should Abbott report as total cash outflows for financing activities in its statement of cash flows under U.S. GAAP? a. $800,000 b. $900,000 c. $300,000 d. $0

Choice "3" is correct. The $300,000 dividend to stockholders should be classified as a financing cash outflow. Other financing activities would include the issuance of stock, the purchase of treasury stock, the issuance of bonds, borrowing funds, and paying back principal related to bonds and loans. The payment of interest on the bonds payable is an operating cash outflow under U.S. GAAP and the payment to acquire the common stock of Marks is an investing cash outflow. Choice "1" is incorrect. The $500,000 payment of interest, although related to a financing activity, is not reported in the financing section under U.S.GAAP. It is reported as an operating cash outflow. Choice "2" is incorrect. The $100,000 payment to acquire someone else's common stock should be classified as an investing activity. Choice "4" is incorrect. The $300,000 dividend to stockholders should be classified as a financing cash outflow.

The following information applied to Fenn, Inc. for the current year: Merchandise purchased for resale $400,000 Freight in $10,000 Freight out $5,000 Purchase Returns $2,000 Fenn's current year inventoriable cost was: a. $403,000 b. $400,000 c. $413,000 d. $408,000

Choice "4" is correct. $408,000 merchandise purchased plus freight-in, insurance, and warehousing costs (net of purchase returns and cash discounts) are inventoriable costs. Freight out is a selling expense which is treated as a period expense, not a product (inventoriable) cost.

Which of the following is an intangible asset that is subject to the recoverability test when testing for impairment? a. R&D Costs for a patent b. A trademark with indefinite useful life c. Goodwill d. Patent

Choice "4" is correct. A patent is a type of intangible asset that has a limited useful life. The recoverability test is only performed on intangible assets with a limited life. The recoverability test compares undiscounted future cash flows to the carrying value of the asset. If the carrying value is greater, then a fair value test would be performed. Choice "1" is incorrect. R&D costs are expensed immediately; therefore, impairment is irrelevant as no intangible asset would exist to test for impairment. Choice "2" is incorrect. This specific trademark is an intangible asset with an indefinite life; therefore, the recoverability test is not performed. Choice "3" is incorrect. Goodwill is an intangible asset with an indefinite life; therefore, the recoverability test is not performed.

A retail store sold gift certificates that are redeemable in merchandise. The gift certificates lapse one year after they are issued. How would the deferred revenue account be affected by each of the following? Redemption of certificates Lapse of Certificates a. D NE b. NE Decrease c. NE NE d. DE DE

Choice "4" is correct. Deferred revenue represents future income collected in advance. When the gift certificates are sold, deferred revenue is increased. When the certificates are redeemed, the revenue is earned and shown in the income statement. Deferred revenue is decreased. When the certificates lapse, the company has no further liability and revenue is earned. Deferred revenue is decreased.

Giaconda, Inc. acquires an asset for which it will measure the fair value by discounting future cash flows of the asset. Which of the following terms best describes this fair value measurement approach? a. Observable inputs b. Market c. Cost d. Income

Choice "4" is correct. The income approach converts future amounts, including cash flows or earnings to a single discounted amount to measure fair value. Choice "1" is incorrect. Observable inputs is not a fair value measurement approach. Observable inputs are inputs other than quoted market values that are directly or indirectly observable for the asset or liability. Choice "2" is incorrect. The market approach uses prices and other relevant information from identical or comparable market transactions to measure fair value. Choice "3" is incorrect. The cost approach uses current replacement cost to measure fair value

Blunder Bus Corp. has an $85,000 balance in its Deferred Tax Asset account. Blunder has determined that is more likely than not that $50,000 of its deferred tax asset will not be realized in the future. Under U.S. GAAP, Blunder should: a. Decrease this period's income tax expense by $35,000 b. Debit their "Deferred Tax Valuation Allowance" account for $50,000 c. Credit their "Deferred Tax Valuation Allowance" account for $35,000 d. Increase this period's income tax expense by $50,000

Choice "4" is correct. Writing down a deferred tax asset increases (debits) income tax expense and increases (credits) the Deferred Tax Valuation Allowance in the amount that is expected to go unrealized. Choices "1", "2", and "3" are incorrect, per the above explanation.

Which of the following statements describes the proper accounting for losses when nonmonetary assets are exchanged for other nonmonetary assets? a. A loss can occur only when assets are sold or disposed of in a monetary transaction b. A loss is deferred so that the asset received in the exchange is properly valued c. A loss, if any, which is unrelated to the determination of the amount of the asset received should be recorded d. A loss is recognized immediately, because assets received should not be valued at more than their cash price

D Under the rule of conservatism, losses are recognized in all nonmonetary exchanges when the book value exceeds the fair value of the asset given up.

Which form is not required to include audited financial statements? a. Form 40-F b. Form 20-F c. Form 10-K d. Form 6-K

D Form 6-K is filed semiannually by foreign private issuers and contains unaudited financial statements. Form 10-Q, which is filed by U.S. registered companies, also contains unaudited financial statements

Which of the following transactions qualify as a discontinued operation? a. Changes related to technological improvements b. Disposal of part of a line of business c. Phasing out of a production line d. Planned and approved sale of a segment

D A planned and approved sale of a segment qualifies as a discontinued operation because a segment is a component of an entity

On March 4, Year 1, Evan Co. purchased 1,000 shares of LVC common stock at $80 per share. On September 26, Year 1, Evan received 1,000 stock rights to purchase an additional 1,000 shares at $90 per share. The stock rights had an expiration date of February 1, Year 2. On September 30, Year 1, LVC's common stock had a market value, ex-rights, of $95 per share and the stock rights had a market value of $5 each. What amount should Evan report on its September 30, Year 1, balance sheet for investment in stock rights? a. $4,000 b. $5,000 c. $10,000 d. $15,000

Formula: Cost of Stock rights = (FMV of rights/(FMV of rights + FMV of stock ex - rights)) * Cost of stock 5k/(5k + 95K) * 80000 Choice "1" is correct. $4,000 investment in stock rights. The purchase price of the stock should be allocated between the stock and the stock rights using a pro rata allocation based on the relative fair values of the stock and the stock rights.

Potterville Charities is staging its annual fund raising appeal. The organization views this as a major ongoing activity that serves to fund the mission of the organization. In exchange for a $500 donation, contributors receive a hand embossed flowerpot valued at $15. Potterville Charities will recognize contributions for each such donation in the amount of: a. $485 b. $500 c. $15 d. $0

RULE: The Statement of Activities shall report gross amounts of revenues and expenses. The cost of premiums given to acknowledge donations is classified as a fund-raising expense. Choice "2" is correct. Potterville Charities would display the contribution revenue of $500 gross and display the $15 as fund raising expense. Choice "1" is incorrect. Generally the difference between the fair value of dues or other purchases and the amount transferred is classified as a contribution. When, however, the contributions relate to a major ongoing portion of the operation of the organization, the contribution revenues are displayed gross and the cost of the premium is displayed as fund-raising expense. Choice "3" is incorrect. The cost of the premium is not recorded as the contribution revenue. Choice "4" is incorrect. The organization has earned contribution revenue that should be recorded as support. To not record contribution revenue would imply that the collections represent revenue from an exchange transaction. This is clearly incorrect from the fact pattern and the discrepancy between the amount of the donation and the nominal cost of the premium.

Feline County established the Calico Landfill as a separate legal entity to operate the city's solid waste landfill. The landfill's operating budget and capital improvement projects are subject to the approval of the county commission. The financial statement of the Calico Landfill should be: a. Presented as a blended component unit b. Displayed as a special purpose government c. Only disclosed in the notes to the financial statements d. Presented discretely as a component unit

Rule: A special purpose government is a government that stands by itself: Separately Elected Legally separate Financially independent Component units that do not meet the criteria for blended reporting (governing boards of component and primary government are substantially the same or the component unit exclusively serves the primary government) are reported discretely. Choice "4" is correct. The Calico Landfill does not meet the SELF criteria and is therefore a component unit. Furthermore, the landfill does not meet the criteria for blended reporting (governing boards of component and primary government are substantially the same or the component unit exclusively serves the primary government) and is therefore reported as a discrete component unit. Choice "1" is incorrect. The landfill does not meet the criteria for blended reporting (governing boards of the component units and primary government are substantially the same or the component unit exclusively serves the primary government) and is reported as a discrete component unit. Choice "2" is incorrect. The Landfill is financially accountable to the county and is not a stand-alone special purpose government. Choice "3" is incorrect. The Landfill's financial statements are presented as a discrete component unit, not simply disclosed.

The Appleton Museum, a not-for-profit organization, sells a painting to its collection to a private investor. The museum plans to use the proceeds associated with the sale to purchase additional works of art for display. In its statement of cash flows, the museum typically would display proceeds from the sale as cash flows from: a. Donor-restricted resources, which are not displayed on the face of the statement of cash flows b. Operating activities c. Financing activities d. Investing activities

Rule: Investing activities include proceeds from the sale of works of art or purchases of works of art. Choice "4" is correct. Sales of collections are displayed on the statement of cash flows within investing activities Choices "2", "3", and "1" are incorrect, per the above explanation.

On December 1, Year 1, Money Co. gave Home Co. a $200,000, 11% loan. Money paid proceeds of $194,000 after the deduction of a $6,000 nonrefundable loan origination fee. Principal and interest are due in 60 monthly installments of $4,310, beginning January 1, Year 2. The repayments yield an effective interest rate of 11% at a present value of $200,000 and 12.4% at a present value of $194,000. What amount of income from this loan should Money report in its Year 1 income statement? a. $0 b. $1,833 c. $2,005 d. $7,833

Rule: Loan origination fees shall be deferred and recognized over the life of the loan as an adjustment of interest income (similar to the treatment of bond discount amortization). Choice "3" is correct. $2,005 income from this loan in Year 1. Face amount of loan 200,000 Nonrefundable loan origination fee (6,000) Net amount loaned 194,000 Effective interest rate (yield) 12.4% 24,056 Outstanding one month (12/1/Y1 - 12/31/Y1) × 1/12 Interest income for Year 1 2,005

A segment of Ace Inc. was discontinued during Year 1. Ace's loss from discontinued operations should not: a. Exclude operating losses from the date the decision to dispose of the segment was made until the end of Year 1 b. Include operating losses of the current period up to the date the decision to dispose of the segment was made c. Include employee relocation costs associated with the decision to dispose d. Include additional pension costs associated with the decision to dispose

A All year 1 operating losses should be included

Ward, a consultant, keeps her accounting records on a cash basis. During Year 2, Ward collected $200,000 in fees from clients. At December 31, Year 1, Ward had accounts receivable of $40,000. At December 31, Year 2, Ward had accounts receivable of $60,000, and unearned fees of $5,000. On an accrual basis, what was Ward's service revenue for Year 2? a. $215,000 b. $175,000 c. $180,000 d. $225,000

A Cash basis rev $200 +End AR 60k - Beg AR -40 - End Unearned fees -5 = 215k

On which of the following dates is a public entity required to measure the cost of employee services in exchange for an award of equity interests, based on the fair market value of the award? a. Date of grant b. Date of exercise c. Date of vesting d. Date of restriction lapse

A Equity instruments issued for employee services are to be valued at the date of the grant

What is the appropriate treatment for goods held on consignment? a. The goods should be included in ending inventory of the consignor b. The goods should be included in cost of goods sold of the consignee only when sold c. The goods should be included in cost of goods sold of the consignor when transferred to the consignee d. The goods should be included in ending inventory of the consignee

A While an agent (consignee) will hold and sell goods on behalf of the consignor, until the inventory is sold, the seller (consignor) will include in his/her inventory because title and risk of loss are retained by the consignor

A property dividend should be recorded in retained earnings at the property's: a. Market value at date of declaration b. Book value at date of issuance (payment) c. Market value at date of issuance (payment) d. Book value at date of declaration

A A property dividend should be reported in RE at the property's market value at date of declaration

In determining earnings per share, interest expense, net of applicable income taxes, on dilutive convertible debt should be: a. Added back to net income for diluted earnings per share, and ignored for basic earnings per share b. Deducted from net income for both basic earnings per share and diluted earnings per share c. Added back to net income for basic earnings per share and diluted earnings per share d. Deducted from net income for basic earnings per share, and ignored for basic diluted earnings per share

A Interest expense (Net of income tax) on convertible debt would be added back to diluted EPS if the effects are dilutive

Lyon Company has the following transactions in the current year. Assuming that all of the transactions are​ material, which of them will most likely have no effect on current year net​ income? a. The settlement of litigation over an accident which occured in the prior year but for which a loss had not previously been considered to be probable and reasonably estimable b. The collection of a receivable from a customer whose account was written off in the period prior year by a charge to the allowance for bad debts account c. The sale of a factory building that was contributed by a shareholder in the prior year d. The determination that certain junk bonds purchased on a speculative basis several years previous were worthless

B

How should a local government's internal service fund report depreciation expense in its fund financial statements? a. Nonoperating expense b. Operating expense c. Separate from revenues and expense d. Not reported

B Depreciation is recognized as a component of operating expenses in proprietary funds such as internal service funds

Gown Inc. sold a warehouse and used the proceeds to acquire a new warehouse. The excess of the proceeds over the carrying amount of the warehouse sold should be reported as a: a. Gain from discontinued operations, net of income taxes b. Part of continuing operations c. Reduction of the cost of the new warehouse d. Reduction of the cumulative depreciation expense on the old warehouse

B When a fixed asset is sold, gain or loss is recognized as part of income from continuing operations. The amount of the gain or loss is equal to the difference between the proceeds from the sale and the carrying amount of the fixed asset sold

Which of the following should be included in general and administrative expenses? Interest Advertising a. Y Y b. N N c. N Y d. Y N

B Interest Expense is classified as a separate line item on the income statement. Advertising is classified as a selling expense

Eureka City should issue a statement of cash flows for which of the following funds? Eureka City Hall CP Fund Eureka Water Enterprise Fund a. Y N b. Y Y c. N Y d. N N

C A SOCF should be issued for all proprietary funds, no SOCF should be issued for either Governmental funds that include Capital Projects in CIPPOE

Hunt Community Development Agency (HCDA), a financially independent authority, provides loans to commercial businesses operating in Hunt County. This year, HCDA made loans totaling $500,000. How should HCDA classify the disbursements of loans on the cash flow statement? a. Non-capital financing activities b. Investing activities c. Operating activities d. Capital and related financing activities

C Cash activities associated with the ongoing and central activities of an entity would be classified as operating activities on its statement of cash flows. The HCDA is in the business of providing loans through a loan program. Cash activities associated with the HCDA's program loans, consistent with its charter, would be classified as operating activities

On December 2 of the current year, Huff Corp. received a condemnation award of $450,000 as compensation for the forced sale of land purchased five years earlier for $300,000. The gain was not reported as taxable income on its income tax return for the year ended December 31 because Huff elected to replace the land within the allowed replacement period for at least $450,000. Huff has an income tax rate of 25% for the current year, and there is an enacted tax rate of 21 percent for future years. There were no other temporary differences. In its December 31 balance sheet, Huff should report a deferred income tax liability of: a. $31,500 b. $0 c. $94,500 d. $37,500

Choice "1" is correct, $31,500. Rule: Establish deferred tax using enacted annual tax rate (current tax rate for the related years) when temporary differences (will) reverse. Deferred tax liability on temporary difference: Sales price of asset 450,000 Cost 300,000 Gain on sale 150,000 Enacted future tax rate × 21% Deferred tax liability 31,500 In this case, gain on sale is taxable in a future year when the enacted tax rate is 21 percent.

Eagle and Falk are partners with capital balances of $45,000 and $25,000, respectively. They agree to admit Robb as a partner. After the assets of the partnership are revalued, Robb will have a 25 percent interest in capital and profits, for an investment of $30,000. What amount should be recorded as a bonus to the original partners? a. $5,000 b. $0 c. $20,000 d. $7,500

Choice "1" is correct, $5,000. Total equity of new partnership: Eagle 45,000 Falk 25,000 Robb 30,000 Total 100,000 Since Robb is receiving a 25 percent interest in the partnership, his capital account will be credited with 25 percent of the total equity of the new partnership, or $25,000. The difference between his contribution of $30,000, and his capital account balance of $25,000, is credited to the other partners as a bonus.

During Year 1, Gum Co. introduced a new product carrying a two-year warranty against defects. The estimated warranty costs related to dollar sales are 2% within 12 months following the sale and 4% and in the second 12 months following the sale. Sales and actual warranty expenditures for the years ended December 31, Year 1 and Year 2, are as follows: Sales Actual Warranty expenditures Year 1 $150,000 $2,250 Year 2 $250,000 $7,500 What amount should Gum report as estimated warranty liability in its December 31, Year 2, balance sheet? a. $14,250 b. $2,500 c. $11,250 d. $4,250

Choice "1" is correct. $14,250. SalesWarranty Liability Total Year of sale2% Year after4% Year 1 150,000 3,000 + 6,000 = 9,000 Year 2 250,000 5,000 + 10,000 = 15,000 Total estimated warranty 24,000 Less: Actual warranty expenditures 9,750 Estimated warranty liability December 31, Year 2 14,250

On December 31, Year 1, Rice, Inc. authorized Graf to operate as a franchisee for an intitial franchise fee of $150,000. Of this amount, $60,000 was received upon signing the agreement and the balance, represented by a note, is due in three annual payments of $30,000 each beginning December 31, Year 2. The present value on December 31, Year 1, of the three annual payments appropriately discounted is $72,000. According to the agreement, the nonrefundable down payment represents a fair measure of the services already performed by Rice; however, substantial future services are required of Rice. Collectibility of the note is reasonably certain. In Rice's December 31, Year 1, balance sheet, unearned franchise fees from Graf's franchise would be reported as: a. $72,000 b. $90,000 c. $100,000 d. $132,000

Choice "1" is correct. $72,000 unearned franchise fee liability at 12/31/Year 1. Franchise Fees EarnedUnearned Nonrefundable down payment represents a fairmeasure of the services already performed 60,000 Present value of 3 annual payments requiresubstantial future services 72,000 Full JE would be: Cash 60,000 Note receivable 90,000 Revenue 60,000 Discount on note receivable 18,000 Unearned franchise revenue

In accounting for its merchandise invnetory, Ingewald International, a company that uses U.S. GAAP, changed from LIFO to FIFO. Assuming the change in beginning inventory was $400,000 and that the change at the end of the year was $300,000 and that the tax rate was 30 percent, what was the amount of the cumulative effect of an accounting change that should have been displayed in Inglewald's retained earnings statement? a. $280,000 b. $210,000 c. $70,000 d. $0

Choice "1" is correct. A change from LIFO to another inventory cost flow assumption requires a cumulative catch-up adjustment as of the beginning of the year of change. The beginning balance of retained earnings is adjusted, net of tax. 400,000 × (1 − .30) = 280,000. A change from LIFO to another method is reported by restating prior period financial statements in a manner similar to a prior period adjustment. Choice "2" is incorrect. Adjustments are made as of the beginning, not the end of the period. Choice "3" is incorrect. Taking the difference between the beginning and ending inventory, net of tax, results in this incorrect amount. Choice "4" is incorrect. An adjustment must be made when switching from LIFO.

Which of the following is not a disclosure requirement related to risks and uncertainties under U.S. GAAP? a. Disclosure of vulnerability due to all identified contracts b. Estimates of the effects of changes in significant estimates c. A statement that actual results could differ from the estimates included in the financial statements d. Disclosure of the relative importance of each business when an entity operates multiple businesses

Choice "1" is correct. Identified concentrations only need to be disclosed if all of the following criteria are met: The concentration exists at the financial statement date. The concentration makes the entity vulnerable to the risk of a near-term severe impact. It is at least reasonably possible that the events that could cause the severe impact will occur in the near-term. Choice "2" is incorrect. This is a disclosure requirement. Choice "3" is incorrect. This is a disclosure requirement. Choice "4" is incorrect. This is a disclosure requirement.

The budget for the City of Goodville for the year ending December 31 was adopted and recorded on January 2 of the same year. After recording the budget, the accounting records showed a debit balance of $50,000 in the Budgetary Fund Balance account. What does this indicate? A. Appropriations are $50,000 greater than estimated revenues. B. Estimated revenues are $50,000 greater than appropriations. C. Appropriations are $50,000 greater than revenues. D. Revenues are $50,000 greater than appropriations.

Choice "1" is correct. A debit balance in budgetary control accounts would indicate that appropriations are in excess of budgeted revenues. Budgetary accounts have natural balances that are opposite of actual accounts. Recording the budget would result in the following entry (with hypothetical amounts: Appropriations = 100, Estimated revenues = 80): Debit (Dr)Credit (Cr)Estimated revenues 80 Appropriations 100 Budgetary control (plug) 20 Choice "1" is incorrect. The relationship between budgetary accounts (appropriations) and actual accounts (revenues) would have no impact on budgetary control accounts. Choice "2" is incorrect. A debit balance in budgetary control accounts would indicate that appropriations are in excess of budgeted revenues. Budgeted revenues in excess of appropriations would result in a credit to budgetary control. Choice "3" is incorrect. The relationship between budgetary accounts (appropriations) and actual accounts (revenues) would have no impact on budgetary control accounts.

In which of the following cases would a municipality record a lease using a right-of-use asset account? a. A proprietary fund entering into an agreement as a lessee in an agreement properly accounted for as a lease other than a short-term lease and contract that transfers ownership b. A proprietary fund entering into an agreement as lessee in an agreement properly accounted for as a contract that transfers ownership c. A governmental fund entering into an agreement as a lessee in an agreement properly accounted for as a lease other than a short term lease and contract that transfers ownership d. A proprietary fund entering into an agreement as a lessor in an agreement properly accounted for as a lease other than a short-term lease and contract that transfers ownership

Choice "1" is correct. A proprietary fund that is a lessee in an agreement that is other than a short-term lease and contract that transfers ownership would record a lease liability and an intangible asset titled right-of-use asset. The entry to record the lease is: DRRight-of-use asset $XXX CR Lease liability $XXX Choice "2" is incorrect. A proprietary fund that is a lessee in an agreement that is a contract that transfers ownership would record a lease liability and the value of the acquired asset as it would for any other financed purchase. Choice "3" is incorrect. Governmental funds that record leases as the lessee party to a lease would record a capital outlay expenditure and other financing sources, not a right-of-use asset. Choice "4" is incorrect. A proprietary fund that is a lessor in an agreement that is other than a short-term lease and contract that transfers ownership would record a receivable and deferred inflow of resources, not a right-of-use asset.

Which of the following activities should be excluded when governmental fund financial statements are converted to government-wide financial statements? a. Fiduciary activities b. Enterprise activities c. Proprietary activities d. Government activities

Choice "1" is correct. Fiduciary activities are excluded from the government-wide financial statements. Transactions with fiduciary funds are treated as if those transactions were conducted with an independent trustee for purposes of display in the government-wide financial statements. Choice "2" is incorrect. Enterprise funds are displayed as business type activities in the government-wide financial statements. Choice "3" is incorrect. Proprietary activities may show up in both governmental and business type activities displayed in the government-wide financial statements. Enterprise funds will be displayed as business type activities while internal services funds will generally be included in governmental type activities, presuming that the internal service funds provide support to general governmental activities. Choice "4" is incorrect. Governmental activities would be displayed in the government-wide financial statements. Generally speaking governmental activities include governmental funds (GRaSPP) and the internal service funds.

The following costs were incurred by Griff Co., a manufacturer, during the current year: Accounting and legal fees $25,000 Freight-In $175,000 Freight-out $160,000 Officer Salaries $150,000 Insurance $85,000 Sales Rep Salaries $215,000 What amount of these costs should be reported as General and administrative expenses for the current year? a. $260,000 b. $810,000 c. $635,000 d. $550,000

Choice "1" is correct. General and administrative expenses include: Accounting and legal 25,000 Officers salaries 150,000 Insurance 85,000 Total 260,000 Freight-in is part of cost of sales; freight-out is a selling expense; and sales salaries are selling expenses. Choice "2" is incorrect. Freight-in is part of cost of inventory; freight-out is a selling expense; and sales salaries are selling expenses. Choice "3" is incorrect. Freight-in is part of cost of inventory; freight-out is a selling expense; and sales salaries are selling expenses. Choice "4" is incorrect. Freight-in is part of cost of inventory; freight-out is a selling expense; and sales salaries are selling expenses.

The statement of activities of the government wide financial statements is designed primarily to provide information to assess which of the following? a. Operational accountability b. Financial accountability c. Fiscal accountability d. Functional accountability

Choice "1" is correct. Government-wide financial statements focus on operational accountability. Rule: The focus of government-wide financial statements is the government's responsibility to report the extent to it has met its operating objectives efficiently and effectively, using all resources available for that purpose, and the extent to which it can continue to meet its objectives for the future. Choice "2" is incorrect. Although all financial statements serve to demonstrate financial accountability in some way, the government-wide financial statements most specifically focus on operational accountability. Choice "3" is incorrect. Fund financial statements focus on fiscal accountability. Fund financial statement accountability objectives complement government-wide financial statement accountability objectives. Choice "4" is incorrect. Functional accountability is a distracter.

Property taxes levied in fiscal Year 1 to finance the general fund budget of fiscal year Year 2 should be reported as general fund revenues in fiscal year Year 2: a. For the amount collected before the end of fiscal year Year 2 or shortly thereafter b. For the amount collected in fiscal year Year 2 only c. Regardless of the fiscal year in which collected d. For the amount collected before the end of fiscal year Year 2 only

Choice "1" is correct. Property taxes levied in fiscal year Year 1 to finance the general fund budget of fiscal year Year 2 should be reported as general fund revenues in fiscal year Year 2 for the amount collected before the end of fiscal year Year 2 or shortly thereafter. The treatment is similar to commercial accounting, but does not record bad debts as revenues, in spite of the fact that potential "property tax bad debts" are more assured of eventual collection because the property is subject to sheriff sales for collection. Choice "2" is incorrect. Modified accrual allows for some accrual. Governmental revenue is recognized when measurable and available, namely collected during the year and shortly thereafter (generally 60 days after year end). Choice "3" is incorrect. The year of collection is significant. Governmental revenue is recognized when measurable and available, namely collected during the year and shortly thereafter (generally 60 days after year end). Choice "4" is incorrect. Modified accrual allows for some accrual. Governmental revenue is recognized when measurable and available, namely collected during the year and shortly thereafter (generally 60 days after year end).

Shared revenues received by an enterprise fund of a local government for operating purposes should be recorded in fund financial statements as: a. Non-operating revenues b. Other financing sources c. Interfund transfers d. Operating revenues

Choice "1" is correct. Shared revenues are revenues levied by one government but shared on a predetermined basis with another government. Shared revenues received by a proprietary fund should be recorded as non-operating revenue if received for operations or if received for either operations or capital expenditures at the discretion of the recipient governmental unit. Choice "2" is incorrect. "Other financing sources" is not a proper classification used in the financial statements of enterprise funds. Choice "3" is incorrect. Shared revenues are not classified as interfund transfers. Choice "4" is incorrect. Shared revenues are revenues levied by one government but shared on a predetermined basis with another government. Shared revenues received by proprietary fund types are not classified as operating revenues.

In a business combination, the valuation of goodwill is a calculation: a. Of the residual paid above the fair value of the identifiable assets b. Of all of the increases in market valuation of the intangible assets acquired c. To offset the bargain purchase cost d. Of all of the unlimited-life intangible assets

Choice "1" is correct. The amount of goodwill recorded on the balance sheet by an acquiring firm for a business combination represents the excess of the price paid over the fair value of the identifiable net assets acquired. Choice "2" is incorrect. The amount of goodwill recorded in a business combination is not determined by just the increases in market valuation of the intangible assets being acquired. Choice "3" is incorrect. The bargain purchase cost does not affect the valuation of goodwill on an acquiring firm's balance sheet. Choice "4" is incorrect. The calculation of goodwill involves the net assets (assets minus liabilities) of the firm being acquired, not just the unlimited-life intangible assets.

Haft Construction Co. has consistently used the percentage-of-completion method. On January 10, Year 1, Haft began work on a $3,000,000 construction contract. At the inception date, the estimated cost of construction was $2,250,000. The following data relate to the progress of the contract: Income recognized at 12/31/Year 1 $300,000 Costs incurred 1/10/Year 1 through 12/31/Year 2 $1,800,000 Estimated cost to complete at 12/31/Year 2 $600,000 In its income statement for the year ended December 31, Year 2, what amount of gross profit should Haft report? a. $150,000 b. $300,000 c. $262,500 d. $450,000

Choice "1" is correct. The gross profit for the percentage-of-completion method is as follows: Contract price 3,000,000 Cost to date 1,800,000 Estimated cost to complete 600,000 Total cost 2,400,000 Expected gross profit 600,000 Percentage complete (18/24) 75% Profit to date 450,000 Profit previously recognized (300,000) Year 2 profit 150,000 Choice "2" is incorrect. $300,000 is the earned profit in Year 1. The profit earned in Year 2 is to be determined. Choice "3" is incorrect. The total gross profit as of December 31, Year 2 must be computed to determine the profit to be recorded in Year 2. Choice "4" is incorrect. $450,000 is the total profit earned to date. Only profit earned in Year 2 is to be determined.

During the year, the City of Exeter reported a $20,000 net increase in fund balance for governmental funds. Also during the year, Exeter purchased general capital assets of $50,000, and recorded depreciation expense of $35,000. What amount should Exeter report as the change in net position for governmental activities in the government-wide financial statements? a. $35,000 b. $5,000 c. $70,000 d. $20,000

Choice "1" is correct. The reconciliation of the change in fund balances in governmental fund financial statements to the change in net position for governmental activities in the government-wide financials is computed using the CPAS RIDES mnemonic. Net Changes in Fund Balance $ 20,000 Capital outlay 50,000 Principal payments on long-term debt 0 Asset disposals 0 Sources—other financing 0 Revenue 0 Interest expense 0 Depreciation Expense (35,000) Internal Service fund net revenue 0 Change in net position of government activities $ 35,000 Choice "2" is incorrect. Capital outlay is added back to the change in fund balance and depreciation is subtracted as per above. Choice "3" is incorrect. The change in fund balance reconciles to the change in net position as a result of not only adding back the capital outlay expenditure but also from subtracting the depreciation expense not included in governmental fund financial statements. Choice "4" is incorrect. The change in fund balance is not equal to the change in net position after consideration of the reconciling items described above.

A county's balances in the general fund included the following: Appropriations $435,000 Encumbrances 18,000 Expenditures 164,000 Vouchers payable 23,000 What is the remaining amount available for use by the county? A. $230,000 B. $248,000 C.$253,000 D. $271,000

Choice "3" is correct. Appropriations is a budgetary account. This account represents the governmental units approved spending (in this case $745,000). Expenditures represent the total amount incurred, whether paid in cash or accrued as vouchers payable, and serve to reduce available appropriations. Vouchers payable, alone, are excluded from this computation of available appropriations since the amount accrued is already included in expenditures. Encumbrances are either commitments or assignments of the fund balance for purchase orders and represent a reduction of available appropriations. Appropriations 745,000 Less: Expenditures 298,000 Less: Encumbrances 37,250 Remaining available appropriations 409,750 Choices "2", "1", and "4" are incorrect per explanation above.

Atomized Enterprises' Neutro Division qualified as a component held for sale during the year ended December 31, Year 2. The net book value of the division is $1,800,000 while its FMV is $1,500,000. The Division lost $240,000 during the year ended December 31, Year 1. The Division lost $150,000 in Year 2 prior to qualifying as being held for sale and $80,000 for the balance of that year. Ignoring income taxes, the results of Atomized Enterprises Discontinued operations displayed in their Year 2 comparative income statements for the year ended December 31, Year 1, would be equal to: a. $240,000 b. $390,000 c. $540,000 d. $0

Choice "1" is correct. The results of discontinued operations of a component are reported in discontinued operations (for the current period and for all prior periods presented) in the period the component is either disposed of or is held for sale. The results of subsequent operations of a component classified as held for sale are reported in discontinued operations in the period in which they occur. Atomized Enterprises would report the amount of the net loss reported Year 1 beneath the heading discontinued operations, $240,000.

Each of the following is a component of the changes in the net assets available for benefits of a defined benefit pension plan trust, except: a. The net change in the actuarial present value of accumulated plan benefits b. The net change in fair value of each significant class of investments c. Benefits paid to participants d. Contributions from the employer to participants

Choice "1" is correct. The statement of changes in net assets available for benefits shows the appreciation in the fair value of investments, any other investment income, investment expenses, contributions, benefits paid, and administrative expenses to arrive at the net increase or decrease in net assets available for benefits during the period. The change in actuarial present value of accumulated plan benefits is shown on the statement of changes in accumulated plan benefits. Choice "2" is incorrect. The net change in fair value is reported on the statement of changes in net assets available for benefits. Choice "3" is incorrect. Benefits paid are reported on the statement of changes in net assets available for benefits. Choice "4" is incorrect. Contributions to the plan, whether from the employer or participants, are reported on the statement of changes in net assets available for benefits.

The portion of special assessment debt maturing in 5 years, to be repaid from general resources of the government, should be reported in the: a. Government-wide statement of net position b. Custodial fund c. General fund d. Capital projects fund

Choice "1" is correct. When special assessment debt is to be repaid from general resources of the government, the debt should be recorded as general long-term liabilities in the governmental activities column of the government-wide statement of net position as is any debt that is to be repaid from general resources. Choice "2" is incorrect. The custodial fund reports special assessment debt that will be repaid by collections from third parties, not by general resources of the government. Choice "3" is incorrect. Since the measurement focus of governmental funds is the flow of financial resources, the general fund does not report long-term debt. Choice "4" is incorrect. The capital projects fund accounts for resources used in the acquisition and construction of major fixed assets.

The format for the presentation of the statement of financial position for a fiduciary fund adheres to the following equation: a. Assets and deferred inflow of resources minus liabilities and deferred outflows of resources equals fund balance b. Assets and deferred outflows of resources minus liabilities and deferred inflows of resources equals net position c. Assets and deferred outflows of resources equals liabilities and deferred inflows of resources plus fund balance d. Assets and deferred inflows of resources minus liabilities and deferred outflows of resources equals net position

Choice "2" correct. Fiduciary funds must present their statement of financial position in a net position format. The solution describes a net position format. Resources owned by the entity (assets and deferred outflows) minus resources owed by the entity (liabilities and deferred inflows) equals net position. Choice "1" incorrect. Fiduciary funds must present their statement of financial position in a net position format. The proposed solution attempts to describe a net position format, however, the inflows are inappropriately coupled with assets instead of liabilities and the outflows are inappropriately coupled with liabilities instead of assets. In addition, a net position format would not produce fund balance. Choice "3" is incorrect. Fiduciary funds must present their statement of financial position in a net position rather than a balance sheet format. A balance sheet format sets assets and deferred outflows of resources equal to liabilities and deferred inflows of resources plus fund balance. A net position format shows resources owned by the entity (assets and deferred outflows) minus resources owed by the entity (liabilities and deferred inflows) equal to net position. Choice "4" incorrect. Fiduciary funds must present their statement of financial position in a net position format. The proposed solution attempts to describe a net position format; however, the inflows are inappropriately coupled with assets instead of liabilities and the outflows are inappropriately coupled with liabilities instead of assets.

Quinn Co. reported a net deferred tax asset of $9,000 in its December 31, Year 1, balance sheet. For Year 2, Quinn reported pretax financial statement income of $300,000. Temporary differences of $100,000 resulted in taxable income of $200,000 for Year 2. At December 31, Year 2, Quinn had cumulative taxable differences of $70,000. Quinn's effective income tax rate is 30%. In its December 31, Year 2, income statement, what should Quinn report as deferred income tax expense? a. $12,000 b. $30,000 c. $60,000 d. $21,000

Choice "2" is correct, $30,000. Deferred tax expense is equal to the current period temporary differences times the enacted future tax rate: $100,000 × 30% = $30,000. Analysis of deferred tax account: 12/31/Year 1Change12/31/Year 2 Temporary Differences $30 (100) (70) Tax rate x 30% x 30% Deferred tax asset 9 (9) 0 Deferred tax liability 0 (21) (21) Net deferred tax 9 (30) (21)

Nolan owns 100 percent of the capital stock of both Twill Corporation and Webb Corporation. Twill purchases merchandise inventory from Webb at 140 percent of Webb's cost. During 20X0, Webb sold merchandise that had cost it $40,000 to Twill. Twill sold all of this merchandise to unrelated customers for $81,200 during 20X0. In preparing combined financial statements for 20X0, Nolan's bookkeeper disregarded the common ownership of Twill and Webb. What amount should be eliminated from cost of goods sold in the combined income statement for 20X0? a. $40,000 b. $56,000 c. $81,200 d. $16,000

Choice "2" is correct, $56,000. Merchandise that cost Webb $40,000 was sold to Twill at 140% of Webb's cost ($56,000) and must be eliminated.

How should a U.S. publicly traded company report a change in fair value of a hedged AFS security attributable to foreign exchange risk if the hedge is a fair value hedge? a. As a contra-asset related to the hedge b. In earnings c. As a change in the cost basis of the hedge d. In other comprehensive income

Choice "2" is correct. A fair value hedge is an instrument designed to hedge the exposure from changes in the fair value of a recognized asset or liability. Gains and losses on both the fair value hedge itself and the hedged item are recognized in earnings in the same accounting period. Choice "1" is incorrect. The appropriate accounting treatment is to record changes in the hedged item in earnings, not as a contra-asset to the hedge. Choice "3" is incorrect. Changes in value for the hedged security do not get recorded as changes to the cost basis of the hedge. Choice "4" is incorrect. Changes in the fair value of the security itself and the fair value hedge are recognized in earnings, not in other comprehensive income.

Which of the following statements is correct concerning a governmental entity's statement of cash flows? a. Cash flows from operating activities may use the indirect method b. Cash flows from capital financing activities are reported separately from cash flows from non-capital financing activities c. The statement format includes columns for the general, governmental, and proprietary fund types d. The statement format is the same as that of a business enterprise's statement of cash flows

Choice "2" is correct. Capital financing activities are reported separately from non-capital financing activities in a governmental entity's statement of cash flows. Capital financing activities include acquiring and disposing of capital assets and any related debt. Non-capital financing activities include borrowing money for other than capital expenditures, such as tax anticipation notes. Choice "1" is incorrect. Cash flows from operating activities must be reported using the direct method. Choice "3" is incorrect. The cash flow statement of a governmental entity is only prepared for proprietary funds. Choice "4" is incorrect. A governmental entity's combined statement of cash flows is not of the same format as that of a business enterprise. It has four classifications: operating, non-capital financing, capital and related financing, and investing activities. Reconciliations from the Statement of Revenues, Expenses and Changes in Net Position to the cash flow from operations on the Statement of Cash Flows is based on operating income, not the bottom line (the change in net position).

In Year 1, a state government collected income taxes of $8,000,000 for the benefit of one of its cities that imposes an income tax on its residents. The state remitted these collections periodically to the city. The state should account for the $8,000,000 in the: a. General fund b. Custodial funds c. Internal service funds d. Special assessment funds

Choice "2" is correct. Custodial funds. The $8,000,000 collected by a state for the benefit of one of its cities that imposes an income tax on its residents should be accounted for by the state in a custodial fund. The initial recording in the custodial fund should be: Debit (Dr)Credit (Cr)Cash 8,000,000 Additions: Tax collections for other governments 8,000,000 To record addition for tax collections for other governments Note: Subsequent disbursement of collected taxes would be recorded as "Deduction: payment of tax collections to other governments."

Opto Co. is a publicly traded, consolidated enterprise reporting segment information. Which of the following items is a required enterprise-wide disclosure regarding external customers? a. Information on major customers is not required in segment reporting b. The fact that transactions with a particular external customer constitute more than 10% of the total enterprise revenues c. The identity of any external customer providing 10% of more of a particular operating segment's revenue d. The identity if any external customer considered to be "major" by management

Choice "2" is correct. In order to conform to GAAP, financial statements for public business enterprises must report segment information about a company's major customers if that customer provides 10% or more of the combined revenue, internal and external, of all operating segments. Choice "1" is incorrect. Disclosure is required. Choice "3" is incorrect. Revenue is 10% of ALL operating segments not "a particular" segment. Choice "4" is incorrect. Disclosure is not at management's discretion.

Riche Township recorded more estimated revenues than appropriations for the coming fiscal year. In integrating its adopted budget with its financial accounting records, the town would: a. Debit budgetary control b. Credit budgetary control c. Debit encumbrance control d. Credit encumbrance control

Choice "2" is correct. Integration of budgetary controls into the accounting records is accomplished with the following journal entry with the estimated revenues and approved expenditures posted on the opposite side of "T" accounts compared to actual amounts and any balancing amount posted to budgetary control: Debit (Dr)Credit (Cr)Estimated revenue control xxx Budgetary control (if deficit) xxx Appropriations control xxx Budgetary control (if surplus) xxx Choice "1" is incorrect. A surplus budget would be recorded with a balancing credit entry to budgetary control. Choice "3" is incorrect. Encumbrance accounts are used internally to account for the issuance of purchase orders and are debited when the purchase order is issued and credited when the liability is incurred. Recording the budget does not impact encumbrance accounts. Choice "4" is incorrect. Encumbrance accounts are used internally to account for the issuance of purchase orders and are debited when the purchase order is issued and credited when the liability is incurred. Recording the budget does not impact encumbrance accounts.

For its first year of operations, Cable Corp. recorded a $100,000 expense in its tax return that will not be recorded in its accounting records until next year. There were no other differences between its taxable and financial statement income. Cable's effective tax rate for the current year is 21%, but a 25% rate has already been passed into law for next year. In its year-end balance sheet, what amount should Cable report as a deferred tax asset (liability)? a. $21,000 asset b. $25,000 liability c. $21,000 liability d. $25,000 asset

Choice "2" is correct. Pretax income was $100,000 more than taxable income for the current period, resulting from the timing difference which is to be settled next year. Next year's enacted tax rate will be 25%. Cable must recognize a $25,000 ($100,000 × 25%) tax liability in the current period which will be paid (settled) next year. Choice "1" is incorrect. Taxes were not paid in advance. Choice "3" is incorrect. Liability will be settled when tax rate is 25% not current 21%. Choice "4" is incorrect. Taxes were not paid in advance.

Which format must an enterprise fund use to report cash flow operating activities in the statement of cash flows? a. Indirect method, beginning with change in net assets b. Direct method c. Either direct or indirect method d. Indirect method, beginning with operating income

Choice "2" is correct. Proprietary funds (internal Service and Enterprise funds) must use the direct method when preparing their statement of cash flows. Choice "1" is incorrect. Proprietary funds (internal Service and Enterprise funds) must use the direct method when preparing their statement of cash flows. The indirect method is not an option. The direct method is accompanied by a reconciliation of accrual basis operating income to cash flows from operations. Unlike commercial accounting, there is no reconciliation of the bottom line (the change in net assets) to cash flows from operations. Choice "3" is incorrect. Proprietary funds (internal Service and Enterprise funds) must use the direct method when preparing their statement of cash flows. There is no other option. Choice "4" is incorrect. Proprietary funds (internal Service and Enterprise funds) must use the direct method when preparing their statement of cash flows. The indirect method is not an option. The direct method is accompanied by a reconciliation of accrual basis operating income to cash flows from operations.

The following information pertains to certain monies held by Blair County at December 31, Year 1, that are legally limited to expenditures for specified purposes: Proceeds of short-term notes to be used for advances to private purpose trust funds $8,000 Proceeds of long-term debt to be used for a major capital project 90,000 What amount of these monies should Blair account for as restricted or committed in special revenue funds? a. $90,000 b. $0 c. $8,000 d. $98,000

Choice "2" is correct. Special revenue funds account for the proceeds of specific revenue sources (other than debt service or for major capital projects) that are legally restricted or committed to expenditures for specific purposes. The $8,000 is to be used for advances to private purpose trust funds and would be accounted for in private purpose trust funds. The $90,000 is for a major capital project, which would be accounted for in a capital projects fund. Choice "1" is incorrect. The $90,000 is accounted for in the capital projects fund. Choice "3" is incorrect. The $8,000 is for the private purpose trust funds, which would account for this money. Choice "4" is incorrect. The $8,000 is accounted for in the private purpose trust funds. The $90,000 is accounted for in the capital projects fund.

A budgetary control (open purchase orders) account balance in excess of a balance of encumbrances indicates: a. An excess of vouchers payable over encumbrances b. A recording error c. An excess of appropriations over encumbrances d. An excess of purchase orders over invoices received

Choice "2" is correct. The budgetary control account (open purchase orders) represents a portion of the budgetary control balance that has been segregated for expenditure on vendor performance, usually being goods ordered but not yet received. Remember the entry to record a purchase order: Debit (Dr)Credit (Cr)Encumbrances XXX Budgetary control XXX Likewise, the encumbrance balance represents purchase orders outstanding. For the budgetary control (open purchase orders) to exceed the encumbrance balance, a recording error must have been made. Choice "1" is incorrect. Encumbrances represent purchase orders for which the goods ordered are not yet received. Vouchers payable represent the amounts owed and not yet paid for goods received. The relative balances of these accounts have no impact on the balance in the budgetary control account. Choice "3" is incorrect. Appropriations are recorded when the budget is adopted and are unrelated to encumbrances. Choice "4" is incorrect. The budgetary control account is a budgetary account indicating the outstanding purchase orders for goods not yet received. Likewise, the encumbrance balance represents purchase orders outstanding. An excess of purchase orders over invoices received would not explain the difference between these two accounts.

On July 1, Year 1, Black & Associates issued 2,000 of its 8%, $1,000 bonds for $1,752,000. The bonds were issued to yield 10%. The bonds are dated July 1, Year 1 and mature on July 1, Year 11. Interest is payable semiannually on January 1st and July 1st. Using the effective interest method, how much of the bond discount should be amortized for the six months ended December 31, Year 1? a. $12,400 b. $7,600 c. $9,920 d. $15,200

Choice "2" is correct. The computations are: Effective interest = $1,752,000 × .10 = 175,200 − Cash interest = 2,000,000 × .08 = 160,000 15,200 ÷ 2 = Disc. Amortization for the 6 months ended Dec. 31, Year 1 7,600 Choices "4", "3", and "1" are incorrect, per the above calculations.

Pinellas Company owns 30% of the voting common stock of Sanibel Company. Pinellas will probably use the equity method of accounting to account for this investment because: a. Pinellas will be able to appoint 30% of the directors of Sanibel Co. b. Pinellas is assumed to be able to exercise significant influence over the affairs of Sanibel Co c. No other shareholder holds more than a 29% interest in Sanibel Co. d. Pinellas receives 30% of dividends paid by Sanibel Co.

Choice "2" is correct. The equity method of accounting should be used when the investor is able to exercise significant influence over the affairs of the investee. Significant influence is generally assumed if the investor owns 20% or more of the voting stock of the investee company. Choice "1" is incorrect. The number of directors that the investor can appoint is not the deciding factor. Choice "3" is incorrect. Other owners' percentage interest is irrelevant. Choice "4" is incorrect. Although this is a true statement, it does not explain the use of the equity method.

A company issues $1,500,000 of par bonds at 98 on January 1, Year 1, with a maturity date of December 31, Year 30. Bond issuance costs are $90,000, and the stated interest rate of the bonds is 6 percent. Interest is paid semiannually on January 1 and July 1. Ten years after the issue date, the entire issue was called at 102 and canceled. The company uses a straight-line method of amortization for bond discounts and issuance costs, and the result of this method is not materially different from the effective interest method. The company should classify what amount as the loss on extinguishment of debt at the time the bonds are called? a. $30,000 b. $110,000 c. $90,000 d. $50,000

Choice "2" is correct. The gain/loss on extinguishment is equal to the difference between the reacquisition price and the net carrying amount. Reacquisition price: $1,500,000 × 102% = $1,530,000 Carrying value: Face 1,500,000 Less: Unamortized discount (20,000) Less: Unamortized bond issuance cost (60,000) Net carrying value 1,420,000 Total loss on extinguishment: $1,530,000 - $1,420,000 = $110,000 Note: The original discount is $30,000 [$1,500,000 - ($1,500,000 × 98%)]. The bond was retired 10 years into a 30-year issue, so two thirds (or $20,000) of the original discount has not been amortized yet. The original bond issuance cost of $90,000 is amortized over the full 30 years. Through 10 years, $30,000 (or one third) has been amortized and $60,000 (or two thirds) remains unamortized. Choice "1" is incorrect. This choice fails to take the unamortized discount or bond issuance costs into account. Choice "3" is incorrect. This choice fails to take the unamortized discount into account. Choice "4" is incorrect. This choice fails to take the unamortized bond issuance costs into account.

The statement of activities in the government-wide financial statements includes which of the following? a. Separate columns for each major governmental fund b. Separate columns for governmental activities, business-type activities, and discretely presented component units c. Separate columns for each nonmajor governmental unit d. A separate section at the bottom of the statement that shows program and general revenues

Choice "2" is correct. The statement of activities in the government-wide financial statements includes separate columns for governmental activities, business-type activities, and discretely presented component units. Choice "1" is incorrect. Major funds are reported in the fund financial statements. Choice "3" is incorrect. Nonmajor funds are reported in the fund financial statements Choice "4" is incorrect. The statement of activities in the government-wide financial statements includes a separate section at the bottom of the statement that shows general revenue. Program revenues are shown in relation to functional expenses within the body of the statement.

On November 15, Quazar Co. declared a property dividend of marketable securities to be distributed on December 15 to stockholders on record on December 1. The market value of the securities was as follows: November 15 $225,000 December 1 $220,000 December 15 $250,000 The marketable securities originally cost Quazar $200,000. What is the net effect on Quasar's retained earnings as a result of declaring a property dividend? a. $250,000 b. $200,000 c. $195,000 d. $225,000

Choice "2" is correct. Two factors will affect retained earnings as a result of this transaction. Quazar will recognize a gain on disposition of the marketable securities as well as a dividend: Gain on marketable securities 25,000 Property dividend (225,000) Net effect on retained earnings (200,000) The property dividend is valued on the declaration date. Choice "1" is incorrect. $250,000 is calculated as if the property dividend was the $250,000 at the distribution date and the gain on the marketable securities was ignored. The property dividend is valued at the declaration date, and any gain on the marketable securities must be taken into account. Choice "3" is incorrect. $195,000 is calculated as if the property dividend was the $220,000 at the record date and then it was reduced by the gain on the marketable securities. The property dividend is valued at the declaration date. Choice "4" is incorrect. $225,000 is the amount of the dividend only. The gain on the marketable securities should have been included also.

The original cost of an inventory item is below the net realizable value and above the net realizable value less a normal profit margin. The inventory item's replacement cost is below the net realizable value less a normal profit margin. Under the lower of cost or market method, the inventory method item should be valued at: a. Original cost b. Net realizable value c. Net realizable value less normal profit margin d. Replacement cost

Choice "3" is correct, net realizable value less normal profit margin. Approach: List costs in descending sequence from highest to lowest and assign arbitrary dollar values in descending sequence. Then use "rule of thumb." Lower of cost or market (take lower of two) Rule of thumb: Market (take middle of three) Net realizable value (ceiling) $.90 Net realizable value less Normal profit margin (floor) .80 .80 Replacement cost .70 Original cost .85 Lower of cost or market (take lower of two) Which is net realizable value less normal profit margin $.80

On March 1, Year 1, Fine Co. borrowed $10,000 and signed a two-year note bearing interest at 12% per annum compounded annually. Interest is payable in full at maturity on February 28, Year 3. What amount should Fine report as a liability for accrued interest at December 31, Year 2? a. $1,200 b. $0 c. $2,320 d. $1,000

Choice "3" is correct. 12%, 2-yr note payable 10,000 Year 1 interest expense [12% x $10,000 x 10/12] 1,000 Year 1 net liability 11,000 Year 2 interest expense = [12% x $11,000] 1,320 Accrued interest payable at 12/31/Year 2 equals $2,320 [$1,000 + $1,320] Choice "1" is incorrect. This is simple interest on the original note [12% x $10,000] for one year and is not the accrued interest payable balance. Choice "2" is incorrect. Accrued interest must be recognized and recorded during the term of the note. Choice "4" is incorrect. Year 1 accrued interest equals $1,000 [$10,000 x 12% x 10/12].

Which of the following examples would require restatement of prior year's financial statements? a. An insurance premium that was due in the prior year but that lapsed because the policy was not paid b. A calculation change of warranty obligations based on updated claim information for the prior year c. A change in from the income tax basis of accounting to the accrual basis d. An intangible asset with a remaining estimated amortization period of two years, which is determined to be obsolete

Choice "3" is correct. A change from the income tax basis of accounting (Non-GAAP) to the accrual basis (GAAP) is an error correction, which will require a prior-period adjustment (restatement) of prior years' financial statements. Choice "1" is incorrect. An insurance policy that lapsed because the premium payment wasn't made is handled prospectively (going forward) and does not require restatement. Choice "2" is incorrect. A calculation change of warranty obligations represents a change in accounting estimate, which is handled prospectively (going forward) and does not require restatement. Choice "4" is incorrect. The write-down of obsolete inventory is a change in accounting estimate, which is handled prospectively (going forward) and does not require restatement.

Which of the following statements is correct regarding reporting comprehensive income? a. Comprehensive income must include all changes in stockholder's equity for the period b. Comprehensive income is reported in the year-end statements but not in the interim statements c. Accumulated other comprehensive income is reported in the stockholder's equity section of the balance sheet d. A separate section of comprehensive income is required

Choice "3" is correct. Accumulated other comprehensive income is a component of stockholders' equity on the balance sheet. Choice "1" is incorrect. Comprehensive includes only nonowner changes in equity. Stock transactions and dividends are not included in comprehensive income. Choice "2" is incorrect. Comprehensive income is reported in interim financial statements and year-end financial statements. Choice "4" is incorrect. Comprehensive income can be reported in a separate statement of comprehensive income or in a statement of income and comprehensive income.

Amar Farms produced 300,000 pounds of cotton during the Year 1 season. Amar sells all of its cotton to Brye Co., which has agreed to purchase Amar's entire production at the prevailing market price. Recent legislation assures that the market price will not fall below$. 70 per pound during the next two years. Amar's costs of selling and distributing the cotton are immaterial and can be reasonably estimated. Amar reports its inventory at expected exit value. During Year 1, Amar sold and delivered to Brye 200,000 pounds at the market price of $.70. Amar sold the remaining 100,000 pounds during Year 2 at the market price of $.72. What amount of revenue should Amar recognize in Year 1? a. $140,000 b. $144,000 c. $210,000 d. $216,000

Choice "3" is correct. Agricultural products (and precious metals) may be stated at above cost by using net selling price less cost of disposal because: There is a ready market for such items (i.e., $.70 price legislation). There is unit-interchangeability (i.e., cotton is cotton). Pounds produced 300,000 Market price .70 /pound Revenue recognized in Year 1 210,000 In essence, revenue is recognized at time of production and not at the time of sale.

Which of the following qualifies as a reportable segment? a. Eastern Europe segment, which reports its results directly to the manager of the European division, and has 20% of the company's assets, 12% of revenues, and 11% of profits b. South American segment, whose results of operations are reported directly to the Chief operating officer, and has 5% of the company's assets, 9% of revenues, and 8% of the profits c. North American segment, whose assets are 12% of the company's assets of all segments, and management reports to the chief operating officer d. Corporate headquarters, which oversees $1 billion in sales for the entire company

Choice "3" is correct. Assets of the North American segment exceed 10% combined assets of all operating segments. Choice "1" is incorrect. Eastern Europe segment does not report to the chief operating officer. Choice "2" is incorrect. The South American segment does not meet any of the 10% minimums (Revenue, P&L or Assets). Choice "4" is incorrect. Corporate headquarters is not considered a segment.

How should a nongovernmental not-for-profit organization report donor-restricted cash contributions for long-term purposes in its statement of cash flows? a. As a noncash transaction b. Operating activity inflow c. Financing activity inflow d. Investing activity inflow

Choice "3" is correct. Cash contributions restricted by the donor for long-term purposes must be reported as a cash inflow in the financing activities section of the statement of cash flows, segregated from other financing activities. Choice "1" is incorrect. Cash contributions restricted by the donor for long-term purposes must be reported as cash inflows in financing actitivies. It would be inappropriate to classify a cash contribution as a non-cash transaction. Choice "2" is incorrect. Cash flows from operations would include cash flows from contributions, program income and agency transactions. Cash flows from operations might also include receipts designated by the entity's governing body for use for long-lived assets. Donor restricted receipts to be used for long-lived assets, however, are classfied as financing actiivities. Choice "4" is incorrect. Investing activities typically result from the purchase of long-lived assets or from sales of assets (e.g. equity securities) restricted for the acquisition of long-lived asset. Donor restricted cash receipts to be used for long-lived assets, however, are classified as financing activities.

What is the appropriate characterization of the net assets of a non-governmental not-for-profit organization? a. Ownership interest b. Equity interest c. Residual interest d. Donor's interest

Choice "3" is correct. Net assets of a nongovernmental not-for-profit organization are most appropriately characterized as the residual interest of the not-for-profit entity. The ownership interests of not-for-profit organizations are unlike business enterprises. Resources do not belong to a defined class of owners, only the not-for-profit entity itself and the mission for which it was formed. The net assets represent the difference between the not-for-profit organization's assets and liabilities. Choice "1" is incorrect. Ownership interest is not an appropriate characterization of the net assets of a nongovernmental not-for-profit organization. Ownership interests do not exist as they would in commercial settings. Choice "2" is incorrect. Equity interest is not an appropriate characterization of the net assets of a nongovernmental not-for-profit organization. Equity interests do not exist as they would in commercial settings. Choice "4" is incorrect. Donor's interest is not an appropriate characterization of the net assets of a nongovernmental not-for-profit organization. Donors do not have a direct interest in the resources or net assets of a not-for-profit organization.

Emma construction Company started building a new administrative headquarters on January 1, Year 1. Emma intends to occupy the building at the project completion date of January 1, Year 3. At December 31, Year 1, Emma had incurred $2,000,000 of construction costs, evenly spread during that first year. Projected remaining costs are $2,500,000. During Year 1, Emma incurred interest cost on specific construction debt in the amount of $40,000 and interest on other unrelated loans in the amount of $30,000. All loans carry 5% interest. How much interest should Emma capitalize for Year 1? a. $70,000 b. $40,000 c. $50,000 d. $0

Choice "3" is correct. The computations are: Total Expenditures Average Accumulated Expenditures $2,000,000 ÷ 2 = $1,000,000 × .05 = $50,000 avoidable interest Compare "avoidable interest" (the potential amount to be capitalized) to total actual interest cost incurred and capitalize the lower amount. $50,000 < $70,000, therefore, capitalize $50,000. Choice "1" is incorrect. Average accumulated expenditures must be used not total expenditures. Since the expenditures were incurred uniformly throughout the year (the usual assumption), the average accumulated expenditures calculation is: $2,000,000 ÷ 2 = $1,000,000 Choice "2" is incorrect. Avoidable interest is compared, not to interest on specific borrowings ($40,000), but to total interest cost incurred for any purpose ($70,000). Choice "4" is incorrect, per the above discussion.

For Year 1, Pac Co. estimated its two-year equipment warranty costs based on $100 per unit sold in Year 1. Experience during Year 2 indicated that the estimate should have been based on $110 per unit. The effect of this $10 difference from the estimate is reported: a. As a correction of an error requiring Year 1 financial statements to be restated b. As an accounting change requiring Year 1 financial statements to be restated c. In Year 2, as income from continuing operations d. As an accounting change, net of tax, below Year 2 from continuing operations

Choice "3" is correct. The effect of the new estimate of warranty costs (from $100 to $110) is a change in estimate and will be reported in Year 2 income from continuing operations. Rule: Changes in estimates affect only the current and subsequent periods (not prior periods and not retained earnings). Choice "1" is incorrect. The facts stating a new estimate of warranty costs indicate a change of estimate, not a correction of an error. Choice "2" is incorrect. Restating prior years financial statements is only required when comparative financial statements are shown for prior period adjustments of subsequently discovered corrections of errors, changes in entity or changes in accounting principle. Choice "4" is incorrect. An accounting change of principle is shown net of tax on the retained earnings statement.

Jordan Township's Water & Sewer fund received interest earnings off its invested funds. What category would Jordan use to classify cash receipts from these earnings on its Water & Sewer enterprise fund statement of cash flows? a. Operating activities b. Noncapital financing activities c. Capital and related financing activities d. Investing activities

D Investing activities include making and collecting loans and acquiring and disposing of debt or equity instruments including interest and dividend income

Under the modified accrual basis of accounting for a governmental unit, earned revenues that are measurable should be recognized in the accounting period in which they are: a. Collected b. Budgeted c. Available d. Appropriated

Choice "3" is correct. Under the modified accrual basis of accounting for a governmental unit, earned revenues should be recognized in the accounting period in which they are measurable and available. Choice "1" is incorrect. Collection or cash receipts is not the criteria for earning revenue in government (any more than it is in commercial accounting). Revenue recognition in governmental funds is more strict than commercial and requires that it be both measurable and available in order to be recognized. Choice "2" is incorrect. The budget represents planned earnings. Revenue cannot be arbitrarily assigned to periods based on their planned earnings. Revenues are recognized when measurable and available. Choice "4" is incorrect. Appropriations represents the period in which the funds are intended to be used. Although the benefitting period is significant, the primary recognition issues for the modified accrual basis of accounting are measurability and availability.

Which of the following statements describes the proper accounting for losses when nonmonetary assets are exchanged for other nonmonetary assets? a. A loss can occur only when assets are sold or disposed of in a monetary transaction b. A loss is deferred so that the asset received in the exchange is properly valued c. A loss is recognized immediately, because assets received should not be valued at more than their cash equivalent price d. A loss, if any, which is unrelated to the determination of the amount of the asset received should be recorded

Choice "3" is correct. Under the rule of conservatism, losses are recognized in all nonmonetary exchanges when the book value exceeds the fair value of the asset given up. An asset's cash equivalent price is the asset's fair value. Assets should not be valued at more than fair value, so when book value exceeds fair value, the asset should be recorded at the lower fair value. When a loss is recorded, the asset received is recorded at the book value of the asset given up plus any cash paid minus any cash received minus the loss recognized. Choice "1" is incorrect. Losses are always recognized in nonmonetary transactions. Choice "2" is incorrect. Losses are never deferred in nonmonetary exchanges. Choice "4" is incorrect. In nonmonetary exchanges, losses always impact the recorded amount of the asset received. If a loss is recorded, the asset received is recorded at the book value of the asset given up plus any cash paid minus any cash received minus the loss recognized.

The following information pertains to Ceil Co., a company whose common stock trades in a public market: Shares outstanding at 1/1 100,000 Stock dividend at 3/31 24,000 Stock issuance at 6/30 5,000 What is the weighted-average number of shares Ceil should use to calculate its basic EPS for the year ended December 31? a. 120,500 b. 123,000 c. 126,500 d. 129,000

Choice "3" is correct. When calculating the weighted - average number of shares to be used in the earnings-per-share calculation, stock dividends are treated as if they occurred at the beginning of the period. Ceil Co.'s weighted average number of shares should therefore be calculated as: 124,000 shares x 6/12 = 62,000 129,000 shares x 6/12 = 64,500 Weighted average 126,500 Choice "1" is incorrect. This weighted average calculation does not treat the stock dividend retroactively. Stock dividends must be treated as if they occurred at the beginning of the period. Choice "2" is incorrect. This weighted average calculation treats the stock issuance as if it occurred at the beginning of the year and does not make a retroactive adjustment for the stock dividend. Stock issuances are not adjusted retroactively, but instead are time weighted for the period of the year in which the shares are outstanding. Stock dividends and stock splits must be treated retroactively. Choice "4" is incorrect. This is the total number of shares outstanding at December 31, not the weighted average number of shares outstanding for the year.

Park, Inc., acquired 100% of Gravel Co.'s net assets. On the acquisition date, Gravel's accounting records reflected $50,000 of costs associated with in-process research and development activities. The fair value of the in-process research and development activities was $400,000. Park's consolidated intangible assets will increase by what amount, if any, as a result of the acquisition of the in-process research and development activities? a. $50,000 b. $350,000 c. $400,000 d. $0

Choice "3" is correct. When one entity purchases another entity, the transaction is considered a business combination. To account properly for a business combination, all assets acquired and liabilities assumed will be measured at fair value. Any amount paid in excess of the identifiable fair value of net assets is recorded to goodwill. Though the cost of the in-process research and development to Gravel Co. was $50,000, its fair value was $400,000 and will be recorded as an asset on Park, Inc.'s books at that amount. Choice "1" is incorrect. $50,000 is the cost related to in-process research and development. As the fair value is known, the acquirer must assign the fair value to the intangible purchased. Choice "2" is incorrect. $350,000 reflects the difference between the cost and the fair value. As Park, Inc. does not currently have the cost reflected on its books until acquisition, it must record the entire $400,000 fair value amount to intangibles. Choice "4" is incorrect. An asset is being purchased and must therefore be reflected with a value on the acquirer's books.

Baker, Inc. reported the following stockholder's equity balances: 8% cumulative preferred stock, par value $100 per share; 10,000 shares issued and outstanding (liquidation value $107) $1mil Common stock, par value $10 per share, 50,000 shares issued and outstanding $500,000 APIC 75k Retained earnings $450,000 Dividends are in arrears on the preferred stock for three years including the current year. What is book value per share of common stock? a. $20.50 b. $19.10 c. $15.70 d. $14.30

Choice "4" is correct. 8% cumulative preferred stock 1,000,000 Common stock 500,000 Additional paid in capital 75,000 Retained earnings 450,000 2,025,000 Less: preferred stock: liquidation value (10,000 × $107) 1,070,000 Dividends in arrears (10,000 × $8 × 3 years) 240,000 (1,310,000) Total value of common stock 715,000 Divided by common shares outstanding ÷ 50,000 Book value per share of common stock 14.30 Choice "1" is incorrect. $20.50 is calculated by ignoring the $107 liquidation value of the preferred stock and the dividends in arrears of $240,000. The total value for the common stock would have been $310,000 larger, or $6.20 on a per share basis. Choice "2" is incorrect. $19.10 is calculated by ignoring the $240,000 dividends in arrears. The total value for the common stock would have been $240,000 larger, or $4.80 on a per share basis. Choice "3" is incorrect. $15.70 is calculated by ignoring the $107 liquidation value of the preferred stock and by using the $100 par value instead. The total value for the common stock would have been $70,000 larger, or $1.40 on a per share basis.

Wizard Co. purchased two machines for $250,000 each on January 2. The machines were put into use immediately. Machine A has a useful life of 5 years and can be used in only one research project. Machine B will be used for 2 years on a research and development project and then used by the production division for an additional 8 years. Wizard uses the straight-line method of depreciation. What amount should Wizard include in research and development expense for the year? a. $50,000 b. $375,000 c. $500,000 d. $275,000

Choice "4" is correct. $275,000 is computed as follows: Machine A (no alternate use) 250,000 Machine B (alternate use)($250,000 ÷ 10 years) 25,000 Total 275,000 Choice "1" is incorrect. $50,000 is calculated by including the depreciation of both Machine A and Machine B in research and development expense. Since Machine A had no alternative future uses, its full cost should have been included in research and development expense in the period of acquisition. Choice "2" is incorrect. $375,000 is the full cost of Machine A and ½ the cost of Machine B. It ignores the use of Machine B by the production division for the additional eight years. Choice "3" is incorrect. $500,000 is the full cost of both of the machines. Since Machine B had alternative future uses, only its depreciation should have been included in research and development expense.

Lizzy Co. traded an old machine to Chang Co. for a similar new machine. The exchange is assumed to lack commercial substance. Lizzy also received $10,000 in cash. The following information relates to the machines on the date of the exchange: CV FV Old Machine $70,000 $100,000 New Machine $45,000 $90,000 What amount of gain should Lizzy record from this exchange under U.S. GAAP? a. $10,000 b. $0 c. $30,000 d. $3,000

Choice "4" is correct. $3,000. The question specifically states that the nonmonetary exchange lacks commercial substance. Under U.S. GAAP, when there is no boot, no gain is recognized in nonmonetary exchanges that lack commercial substance. However, in this question, boot/cash is received so a proportional part of the gain is recognized. If we looked into this question more closely, we would find that we could have also determined that the exchange lacked commercial substance, even if that information had not been provided in the question. First, the assets are similar productive assets (i.e., both assets are machines), which is often an indication that the risk, timing, or amount of the expected future cash flows from the assets received will not differ significantly from the risk, timing, or amount of expected future cash flows from the assets transferred, especially if the cash received in the same transaction is insignificant. In this question, the amount of cash that changes hands is insignificant (10% of the fair value of the asset relinquished and 10% of the total fair value of the assets received). Based on these facts, a conclusion that the exchange lacks commercial substance appears to be warranted. To determine the gain to be recognized, a gain of $30,000 ($100,000 fair value − $70,000 carrying value) is realized on the exchange. The total consideration received is $100,000 ($90,000 + $10,000). The cash received is $10,000, which is less than 25% of the total consideration received. A gain of $3,000 (1/10th) is recognized. The journal entry is as follows: Debit (Dr)Credit (Cr)Cash 10,000 New machine 63,000 Gain on exchange 3,000 Old machine (net) 70,000 Note that the question did not really ask for the initial carrying value (cost) of the new machine, but it certainly could have. The initial carrying value of the new machine could also be calculated as the fair value of the new machine less the realized gain not recognized ($90,000 − $27,000 = $63,000). It is not a bad idea to calculate the initial carrying value using both methods as a quick check. On these transactions, it is really easy to get the numbers mixed up.

On December 29, Action Corp. signed a seven year lease for an airplane to transport its sports team around the country. The airplane's fair value was $841,500. Action made the first annual lease payment of $153,000 on December 31. Action's incremental borrowing rate was 12 percent, and the interest rate implicit in the lease, which was known to Action, was 9 percent. The following are the rounded present value factors for an annuity due: 9% for 7 years 5.5 12% for 7 years 5.1 What amount should Action report as a lease liability in its December 31 balance sheet? a. $780,300 b. $627,300 c. $841,500 d. $688,500

Choice "4" is correct. $688,500. Rule: The present value rate used to value a finance lease is the rate implicit in the lease if known by the lessee. Annual lease payments 153,000 PV of annuity due of 1 for 7 years at 9% x 5.5 Lease liability before 12/31 payment (FMV) 841,500 Less 12/31 payment (153,000) Equals lease liability in 12/31 BS 688,500

When the allowance method for recognizing uncollectible accounts is used, the entries is used, the entries at the time of collection of a small account previously written off would: a. Increase net income b. Have no effect on the allowance for uncollectible accounts c. Decrease the allowance for uncollectible accounts d. Increase the allowance for uncollectible accounts

Choice "4" is correct. A collection of a previously written-off account receivable would increase the "allowance" account, which is a credit balance account. Debit (Dr)Credit (Cr)Cash 1,000 Allowance for doubtful accounts 1,000

On December 31, Year 1, an entity awaiting judgement on a lawsuit determined that a loss from the suit ranging between $1,000,000 and $2,000,000 was reasonably possible. On March 15, Year 2, after the entity issued its financial statements, the suit was settled. The settlement required the entity to pay damages of $1,400,000. What amount of contingent liability should the entity have reported on its December 31, Year 1 balance sheet? a. $2,000,000 b. $1,400,000 c. $1,000,000 d. $0

Choice "4" is correct. A contingent liability is not accrued for reasonably possible loss contingencies. If the suit had been settled before the financial statements were issued, then a liability would have been recorded on the December 31, Year 1 balance sheet.

The billings for transportation services provided to other governmental units are recorded by the internal service fund as: a. Transportation appropriations b. Intergovernmental transfers c. Interfund exchanges d. Operating revenues

Choice "4" is correct. Billings for transportation services provided to other governmental units are considered quasi-external transactions and are treated as revenues by the internal service fund. The journal entry is as follows: Debit (Dr)Credit (Cr)Due from other funds XXX Operating revenues XXX Choices "1", "3", and "2" are incorrect, per the journal entry explanation above.

On March 2, Year 1, Finch City issued 10-year obligations at face amount, with interest payable March 1 and September 1. The proceeds were to be used to finance the construction of a civic center over the period April 1, Year 1, to March 31, Year 2. During the fiscal year ended June 30, Year 1, no resources had been provided to the debt service fund for the payment of principal and interest On June 30, Year 1, Finch's financial statements should report the construction in progress for the civic center in the: Capital Projects Fund Government wide financial statements a. Y Y b. Y N c. N N d. N Y

Choice "4" is correct. No, Yes. At the end of the fiscal year on June 30, Year 1. Finch's government-wide state of net position should report the construction in progress for the civic center in the governmental activities column. It would not be in the capital projects fund, since each year's capital project activities are closed out at year end. (Prior to year end, construction in process expenditures of the current year would be reported in the capital projects fund.)

Cabot Township's Water and Sewer Fund has $40,000 in operating income and a total change in net position of 150,000. The Water and Sewer Fund also had the following transactions either derived from a comparison of current and prior year Statements of Net Position or displayed on the current year Statement of Activities: Increase in current assets other than cash $30,000 Increase in current liabilities including the $15,000 current portion of revolving debt $50,000 Depreciation expense $70,000 Gain on disposal of fixed assets $2,000 Cash flows from operating activities would reconcile to what amount? a. $130,000 b. $132,000 c. $242,000 d. $115,000

Choice "4" is correct. Cash flows from operating activities would reconcile as follows: Operating Income 40,000 Less: Increases in current assets other than cash (30,000) Plus: Increases in current liabilities (net of change in revolving debt) 35,000 Plus: Depreciation 70,000 Total 115,000 Operating income is used to reconcile to cash flows from operations. Non operating income items (such as revolving debt financing and gains on the sale of assets) are excluded from the reconciliation. Choice "1" is incorrect. Operating Income is used to reconcile to cash flows from operations. Non operating income items (such as revolving debt financing) are excluded from the reconciliation. Choice "2" is incorrect. Operating income is used to reconcile to cash flows from operations. Non operating income items (such as revolving debt financing and gains on the sale of assets) are excluded from the reconciliation. Choice "3" is incorrect. Operating income, not the total change in net position is used to reconcile to cash flows from operations. Non operating income items (such as revolving debt financing and gains on the sale of assets) are excluded from the reconciliation.

The stockholders of Meadow Corp. approved a stock-option plan that grants the company's top three executives options to purchase a maximum of 1,000 shares each of Meadow's $2 par common stock for $19 per share. The options were granted on January 1 when the fair value of the stock was $20 per share. Meadow determined that the fair value of the compensation is $300,000 and the vesting period is three years. What amount of compensation expense from the options should Meadow record in the year the options were granted? a. $300,000 b. $60,000 c. $20,000 d. $100,000

Choice "4" is correct. Compensation expense is calculated at the grant date of the option and allocated over the vesting period: $300,000 / 3 years = $100,000 per year. Choice "1" is incorrect. Compensation expense is not recorded in full in the year the options are issued when the vesting period is three years. Compensation expense is recognized equally over the vesting period. Choice "2" is incorrect. Compensation expense is not recorded at the total par value of the stock. Choice "3" is incorrect. Compensation expense is not recorded at the total par value of the stock divided by the vesting period.

What is the basic criterion used to determine the reporting entity for a governmental unit? a. Special financial arrangement b. Geographic boundaries c. Scope of public services d. Financial accountability

Choice "4" is correct. Financial accountability. An accountability perspective provides the basis for defining the financial reporting entity. Choices "1", "2", and "3" are incorrect. Special financing arrangements, geographic boundaries and scope of public services are not the basic criterion used to determine the reporting entity for a governmental unit.

The measurement focus on government fund accounting is on which of the following? a. Working capital b. Cash c. Economic resources d. Current financial resources

Choice "4" is correct. Governmental funds (GRSPP) use the current financial resources measurement focus. Choice "1" is incorrect. Governmental funds (GRSPP) use the current financial resources measurement focus, not the "working capital" measurement focus. No "working capital" measurement focus exists within governmental accounting. Choice "2" is incorrect. Governmental funds (GRSPP) use the current financial resources measurement focus, not the "cash" measurement focus. No "cash" measurement focus exists within governmental accounting. Choice "3" is incorrect. Governmental funds use the current financial resources measurement focus, not the economic resources measurement focus. The economic resources measurement focus is used for the proprietary (SE) and fiduciary (CIPPOE) funds as well as the government-wide presentation.

Interfund transfers received by a governmental-type fund should be reported in the Statement of Revenues, Expenditures, and change in Fund balance as a (an); a. Reimbursement b. Addition to contributed capital c. Addition to net assets d. Other financing source

Choice "4" is correct. Governmental funds report interfund transfers as other financing sources (uses). Choice "1" is incorrect. Interfund transfers are recognized on the face of the financial statements while reimbursements are not. Choice "2" is incorrect. Contributed capital is not recorded in governmental fund financial statements. Choice "3" is incorrect. Governmental-type funds do not report net assets.

Under Regulation S-X, an entity's interim financial statements filed with the SEC should include all of the following, except: a. A balance sheet as of the end of the preceding fiscal year b. An income statement for the period between the end of the preceding fiscal year and the end of the most recent fiscal quarter c. An income statement for the cumulative 12 month period ending during the most recent fiscal quarter d. A statement of cash flows for the most recent fiscal quarter

Choice "4" is correct. Interim financial statements filed with the SEC would not include a statement of cash flows for the most recent fiscal quarter, but should include statements of cash flows for the period between the end of the preceding fiscal year and the end of the most recent fiscal quarter, and for the corresponding period for the preceding fiscal year. The financial statements may also present statements of cash flows for the cumulative 12 month period ended during the most recent fiscal quarter and for the corresponding preceding period. Choice "1" is incorrect. Interim financial statements filed with the SEC should contain balance sheets as of the end of the most recent fiscal quarter and as of the end of the preceding fiscal year. A balance sheet for the corresponding fiscal quarter for the preceding fiscal year is not required unless necessary to understand the impact of seasonal fluctuations. Choice "2" is incorrect. Interim financial statements filed with the SEC should contain income statements for the most recent fiscal quarter, for the period between the end of the preceding fiscal year and the end of the most recent fiscal quarter, and for the corresponding periods of the preceding fiscal year. The financial statements may also include income statements for the cumulative 12 month period ended during the most recent fiscal quarter and for the corresponding preceding period. Choice "3" is incorrect. Interim financial statements filed with the SEC should contain income statements for the most recent fiscal quarter, for the period between the end of the preceding fiscal year and the end of the most recent fiscal quarter, and for the corresponding periods of the preceding fiscal year. The financial statements may also include income statements for the cumulative 12 month period ended during the most recent fiscal quarter and for the corresponding preceding period.

Which of the following is an example of a transaction involving a market participant? a. A judge orders a company to sell machinery during a bankruptcy proceeding b. A company purchases a commercial rental property from a company that is owned by the same shareholders c. A company sells land to a local government to satisfy an outstanding lien d. A company purchases real estate zoned for recreational use

Choice "4" is correct. Market participants are buyers and sellers acting in their economic best interests who are independent (not related parties), who are knowledgeable about an asset or liability, and are willing and able to transact for that asset or liability. A company that purchases real estate zoned for recreational use would be considered a market participant, because it meets the criteria identified above. Choice "1" is incorrect. A judge-ordered sale resulting from a bankruptcy proceeding does not involve a market participant. Choice "2" is incorrect. This transaction does not involve a market participant, because these companies would be considered related parties. Choice "3" is incorrect. Selling land to the government to satisfy a tax lien does not involve a market participant.

A company has 10,000 shares of common stock issues and 2,000 shares of treasury stock. The par value of the stock is $10 per share. On January 1, Year 1, the company declared a 5 percent dividend to be paid in cash on June 30, Year 1. What journal entry should the company record on the declaration date? a. Debit retained earnings for $5,000 and credit dividends payable for $5,000 b. Debit retained earnings for $5,000 and credit dividends payable for $5,000 c. Debit dividends expense for $4,000 and credit dividends payable for $4,000 d. Debit retained earnings for $4,000 and credit dividends payable for $4,000

Choice "4" is correct. The accounting treatment of a cash dividend involves a debit to retained earnings and a credit to dividends payable. Dividends are not paid on treasury stock, so the number of shares receiving the cash dividend are 10,000 − 2,000 = 8,000. 8,000 shares × $10 par × 5% = $4,000. DR Retained earnings$4,000 CR Dividends payable $4,000 Choice "1" is incorrect. Dividends are not paid on treasury stock. Choice "2" is incorrect. Debiting retained earnings is the right entry, not dividends expense. Also, dividends are not paid on treasury stock. Choice "3" is incorrect. Debiting retained earnings is the right entry, not dividends expense.

It is inappropriate to record depreciation expense in a (an): a. Internal service fund b. Private purpose trust fund c. Enterprise fund d. Capital projects fund

Choice "4" is correct. The capital projects fund is a governmental fund. The measurement focus of governmental funds is the flow of financial resources. Because depreciation expense does not reflect the use of financial resources, it is not recorded in the capital projects fund. Choice "1" is incorrect. An internal service fund is a proprietary fund that uses full accrual accounting and has capital maintenance as its measurement focus. Such funds record depreciation expense. Choice "2" is incorrect. A private purpose trust fund is a fiduciary fund that uses full accrual accounting and has capital maintenance as its measurement focus. Such funds record depreciation expense. Choice "3" is incorrect. An enterprise fund is a proprietary fund that uses full accrual accounting and has capital maintenance as its measurement focus. Such funds record depreciation expense.

A city's water division generated $1.5 million in revenue. It reported expenses of $1 million, which included $200,000 paid to an internal service fund. The water division also transferred $50,000 to the general fund. What amount is the water division's change in net position on the statement of revenues, expenses, and changes in net fund position? a. $500,000 b. $250,000 c. $300,000 d. $450,000

Choice "4" is correct. The change in fund net position for a proprietary fund would include all nominal activity recorded on the statement of revenues, expenses, and changes in fund net position. The water division would compute the change in net position as follows: Revenue$ 1,500,000Expenses(1,000,000) Income (loss) before contributions and transfers 500,000Transfers (50,000) Change in net position $ 450,000 Choice "1" is incorrect. The change in net position of the water division is not the income (loss) before contributions and transfers. Transfers must be deducted from income (loss) before contributions and transfers to compute the change in net position. Choice "2" is incorrect. The change in net position for the water division is not the sum of the internal service fund charges and transfers as proposed by this choice. Choice "3" is incorrect. The change in net position of the water division is not the income (loss) before contributions and transfers net of the amounts charged by the internal service fund. Internal service fund charges are appropriately included in the expenses of the water division and need not be considered twice. In addition, transfers must be deducted from income (loss) before contributions and transfers to compute the change in net position.

Harland County received a $2,000,000 capital grant to be equally distributed among its five municipalities. The grant is to finance the construction of capital assets. Harland had no administrative or direct financial involvement in the construction. In which fund should Harland record the receipt of cash? a. Private purpose trust fund b. Special revenue fund c. General fund d. Custodial fund

Choice "4" is correct. The fact pattern describes a transaction in which Harland County is collecting and holding cash temporarily on behalf of the cities within its borders. Transactions of this type are handled in custodial funds. Choice "1" is incorrect. The private-purpose fund would account for trust arrangements that are not better met by other fiduciary fund types. The custodial fund is preferable given the pure conduit arrangement described by the fact pattern. Choice "2" is incorrect. Although grants are frequently accounted for in special revenue funds, the use of a special revenue fund anticipates the County would have responsibility for grant compliance. This cash conduit arrangement is best suited for a custodial fund since the money is purely a passthrough arrangement. Choice "3" is incorrect. The general fund accounts for most of the general governmental activities of a governmental unit and is the default classification for transactions not otherwise appropriately handled in some other fund. The grant is clearly best handled in a custodial fund, thus the general fund is not appropriate.

Roy City received a gift, the principal of which is to be invested in perpetuity with the income to be used to support the local library. In which fund should this gift be recorded? a. Special revenue fund b. Private-purpose fund c. Investments trusts fund d. Permanent fund

Choice "4" is correct. The gift will be accounted for in a permanent fund. Permanent funds are used to report resources that are legally restricted to the extent income, and not principal, may be used for purposes supporting the reporting government's programs for the benefit of the public. The library is to be supported by a gift endowment and the library is intended to support the needs of the general community, not a specifically identified individual. Choice "1" is incorrect. Special revenue funds account for revenues from specific taxes or other earmarked sources that (by law) are restricted or committed to finance particular activities of a government. Roy City is accounting for income from a gift that is not appropriate for accounting within a special revenue fund. Choice "2" is incorrect. Private purpose trust funds are fiduciary relationships for the benefit of specific individuals, private organizations or other governments that do not meet the definition of a pension or investment trust fund. The library to be benefitted by the endowment is for the general public, not specific private individuals. Choice "3" is incorrect. Investment trust funds are used to account for external investment pools. An endowment contributed in support of a library to benefit the public is not an external investment pool.

When purchasing a bond, the present value of the bond's expected net future cash inflows discounted at the market rate of interest provides what information about the bond? a. Interest b. Yield c. Par d. Price

Choice "4" is correct. The issue price of a bond is a function of two different cash flows, one a lump sum and the other a regular stream of payments. First, we need to present value the eventual return of principal using present value tables and factors at the market or effective rate of interest, also known as the yield. Second, we need to present value the regular interest payments, whether annual or semiannual, using present value of an annuity tables and factors at the same market or effective interest rate (yield). Remember that the face rate of interest only has one job, and this is to compute the regular interest payments. It's the market rate of interest that will determine the bond's selling price. Choice "1" is incorrect. Interest will be paid semiannually (usually) or annually, and this interest must be present valued at the market rate along with the bond principal in order to come up with the bond price. Choice "2" is incorrect. The yield is another term for market or effective interest rate. It is this rate that will be used when using the PV table and the PV annuity table to determine the issue price of the bond. Choice "3" is incorrect. The par value of the bond is also known as the maturity value since this is the amount that must be paid back at maturity.

On July 2, Year 1, Wynn, Inc., purchased as an available-for-sale security a $1,000,000 face value Kean Co. 8% bond for $910,000 plus accrued interest to yield 10%. The bonds mature on January 1, Year 7, and pay interest annually on January 1. On December 31, Year 1, the bonds had a market value of $945,000. On February 13, Year 2, Wynn sold the bonds for $920,000. In its December 31, Year 1, balance sheet, what amount should Wynn report for available-for-sale investments in debt securities? a. $945,000 b. $950,000 c. $920,000 d. $910,000

Choice "4" is correct. The security would be recorded at fair value on July 2, Year 1, or $910,000. Accrued interest is a receivable and does not affect cost. The $90,000 discount is not amortized on short-term investments. On December 31, Year 1, the investment would be adjusted to fair value, $945,000. The unrealized holding gain of $35,000 would be reported as a separate component of other comprehensive income. Choice "2" is incorrect. The accrued interest of $40,000 at 12/31/Year 1 would be recorded as interest receivable, not as part of the investment account. Choice "3" is incorrect. The investment would be recorded at cost on July 2, Year 1, or $910,000. However, the investment would reflect fair value as of December 31, Year 1. Choice "4" is incorrect. $920,000 reflects the fair value of the investment on the date it was sold, not 12/31/Year 1. The investment is short-term.

A city government reported a $9,000 increase in net position in the motor pool internal service fund, a $12,000 increase in net position in the water enterprise fund, and a $7,000 increase in the employee pension fund. The motor pool internal service fund provides service primarily to the police department. What amount should the city report as the change in net position for business-type activities in its statement of activities? a. $28,000 b. $21,000 c. $9,000 d. $12,000

Choice "4" is correct. The statement of activities is the government-wide financial statement used to report revenues and expenses for governmental and business-type activities. The only fund type that will fall in the "business-type" classification is the enterprise fund, which in this example is the water enterprise fund. Because the net position of the water enterprise fund increased by $12,000, this amount will be reported under business-type activities in the statement of activities. The $9,000 for the motor pool internal service fund will be reported under "governmental activities" and the $7,000 change in fiduciary net position for the pension fund will be excluded from government-wide presentations. Fiduciary fund activity is only displayed in the fund financial statements. Choice "1" is incorrect. This choice incorrectly includes the $9,000 for the motor pool internal service fund and the $7,000 for the pension fund as business-type activities. Choice "2" is incorrect. This choice incorrectly includes the $9,000 for the motor pool internal service fund as a business-type activity. Choice "3" is incorrect. This choice incorrectly includes the $9,000 for the motor pool internal service fund as a business-type activity and excludes the $12,000 for the water enterprise fund.

On November 30, Year 1, Parlor, Inc. purchased for cash at $15 per share all 250,000 shares of the outstanding common stock of Shaw Co. At November 30, Year 1, Shaw's balance sheet showed a carrying amount of net assets of $3,000,000. At that date, the fair value of Shaw's property, plant and equipment exceeded its carrying amount by $400,000. In its November 1, Year 1, consolidated balance sheet, what amount should Parlor report as goodwill under U.S. GAAP? a. $400,000 b. $0 c. $750,000 d. $350,000

Choice "4" is correct. Under U.S. GAAP, goodwill is the difference between the fair value of the subsidiary of $3,750,000 ($15 × $250,000) and fair market value of the net assets acquired, $3,400,000. $3,750,000 − $3,400,000 = $350,000. Choice "1" is incorrect. Goodwill is the difference between the fair value of the subsidiary and fair market value of the net assets acquired, not the difference between carrying value and the fair market value of the assets acquired. Choice "2" is incorrect. Goodwill is recorded in the acquisition. It is the fair value of the subsidiary less the fair market value of the net assets acquired. Choice "3" is incorrect. Goodwill is the difference between the fair value of the subsidiary and the fair market value of the net assets acquired, not the carrying amount.

On December 31, an entity analyzed equipment with a net carrying value of $250,000 for impairment. The entity determined the following: Fair value $215,000 Undiscounted future cash flows $240,000 What is the impairment loss that will be reported on the December 31 income statement under U.S. GAAP? a. $10,000 b. $0 c. $25,000 d. $35,000

Choice "4" is correct. Under U.S. GAAP, impairment analysis begins with a test for recoverability in which the net carrying value of the asset is compared to the undiscounted cash flows expected from the asset. If the net carrying value exceeds the undiscounted cash flows, then an impairment loss is recorded equal to the difference between the carrying value and fair value of the asset. In this problem, the carrying value of $250,000 is greater than the undiscounted future cash flows of $240,000, so an impairment loss must be recorded. The impairment loss is calculated as follows: Impairment loss = Fair value − Carrying value = $215,000 − $250,000 = $(35,000) Choice "1" is incorrect. The impairment loss is not equal to the difference between the undiscounted future cash flows and the carrying value of the equipment. The impairment loss is equal to the difference between the fair value and carrying value of the asset. Choice "2" is incorrect. An impairment loss must be recorded because the carrying value of $250,000 is greater than the undiscounted future cash flows of $240,000. Choice "3" is incorrect. The impairment loss is not equal to the difference between the undiscounted future cash flows and the fair value of the equipment. The impairment loss is equal to the difference between the fair value and carrying value of the asset.

Gei Co. determined that, due to obsolescence, equipment with an original cost of $900,000 and accumulated depreciation at January 1, Year 9, of $420,000 had suffered permanent impairment, and as a result should have a carrying value of only $300,000 as of the beginning of the year. In addition, the remaining useful life of the equipment was reduced from 8 years to 3. Assuming straight-line depreciation, what amount should Gei report as accumulated depreciation in its December 31, Year 9, Balance sheet? a. $520,000 b. $600,000 c. $100,000 d. $700,000

Choice "4" is correct. When a permanent impairment occurs, the book value is reduced and a loss is recorded. The loss is credited to accumulated depreciation. In addition, the current year's depreciation expense should be added. The new book value is depreciated over the new life. Accumulated depreciation, 1/1/Year 9 420,000 Loss ($900,000 − 420,000) − 300,000 180,000 Depreciation for Year 9 ($300,000 / 3) 100,000 Accumulated depreciation, 12/31/Year 9 700,000 Choice "1" is incorrect. The $180,000 loss is added to accumulated depreciation. Choice "2" is incorrect. The $600,000 is the accumulated depreciation as of the date of impairment, 1/1/Year 9. The question asks the balance on 12/31/Year 9. Choice "3" is incorrect. The $100,000 is the depreciation expense for Year 9. The question asks for accumulated depreciation balance on 12/31/Year 9.

During the first quarter of the calendar year, Worth Co. had income before taxes of $100,000, and its effective income tax rate was 15%\. Worth's effective annual income tax rate for the previous year was 18%. Worth expects that its effective annual income tax rate for the current year will be 16%. The statutory tax rate for the current year is 21%. In its first quarter interim income statement, what amount of income tax expense would Worth report? a. $21,000 b. $15,000 c. $18,000 d. $16,000

Choice "4" is correct. When preparing interim financial statements, income tax expense is estimated each quarter using the effective tax rate expected to apply to the entire year. Choice "1" is incorrect. Worth should use the effective annual tax rate, not the statutory tax rate. Choice "2" is incorrect. Worth should use the effective annual tax rate, not the effective tax rate for the quarter only. Choice "3" is incorrect. Worth should use the effective annual tax rate expected to apply to the current year, not the prior year's effective tax rate.


Related study sets

ELA10 Structure and Suspense Exam

View Set

The War of 1812 4:The New Republic

View Set

Old Testament Survey Quiz Nehemiah & Ezra

View Set

AP Environmental Science Chapter 19 Vocab

View Set

Declaration of Independence Facts

View Set